寄托天下
楼主: weidancn
打印 上一主题 下一主题

[资料分享] 可直接打印的242道argument题目及分析 [复制链接]

Rank: 2

声望
9
寄托币
139
注册时间
2009-4-2
精华
0
帖子
28
16
发表于 2009-4-26 22:09:45 |只看该作者
121.The following is a memo from the president of Cyberell Computer Company.
    "All of our customer-service employees recently attended a two-day retreat during which they received retraining in effective customer service. Subsequently, Cyberell's employee-performance study showed that the retreat benefited new employees-those who have worked with Cyberell for less than two years-far more than it did experienced employees. According to the study, after the retreat new employees were able to handle an average of ten percent more calls per hour, and the total number of customer complaints about new employees decreased, but experienced employees showed little improvement in these areas. Therefore, Cyberell should send only new employees to future retreats and should use the resulting savings to double the length of the retreats so that the retreats will be more likely to yield optimum employee performance."
     我们所有的客户服务人员最近参加了一次为期两天的进修,进修期间他们接受了关于有效的客户服务方面的再训练。结果,Cyberell的员工绩效考察发现进修对于新员工--即那些在Cyberell工作不到两年的员工--的作用远大于对老员工的作用。根据这次考察,进修之后新员工平均每小时能够多应答10%的电话,而客户对于新员工的投诉总量下降了,但老员工在这些方面的提高很小。因此,Cyberell应该只把新员工送去参加未来的进修,并应该把因此而节省的开支用于把进修的时间加倍,这样的话进修将会导致更好的员工绩效

customer-service employees,retreat 论断:C今后应该只送新员工去培训,并用节省下来钱加长培训的时间,这样培训就可能产生更好的员工表现。因为培训对新员工的作用比对老员工的大。根据研究,培训后新员工每小时能够解决的投诉电话比以前多了10%,而且对新员工的投诉数目也有下降,但是老员工只有很小的提高。 论者根据新老员工在教练后的不同提高程度提出的建议乍一看的确很有道理,但仔细想想,论者忽略了很多因素,得出来的结论很武断。
        这一次的培训对老员工的提高不大,只能说明这一次的培训内容不适合老员工,但并不说明老员工就不需要培训。这次C把新老员工送去参加同样的培训,有可能培训主要解决的是经验问题,即把老员工在工作中掌握的经验传播给新员工,所以会出现新员工长进快,而老员工没有变化。但是C仍然可以增设一些针对有经验的员工的培训内容,比如培养他们的创新能力,管理能力等等。
        论者用新员工和老员工作为区分标准不合适,因为工作的表现更多为有能力和没能力。也许老员工中仍然有一些没能力的,虽然为数不多,不会影响统计,但对他们的培训仍然能够提高表现。
        评判新员工表现进步的标准不合适。处理电话多就证明表现提高了吗?如果这段时间里电话里顾客提出的问题都是很容易处理的,或是新员工在电话里敷衍顾客,都无法证明培训提高了员工的表现。另外,对新员工的投诉减少,也有可能是其它原因造成的,比如业务总量的下降,比如这段时间的顾客都不太挑剔等。总之,要了解员工的表现是不是有所提高,还需要了解顾客是不是比以前更满意了,以及他们是否能更好地处理问题。
        关于培训时间是否需要加长,论者的结论有些武断,因为培训的效果与培训的时间长短没有必须的成正比的关系。如果培训仅仅是解决工作中遇到的经验问题,也许像这次这样两天就足够了。而如果是一些更高层次的培训,也许需要的时间会长一些。
结论:论者的推论是基于错误的前提和标准得出来的,论断的结论也很武断。要设计将来的培训,还是应该针对员工的特点进行,而不应该仅用工作时间来决定。

122.The following appeared in the Pine City Gazette.
"Fifteen years ago, Pine City launched an electricity-conservation program that reimbursed residents some of the cost for replacing energy-wasteful motors, home office equipment, and home appliances with energy-efficient ones. For ten years, spending on this program increased annually, and annual total energy consumption declined. But spending on the program began to decline five years ago, and since then Pine City's total electricity consumption has increased sharply. If this increased usage continues, the city will have to build a costly new power plant. Obviously the best way to avoid this expense is to increase reimbursement to residents for replacing energy-wasteful equipment. This will reduce energy usage to the levels of five years ago."
    15年前,Pine City实施了一项节电措施,该措施给予市民一定的补偿用于把能耗大的电机、家用办公设备和家用电器更换为节能的型号。在开始的十年,在这项措施上的支出每年都在增加,而每年的总用电量下降了。但在这项措施上的支出在五年前开始减少,并且从那以后Pine City的总用电量急剧上升。如果这种用量上升的势头持续下去,城市将必须修建一座昂贵的新的发电厂。显然避免这项开支的最好办法就是增加对居民更换大能耗设备的补偿。这将把能源消耗降低到五年前的水平。

Pine City,electricity-conservation program 论断:避免开支的最佳途径是提高对居民更换费电设备的补偿,它将使能源使用降低到五年前的水平。15年前松村城开始了一项省电工程,即补偿居民用省电设备更换费电的旧设备。十年里,这项开销每年都在增长,而每年的用电总量在降低。但是五年前这项开销开始下降,而用电总量一直在猛增。如果这种增长继续,该城将必须再花钱建一个新的电厂。
        被忽略的前提,即松树城居民使用的设备是否还有费电的。该城这个项目进行了十五年,很有可能目前居民使用的都已经是节电的装备。而从五年前该项目上花费的降低,也许正是因为居民需要更换的设备已经不多了。如果没有要换的设备,提高补偿也不会降低用电量。
        不成立的前提,即用电量的增长是因为费电设备。很多因素都会导致用电上升,比如人口增加,电脑的普及,新的用电设备的诞生,生活条件的提高,这些都使得用电上升成为必然。另外工业的发展,公共设施增加,也都会带来用电量的上升。这些用电的上升都会抵消费电设备减少的作用。
        被忽略的前提,即松树城的电量供应不够了。论者没有提供关于该城面临供电紧张的资料,没有根据的提出用电量增长,将会需要再建一个电厂。但如果目前松树城的供电情况很好,这个需要将不会存在,政府也就没有必要提高补偿。
        结论武断,如果提高补偿的开支甚至超出了建新厂的开支,就不好了,更谈不上最好。
结论:论者忽略了十几年的时间里该城发生的变化,仍然想用以前的方法解决目前的问题,必须会导致决策上的失误。在做出决定之前,该城还是应该再考虑一下目前的供电能力,并找出节约用电的有效办法。

123.The following appeared in a health newsletter.
    "Eating a heavy meal may increase the risk of heart attack. A recent survey of 2,000 people who had had a heart attack revealed that 158 of them said they had eaten a heavy meal within 24 hours before their heart attack, and 25 of them said they had eaten a heavy meal within 2 hours before their heart attack. Eating and digesting food releases hormones into the bloodstream and temporarily increases heart rate and blood pressure slightly. Both of these things put stress on the heart. Therefore, people who are at risk of having a heart attack can lower that risk by not overeating."
    每顿饭食量过大会增加患心脏病的危险。最近一项对于2000名曾经患心脏病的人调查发现,其中有158人说在患心脏病前的24小时之内吃过一顿量很大的饭,其中有25人说在患心脏病前两小时之内吃过量很大的饭。食用和消化食物将会向血液中释放荷尔蒙,并暂时轻微增加心率和血压。这些都会增加心脏的压力。因此,那些有患心脏病危险的人可以通过避免饮食过量而减少这种危险。

heavy meal,heart attack 论断:别吃多了,能让有心脏病的人降低发作的危险。因为吃多了会增加心脏病发作的危险。最近一份对2000个有过心脏病发作的人的调查里,158个人说他们在发作前24小时里吃了大餐,而25个人说在两小时前吃过大餐。因为吃饭和消化食品会往血液里释放荷尔蒙,并会使心率和血压有暂时小量的提高。这些都对心脏有压力。
        不一定成立的前提,即吃多了会增加心脏病发作的危险。论者没有提供直接的资料证明存在有因为吃多了引发的心脏病。虽然吃饭和消化食品会给心脏带来压力,但没有资料证明这样的压力有可能足够引发心脏病发作。
        不一定成立的前提,即有人因为吃多了引发了心脏病。论者为此提供的调查没有说服力。首先就心脏病这样一种患者众多的病症来说,2000人的调查取样实在太少,这样158这个数字也就没有代表性。另外调查没有排除这158个人因为其他问题引发心脏病发作的可能,比如这期间进行了剧烈的运动,比如受到了精神、感情上的强烈冲击,这些都会引发心脏病。不排除这些,我们无法得出结论是吃多了使心脏病发作。
        结论推理有错误。有很多事情都会使得心脏病可能发作,所以即使吃多了会提高心脏病发作的危险,也无法保证不吃多危险就会降低。就像看电视能开拓我们的眼界,但是不看电视我们的眼界不会因此缩小。
结论:论者的推论中存在逻辑错误,而且没有坚实的资料基础,论断没有说服力

124.The following is a memo from the business manager of National Daily News.
     "To expand the home delivery service of our national newspaper, we should concentrate on the state of Urba rather than on the state of Sylva. First, the population of Sylva is more widely dispersed, which would require us to spend more money to deliver our papers in that area, resulting in less profit per customer. Second, a long-term study of television viewing habits suggests that Sylvans prefer local to national news, since they spend twice as much time viewing local news programs as they do viewing national programs. Finally, because events in Urba receive more coverage in our newspaper than do events in Sylva, we can expect Urbans to be more interested in reading our newspaper."
     为扩展我们国家报纸的家庭递送服务,我们应该集中力量于Urba州而不是Sylva州。首先,Sylva的人口分布得更加分散,这要求我们在这个州递送报纸的时候要花更多的钱,从而导致每个客户的平均利润减少。其次,一项长期的对于观看电视的习惯的研究表明,Sylva的居民相对于国家新闻而言更喜欢看地方新闻,因为他们用于观看地方新闻节目的时间是观看国家节目的两倍。最后,由于Urba州的事件在我们报纸上所占的版面要多于Sylva的事件,我们可以预计Urba的居民对于我们的报纸会更感兴趣。

home delivery service,national newspaper 论断:扩大投递服务,我们应该把重点放在U州而不是S州。因为首先S州的人口太分散,这使得我们要花更多的钱去投递,这会降低每个顾客的利润。第二,一项对S电视收视习惯的长期调查显示,S人更愿意看当地新闻而不是全国新闻,因为他们花在当地新闻节目上的时间是全国节目的两倍。第三,我们的报道中有关U的比有关S的更多,U可能对我们的报纸更感兴趣。
        被忽略的因素,扩大投递的目的是为了增加报纸的利润,所以利润潜力大的地区,应该成为重点。在报纸经营的利润中,广告收入远比发行收入更大,所以在选择重点地区的时候,应该看重的是该地区可能给报纸带来广告收入的潜力。在这个问题上,当地居民的人口,收入,受教育程度,购物消费习惯都是很重要的因素,而论者却没有提供有关的资料,我们也无法判断是否U比S更值得我们去扩大投递服务。
        不一定成立的前提,S的投递成本比U高。分散的居住并不一定意味着高的投递成本,如果S的交通比U更发达,费用更低,都会使得在该城的投递不一定高出U。另外分散的居住意味着城市增容的可能,即S的人口将来有可能比U增长快。这样都使得我们在S每位顾客上的利润会比U少。
        不一定成立的前提,即S地区喜欢看当地新闻,不看全国新闻。这个结论论者是从对S的电视收视习惯中得出的。虽然这个电视新闻与报纸新闻有一定的可比性,但是论者却在比较中忽视了一个很要的因素,就是新闻报道的水平。当地居民的收视倾向有可能是因为当地有较高的电视新闻制作水平,而不能证明他们不关心全国新闻。所以如果我们的报纸吸引人,就能够吸引S地区的居民读全国新闻。而且S地区全国新闻提供的不足,也使我们报纸会较容易取得效果。另外,电视节目的观众与报纸的读者并不完全相同,这也使得这份调查不能反映读者对于报纸的选择。
        不一定成立的前提,U地区居民比S地区居民对我们的报纸更感兴趣。论者说我们的报道中有关U的比S的多,但这样的情况不是僵化不变的。首先我们完全可以增加对S地区的报道,以吸引他们看我们的报纸。其次扩大在该地区的发行,也将使也使得当地居民与我们报纸的记者有更多的接触,从而使得记者的报道中更多地出现有S地区有关。
结论:论者的推断忽略了措施的最终目的,这使得他必然不能得出正确的结论。其次论者武断地强调在S地区的困难,却没有意识到这些困难同时也是S地区的优势。在作出决定之前,论者还应进一步了解两地的具体情况,以及相应的解决办法。

125.The following is a recommendation from the president of Appleby College.
      "Whereas Appleby College holds class reunions every five years, Edelston College holds annual reunions for all classes, during which Edelston's alumni are treated to banquets, lectures, and student performances, enhancing their loyalty to the college and their willingness to donate money. Edelston College receives most of its alumni donations during or shortly after these reunions. Therefore, the best way for Appleby to increase its alumni donations is to offer similar reunion activities and to have each graduating class hold annual reunions."
     Appleby学院每五年举行一次班级同学聚会,而Edelston学院的所有班级每年举行聚会,并且在聚会上Edelston的校友可以参加宴会、讲座和学生表演,这提高了他们对学校的感情以及他们给母校捐款的愿望。Edelston学院的大部分校友捐款是在聚会期间或之后很短时间内获得的。因此,Appleby增加校友捐款的最好途径就是组织类似的聚会活动,并且让每个毕业班级组织一年一度的聚会。

class reunions,Appleby College 论断:提高A校校友捐献的最好办法就是提供和E校一样的校友聚会活动,并让每个毕业班每年都举行聚会。因为A校每五年举行一次班集聚会,而E校每年都有。在聚会上E校校友被宴请,讲座还有学生表演,这提高了他们对大学的忠实度,使得他们愿意捐钱。E校在这些聚会中或聚会后都会收到大多数校友的捐赠。 论者的结论完全从A与E的比较中得出,但是它的比较却忽略了很多问题。
        A校校友的捐赠是否就比E少,论者没有提供具体两校收到的校友捐赠数额,我们无法肯定A校一定少于E校。虽然E校每年聚会后都会收到校友的捐赠,但是次数多不一定金额就会多。有可能A校校友捐赠的次数少,但每笔都是大数呀。
        A校校友是否还能捐出更多的钱来。论者没有提供有关两校学生毕业后的经济状况,有可能E校是商学院,出来的校友经商赚钱多,而A校培养的都是文化人,属清贫一族。
        是否举行聚会A校校友一定能参加。如果A校学生毕业后很分散,五年聚一次已经不容易了,那么每年一次就更不能保证。所以如果没人能来参加,举行也没有用。
        要校友捐钱不就为了让学校腰包鼓起来嘛,如果E校每年搞的聚会花销比校友捐的钱还多,这种为了拿捐钱的聚会还不如不搞。
结论:论者没有考虑到两校的差别,进行的比较很荒谬。而且为了让校友捐钱而搞聚会只能破坏校友对学校的忠实,从而可能影响学校的声誉。这样的建议要不得呀。

126.The following appeared in a health magazine.
     "It has long been believed that getting at least eight hours of sleep a night is good for health. But a recent six-year study of adult sleeping habits found that people who reported sleeping eight or more hours a night had a higher rate of certain health problems than did those who reported sleeping seven hours a night. People who reported sleeping five hours a night also had an increased rate of the health problems, but this increase was slight compared to that for people who reported sleeping eight or more hours a night. Clearly, people should try to get seven hours of sleep, and they should worry more about getting too much sleep than too little."
      长期以来人们相信每天睡眠至少8小时是有利于健康的。但是最近一次对于成年人睡眠习惯的为期6年的研究发现,那些报告说每天睡眠8小时或以上的人某些健康问题的发生率比那些报告说每天睡7小时的人高。报告说每天睡5小时的人健康问题发生率也高,但和报告8小时睡眠的人相比比较轻微。显然,人们应该尽量每天睡7小时,并且相对于睡眠不足来说,他们更应该避免睡眠过量。

eight hours of sleep 论断:人们应该尽可能每天睡七个小时,而且睡多了比睡少了更应让人担心。这是最近一份长达六年的成人睡眠习惯里,人们报告的睡眠时间以及健康问题频率的总结。
        前提不一定成立,每天七个小时睡眠好,但没有提供直接的资料证明。
        论据不足信,关于睡七个小时、长于八个小时和五个小时的比较,论者都是从这份研究里得出来的,但这份研究存在一些问题,让人怀疑他的准确性。 首先,调查的样本人群是多少,年龄、性别、职业各有什么样的分布,论者没有提供。这样这份研究结果是否具有代表性值得怀疑。如果它只是个别情况,就可能对其他人不适合。 其次,三组人除了睡眠时间以外,其他身体特征是否相同。因为存在有可能身体不好的人才会睡得多,另外他们的饮食习惯、工作特征都会影响身体,这样睡眠就不是影响他们身体状况的主要原因。
        结论武断。首先论者忽略了其他人群的需要,比如儿童、老人,他们的身体都和中年人不一样,所需要的睡眠时间也是不同的。其次,人们每天活动的强度会影响身体对睡眠的需要,有时每天都不一样,人和人更应区别对待。
结论:论者的资料不足以证明他所引用的研究具有准确性,而且他的结论也过于武断。睡眠时间还是因人不同。

127.The vice president for human resources at Climpson Industries sent the following recommendation to the company's president.
     "In an effort to improve our employees' productivity, we should implement electronic monitoring of employees' Internet use from their workstations. Employees who use the Internet from their workstations need to be identified and punished if we are to reduce the number of work hours spent on personal or recreational activities, such as shopping or playing games. By installing software to detect employees' Internet use on company computers, we can prevent employees from wasting time, foster a better work ethic at Climpson, and improve our overall profits."
     为提高我们的员工的生产效率,我们应该在员工的终端上加装电子监控来监视员工对互联网的使用。如果我们要减少用于私人以及娱乐活动,比如购物或玩游戏的工作时间,在终端上使用互联网的员工必须要被确认并且受到惩罚。通过在公司电脑上安装检测员工使用互联网的软件,我们可以防止员工浪费时间,并培养Climpson更好的工作氛围,以及提高我们的整体利润。

electronic monitoring of employees' Internet 论断:在公司的电脑上装监测员工因特网使用的软件,可以提高我们整体的利润。因为我们可以防止员工浪费时间,促进好的工作态度。如果想降低员工在工作中花在私事和娱乐活动上的时间,在工作间使用因特网的员工就应该被查出来被并处罚。(都是前提不成立)
        装软件可以防止员工浪费时。也许确实可以防止员工在上班期间使用因特网干私事和玩游戏,但不能保证员工通过其他途径的私事和娱乐时间不会上升。比如会有人不能在因特网上买东西了,他就会在上班时间溜出办公事出商场买。
        减少干私事可以促进好的工作态度和效率。论者没有提供目前员工的工作态度如何,私事和娱乐是否已经影响到了他们的工作效果。如果工作不是很多,员工少量的私事和娱乐也许不会影响他们的工作。但公司通过这种监测方式,监测员工的行为,倒有可能引起员工的不满,尽管不表达,这样有可能会使他们消极怠工。
        结论值得怀疑,装软件是否会影响电脑的运行,软件的花费的是多少,论者没有提供资料。如果电脑工作的速度会下降,软件的花费过大,也不能提高我们整体的利润。而且利润的上升牵扯到成本、业绩等等内容,不是工作态度好了,就可以上升的。
结论:论者没有资料证明目前员工的工作效率和态度已经影响到了公司的运行,所以我们无法判断是否有必要这种对员工行为的进行监测。而监测是否能够促进工作态度也值得怀疑,这样的建议还需要更多的考虑。

128.The following appeared in an editorial in the Garden City Gazette.
     "To address the parking problems in our downtown business district, it has been proposed that the city increase parking capacity by building a four-story parking garage. However, this project would cost more than it would to improve the downtown pedestrian plaza. Because the pedestrian plaza is an important attraction that draws people to the downtown area, improvements to it will increase business for downtown merchants. The merchants' higher profits will ultimately produce increased tax revenues for the city. Therefore, we should invest in the plaza improvements first and then use the revenues thus generated to pay for the construction of the parking garage."
     为解决我们闹市商业区的停车问题,曾有提案提出建造一幢4层停车楼来增加车位。然而,这一方案的花费要比改善闹市步行广场的花费多。由于步行广场是把人们吸引到闹市区的重要因素,对于它的改善将会促进闹市的商业。更多的商业利润将最终增加城市税收。因此,我们应该首先投资于广场改造,然后用它产生的收入来建造停车楼。

parking problems 论断:我们应该先投资改善商场,而且用这项收入去建停车库。因为建停车库的花销大过它可能给商场带来的好处。因为商场是吸引人们到市区来的重要因素,对于它的改善将增加市场区的商业。商业的高利润将最终给这个城市产生上升的税收收入。
        不一定成立的前提,投资改善商场将增加市区的商业。虽然商场市区的重要吸引,但是论者没有提供资料表明改善它必然会带来商机增加。因为如果目前的空间和购物环境很让人们和商人满意,那么改善以后它的吸引力并不一定会比现在提高。
        被忽略的前提,停车库的建设是否不着急。如果停车问题很严重,那么不方便的交通就会使人们不愿意来。这样即使商场改善了,来的人也不会多。相反,停车位的增加到有可能吸引更多的人和商机。
        停车场的花销大。或许停车场的确需要花费很大,但是这项花费完全可以从对停车位的经营中收回。所以停车场的建设不一定需要花费市镇收入。
结论:论者应该进一步了解目前人们对商场是否满意,交通问题有否影响到人们的选择。

129.The following appeared in the Sherwood Times newspaper.
     "A recent study reported that pet owners have longer, healthier lives on average than do people who own no pets. Specifically, dog owners tend to have a lower incidence of heart disease. In light of these findings, Sherwood Hospital should form a partnership with Sherwood Animal Shelter to institute an 'adopt-a-dog' program. The program would encourage dog ownership for patients recovering from heart disease, which will help reduce medical costs by reducing the number of these patients needing ongoing treatment. In addition, the publicity about the program will encourage more people to adopt pets from the shelter, which will reduce the risk of heart disease in the general population."
     一项最近的研究报告说养宠物的人平均而言比不养宠物的人活得更长更健康。特别是,养狗的人心脏病发病率更低。根据这些发现,Sherwood医院应该和Sherwood动物收养所合作建立一个"收养狗"的计划。这一计划将会鼓励正在治疗心脏病的患者养狗,这将通过减少这些患者接受治疗的次数而减少医疗费用。而且,对于这一计划的宣传将鼓励更多的人从收养所领养宠物,这将减少整体人群患心脏病的危险。

pet owners 论断:收养狗工程能降低医药费,并降低一般人群患心脏病的危险。因为最近一项研究报告宠物主人一般有更长和健康的生命,尤其是狗主患心脏病的数目更小。医院应该与AS建立合作收养狗工程。这个工程能在病人中提倡拥有狗来从心脏病中康复,这将减少病人来往治疗的次数从而帮助降低医药费。另外对这项工程的宣传也将鼓励更多的人从收养所收养宠物,这将降低一般人群患心脏病的危险。
        不一定成立的前提,收养狗能降低心脏病或是帮助康复。论者没有提供任何资料直接证明收养宠物对身体有什么样的好处,尤其是收养狗是否能帮助降低心脏病发生。要知道这是比较难以让人置信的,考虑到要多担心宠物,这是有可能增加人们心理压力的。
        论据不足信,论据引用最近一项研究,但研究仅说有狗的人中得心脏病的人少,但并没说养够使得人们患心脏病少。另外这项研究也存在一些值得怀疑的地方。首先,没有提供关于宠物主人的其他特征,比如年龄、职业、性别我们不能判断这项研究是否有代表性。其次,研究没有排除其他可能降低发病的可能,比如养宠物的人是否经济条件好,生活比较悠闲等等这些都会影响健康。要知道,有心情有能力关心宠物的人,一般都已经把自己照顾得很好了。
        结论,即使能够帮助心脏病人康复,医药费也不一定降低。比如收养所的宠物身上很可能带有细菌,它们会把病菌传染给病人,这样其它疾病的增多使得医药费不一定降低。同样,对这项工程的宣传也不一定会让更多的人去收养所收养宠物。有人不愿意养宠物,所以普遍下降不被保证。
结论:论者应该再进一步提供关于宠物如何或为什么可以帮助人们健康的资料,这样我们才能考虑是否应该开展这样的活动。

130.The following appeared in the annual report of a chain of stores that sell supplies for do-it-yourself home improvements.
    "Current reports indicate that sales of new homes nationwide declined by 9 percent during the past year, while sales of existing homes increased by 16 percent. Thus we can expect that the percentage of homeowners who will be making repairs or improvements to their homes will also increase. In addition, people who buy existing homes will have more money left over after the purchase to spend on home improvements. Finally, people who buy existing homes are more likely to make improvements themselves, as opposed to paying someone else. These factors combined indicate we can expect an increasing demand for products in all our stores nationwide, thus increasing our profits in the coming year."
    当前的报告指出全国的新住宅销量在去年下降了9%,而旧房销量增加了16%。因此我们可以预见将会修缮或改造其房屋的业主比例也会增加。而且,那些买旧房的人在购房后将会有更多的富余金钱用于房屋改造。最后,买旧房的人更可能自己进行改造,而不是雇人来做。这些因素说明我们可以预计我们全国的商店产品需求将会增加,因而我们来年的利润也会上升。

do-it-yourself home improvements 论断:我们可以期待全国店的货物需求上升,明年利润上涨。因为目前的报告显示全国范围新房的销售去年下降了9%,同时旧房销售上升了16%。这样房主中要进行修理我改善的比例会也上升。另外,买旧房的人有更多剩下的钱花去改善,而不愿意花钱让别人做。
        被忽略的前提,是否购买的旧房需要修理。没有证据表明旧买的状况都很糟糕,同样也没有证所表明买旧房的人都愿意对旧房进行维修。如果房主不对旧房进行改善,就不会需要他们的货物。
        没有根据的推论,买旧房的人有更多的钱改善。首先不买新房而买旧房有很有可能是因为房主本身没钱,买不起新房。其次,旧房不一定就比新房便宜,如果路段好又豪华,价格同样会很贵。
        凭空猜测。需要对旧房进行改善的人也不一定会自己动人,他们完全有可能雇别人来干。因为自己动手虽然花钱少,但是费时费力还可能干不好。
        结论武断,尽管旧房销售也有可能意味着对这类货物的需求上升,但由于质量、服务等原因,不一定会买你们的。而同时全国性的趋势如此,不代表处处如此,所有商店的结论不成立。
结论:论者没有了解房主买旧房的原因,也没了解他们对于改善和装修的看法,仅凭几个数据就得出结论,盲目乐观。

131.The following appeared in an environmental newsletter published in Tria Island.
    "The marine sanctuary on Tria Island was established to protect certain marine mammals. Its regulations ban dumping and offshore oil drilling within 20 miles of Tria, but fishing is not banned. Currently many fish populations in Tria's waters are declining, a situation blamed on pollution. In contrast, the marine sanctuary on Omni Island has regulations that ban dumping, offshore oil drilling, and fishing within 10 miles of Omni and Omni reports no significant decline in its fish populations. Clearly, the decline in fish populations in Tria's waters is the result of overfishing, not pollution. Therefore, the best way to restore Tria's fish populations and to protect all of Tria's marine wildlife is to abandon our regulations and adopt those of Omni."
    Tria岛的海洋生物禁猎区是用来保护某些海洋哺乳动物的。它禁止在Tria20英里以内倾倒废物和近海采油,但捕鱼并不禁止。当前Tria水域的很多种鱼类数量都在下降,这种现象被认为是污染造成的。相比之下,Omni岛的海洋生物禁猎区禁止倾倒废物、近海采油和在Omni10英里以内捕鱼,Omni没有上报任何鱼类数量的显著下降。显然,Tria水域鱼类数量的下降是过度捕鱼而不是污染造成的。因此,回复Tria鱼类数量和保护Tria所有海洋野生动物的最好办法就是废止我们的规定而采用Omni的规定。

marine sanctuary 结论:恢复鱼数目和保护T野生海洋生命的最好办法是放弃我们的规定采用O的。因为T地区很多鱼的数目前最近在下降,有人说是污染。但是O地区没有报告鱼群有明显下降。我们禁倒垃圾和近海20英里的探油,但不禁钓鱼。而O地区不仅禁我们禁的,还禁10英里内的钓鱼。很显然T地区鱼数的下降是因为过度捕渔。
        前提,鱼数下降一定是因为过度捕捞。论者没有提供任何资料证明T地区的捕鱼使得鱼群下降,既没有提供有关渔民数量,也没有提供这次鱼群下降的同时是否捕鱼量在上升。所以这个前提不一定成立。
        前提,鱼数下降不是因为污染。论者没有提供资料,证明近期T海域内水质没有发生变化,这样我们不能排除有污染存在,并导致了鱼群下降。虽然我们禁倒垃圾和20米的探油,但不排除垃圾仍在倒,油从20英里外的地方过来。
        前提,我们和O地区情况完全一样。但是地区不同,海域的鱼群种类不同,都使得O地区的经验不一定适合我们。如果说O地区在近海十公里内鱼群很多,而T地区十公里内根本没有鱼,那么这样的措施没有帮助。
        一定还有其他办法,不一定学人家嘛。干嘛非说最好
结论:论者还应该进一步收集资料,了解本地区鱼群下降的原因是什么。再做出决定。

132.The following appeared in an editorial in the local newspaper of Workville.
    "Workers should be allowed to reduce their workload from 40 to 25 or even 20 hours per week because it is clear that people who work part-time instead of full-time have better health and improved morale. One store in Workville, which began allowing its employees to work part-time last year, reports that fewer days of sick leave were taken last year than in previous years. In contrast, the factory in Workville, which does not allow any of its employees to work part-time, had a slight increase in the number of days of sick leave taken last year. In addition, a recent survey reports that most of the store employees stated that they are satisfied with their jobs, while many of the factory employees stated that they are dissatisfied with their jobs."
      应该允许工人把他们的工作量从每周40小时减少至25甚至20小时,因为显然那些部分时间工作而不是全时工作的人更加健康,精神更好。Workville的一个商店去年开始允许员工部分时间工作,它报告说去年的病假数量比前些年减少。相比之下,Workville的工厂不允许员工部分时间工作,去年它的病假数量有少量上升。而且,最近的一次调查报告说该商店多数雇员声称他们对于自己的工作满意,而很多工厂的员工声称对于自己的工作不满意。

reduce their workload 论断:工人的工作量应该被允许从每周40小时降到25甚至20个小时,因为人们兼职工作比全职业工作有更好的健康和高的士气。证据有三:1、W店去年开始让员工兼职工作,他们报告去年病假的天数发生少了。2、相反,W厂不允许任何职工兼职工作,他们去年病假的天数有了小量上升。3、另外,一项最近的调查报千大多数店里的员工说他们满意他们的工作,而很多厂里的员工说他们不满意他们的工作。 论者一切结论皆从店和厂的比较中得出,但是比较有错误之处。
        是否店的员比以前健康了。论者用的证据是病假天数少了。但是我们知道因为兼职的原故,员工整个上班时间减少了,病假天数减少是避然的,并不是身体健康的结果。完全有可能员在不上班的时候生病了,去了医院,但不用请病假了。
        是否厂的员工不健康了。首先病假天数只是小量上升,并不能说明今年的员工的健康状况就比去年糟糕,因为不排除有员工请病假只是借口。其次,今年厂里的工作环境是否不如以前,也会影响员工是否请病假。
        是否店员工比厂员工更满意目前的工作。对工作的满意包括薪水,工作压力,以及社会地位等问题,工作量不是最主要的。有可能店里的员工薪水比厂里的员工高。
        是否店和厂有可比性。两家单位工作性质完全不同,这种比较是否能得出有价值的结论值得怀疑。比如一般而言店里的工作没有厂里的对技术要求高,而且商店里的员工也比较少,这些都使得店的经验是否值得厂借鉴也不一定值得厂借鉴。
结论:论者没有任何资料证明每周20或25小时的工作量是最合理的,而所引用的证据又是建立在错误的对比上。论断没有说服力。

使用道具 举报

Rank: 2

声望
9
寄托币
139
注册时间
2009-4-2
精华
0
帖子
28
17
发表于 2009-4-26 22:10:59 |只看该作者
133.The following is a memo from the principal of Academia High School.
     "Academia High School should abolish its after-school performing-arts programs and replace them with computer-technology programs. When nearby Techno High School did so last year, total enrollment in all of its after-school activities remained about the same. Moreover, on entering college, many Techno students chose a major directly related to their after-school activities. On the other hand, last year only 10 percent of Academia's graduating seniors chose performing arts as their major field of study in their first year of college, clearly indicating that most students do not have a strong interest in the performing arts."
     Academia高中应该废止其课外表演艺术课程而应该以计算机科技课程取而代之。当附近的Techno高中去年采取了这种变化之后,其所有课外活动的参加数量基本没变。而且,很多Techno的学生在走进大学的时候选择了与他们所参加的课外活动直接相关的专业。另一方面,去年Academia的毕业生中只有10%的人选择表演艺术作为他们大学第一年的专业,这显然表明多数学生对于表演艺术并没有很强烈的兴趣。

after-school performing-arts 论断:A校应该取消它的课后艺术表演项目换成计算机科技项目。隔壁T校去年这么做了,他们整个课后活动的注册保持了。另外进入大学时,很T校学生选择与课后活动直接有关的专业。而A校去年只有10%的毕业生选择表演艺术作为他们在大学第一年的专业,很显然大多数学生对表演艺术没有很强的兴趣。
        不一定成立的前提,A校学生学生对于表演艺术没有兴趣而对计算机有兴趣。论断里没有学生的声音,论者可能根本没有问题过学生。没有学生说对表演没有兴趣。
        论据不足信,A校与T校的对比不能说明问题。首先,两校在师资、设备及学生等方面都有差异,比如T校在计算机教育上比A校有优势,而A校在表演艺术培训力量又比T校有经验,还有可能对计算机感兴趣的学生都选择去了T校,而选择A校的学生都是对文课有兴趣的,他们都对表演艺术而不是计算机更有兴趣。其次,论者选择的标准不合适,论者选用学生在大学里的专业选择作为学生是否对各自的课后活动项目有兴趣,这不准。因为选择专业学生更多地考虑的是今后的工作,比如薪水、就业机会等等,兴趣并不是他们选择专业的唯一或最主要标准。而选择电脑作专业而不是表演艺术,更可能是因为电脑的就业机会比表演艺术广,应用泛范大。而他们完全可能对表演艺术仍有浓厚的兴趣。毕竟课后活动是为培养学生的兴趣,而不是要对他们进行生存技能培训。
        结论:10%的A校学生选择了表演艺术作专业,即意味着他们将以此为今后的职业。考虑到表演艺术的就业机会比电脑小很多,这已经是一个不小的比例。这部分学生的要求不容忽视。
结论:论者在没有了解学生意见的情况下,是不可能知道学生到底对什么感兴趣。是否要改变课后活动的内容,论者还是应该听听他们的意见并综合本校的教师力量再做决定。

134.The following was taken from a guide for aspiring writers.
     "New writers usually cannot convince large, prestigious publishers that their work is marketable, so they must first publish with smaller, less well-known presses to establish a sales record. But the editors of Mystery Writers Magazine have good news for aspiring writers: the number of mystery novels published in the last two years has grown significantly, and more people read mysteries than any other type of novel. In addition, almost half of the mystery novels published last year were written by first-time novel writers. Since there is apparently an expanding market for mystery novels, all publishers will want to increase the number of mystery novels they publish. Therefore, new writers should write mystery novels to increase their chances for first-time publication with a larger, prestigious company."
    新作家往往无法使大型有威望的出版商相信他们的作品是有市场的,因此他们必须首先找一些较小的、知名度不那么高的出版社出版作品来建立自己的销售记录。但Mystery Writers Magazine的编辑为有抱负的作家带来了好消息:过去两年中神秘小说的出版数量显著增长,读这类小说的读者多于其他任何类型小说的人。而且,去年出版的神秘小说中差不多有一半是由第一次写小说的作家写的。由于神秘小说显然存在正在扩展的市场,所有出版商将会增加他们出版的神秘小说的数量。因此,新作家应该创作神秘小说来增加他们的处女作被大型有威望出版社出版的机会。

New writers,publishers 论断:新作家应该写神话小说来提高在大出版公司出处女作的机会。因为神话小说过去两年出版增长很快,看神话小说的人比其它书更多了。另外去年几乎一半的神话小说是由第一次出书的人写的。既然神话小说的市场有明显上升,所有的出版社会愿意增加该种小说的出版。
        被忽略的前提,大公司是否愿意出版神话小说。论者没有提供任何有关情况,很有可能大公司不原涉足进来。虽然神话小说的市场有明显上升,但也许仍属于边缘作品,他们更愿意出版有影响力的书。所以作者有关所有公司都会愿意增加该种小说的出版不一定成立。
        不一定成立的前提,新手写神话小说受到欢迎。虽然神话小说受欢迎,但是新手写了其中一半,很有可能这一半的书不受欢迎。
        不一定成立的前提,神话小说还受欢迎。those pursue for fashions always feel they are behind the fashions.
        不是所有新手作家都能写好神话小说,出版社也不一定对平庸mediocre的小说感兴趣。
结论:这个指导被表面现象蒙住了眼睛,建议新手作者迎合变幻的市场口味,而不重视作者的优势。这种指导书不看也罢。

135.The following is a memorandum from the business manager of WLSS television station.
     "Over the past year, our late-night news program has devoted increasingly more time to covering national news and less time to covering weather and local news. During the same time period, most of the complaints we received from viewers were concerned with the station's coverage of weather and local news. In addition, several local businesses that used to run advertisements during our late-night news program have just cancelled their advertising contracts with us. Therefore, in order to attract more viewers to our news programs and to avoid losing any further advertising revenues, we should expand the coverage of weather and local news on all our news programs."
      去年,我们的深夜新闻节目投入了更多的时间播放全国新闻,较少的时间播放天气预报和地方新闻。在同一时期,我们从观众那里获得的投诉大部分是关于电视台对于天气和地方新闻的播放的。而且,若干曾经在我们深夜新闻节目播放广告的地方企业刚刚取消了和我们的广告契约。因此,为吸引更多的电视观众观看我们的新闻节目并避免失去未来的广告收入,我们应该在我们所有的新闻节目中增加天气和地方新闻的播报。

late-night news program 论断:为吸引更多观众并避免广告收入继续流失,我们应该在所有新闻节目中加大对天气和当地新闻的报道。因为去年我们夜间新闻一直在增加全国新闻的时间,天气和当地新闻的时间少了。同一时间我们收到的大多观众投诉都是与与天气和当地新闻有关的。另外当地几家以前在夜间新闻节目有广告的公司刚撤消了与我们的广告合同。
        不一定成立的前提,这样的改变使得夜间新闻不受观众欢迎。首先论者没有提供数据表明夜间新闻的收视率从发生这样的改变后是否有下降的趋势,所以我们不能得出这样的结论。其次,论者说接到的大多数投诉是关于天气和当地新闻的,但抱怨的内容是什么论者没有提供,我们不能排除是说节目不真实,而不一定是说量不够。另外大多数是多少?如果最近的投诉一共只有两三个,那么一两个抱怨也同样不能说明这样的改变有问题。
        不一定成立的前提,这样的改变使得广告被撤。论者没有提供有关为什么广告被撤,因而不能在这两者之间建立起必然的因为联系。因为完全有其它原因存在,比公司财力发生问题,或是公司的其他策略有变化,这都有可能使广告被撤。
        结论,即使这样的改变使得夜间新闻不受观众欢迎,但也不能推广到全天节目。因为夜间新闻的收视率一般来说最小,其观众的收视兴趣也与其他时段不同。所以不代表所有节目都需要这么做。
结论:论者应该对变化后的节目做一次收视调查,并了解撤广告的原因,才能得出正确的答案。另外,各时段节目特点不一样,不能一刀切。

136.The following is part of a letter from the Brookfree School Board.
      "As part of our new educational reform program, we identified ten schools most in need of immediate improvement. An advisory committee is now recommending that we hire consultants to study further the schools' problems and to propose a plan of action. Only two years ago, however, another group of consultants determined that several schools in our district had problems because their principals were inexperienced. Thus, if we want to see immediate improvement in our schools and save the district the money it would spend on hiring more consultants, we should replace all the principals in the ten worst schools with ten of our most experienced principals in the district."
     作为我们新的教育改革计划的一部分,我们确定了10所最急需改善的学校。一个顾问委员会现在建议我们雇佣顾问来进一步研究学校的问题并设立行动方案。然而仅仅在两年以前,另一组顾问认为我们区的若干学校因为他们的校长没有经验而存在问题。因此,如果我们想要看到我们学校的立即改善并节省本地区用于雇佣更多顾问的花费的话,我们应该撤销这10所最差学校的校长并让本地区最有经验的校长来代替他们。

educational reform program 论断:要我们的学校马上进步并省下在顾问上花更多的钱,我们应该把最差的十所学校校长换成我们区最有经验的校长。因为我们发现有十所学校急需改进。一个顾问委员会现推荐我们雇用顾问深入研究学校的问题并提出行动方案。仅仅两年前,另一个顾问组说,区里几所学校因为校长没有经验而出现了问题。
        前提,校长有问题。论者没有提供几所学校目前的状况,也没有有关其校长的表现,我们无从判断。而论者说两年前有顾问说有学校因为校长而出现问题,但是两年前的情况,而现在不同。如果校长没问题,换校长就不能解决问题。
        前提,两年前的建议是正确的。关于两年前的状况论者也没有提供资料,顾问的建议不一定正确。
        结论:即使校长有问题,换校长不一定能解决。首先最有经验的校长离开原学校,会使原来的学校出现问题。其次,对新学校不了解,有经验也不一定能马上改变。另外有很多问题是长期存在的,要出现立即改观,不能保证。
结论:论者还是应该采取行动研究问题所在。

137.The following appeared in an editorial in the Mason City newspaper.
     "At present, Mason City residents seldom use the nearby Mason River for any kind of recreational activity, even though surveys of the region's residents consistently rank water sports (swimming, fishing, and boating) as a favorite form of recreation. Since there have been complaints about the quality of the water in the river, residents must be avoiding the river because they think that it is not clean enough. But that situation is about to change: the agency responsible for rivers in our region has announced plans to clean up Mason River. Therefore, recreational use of the river is likely to increase, so the Mason City council will need to increase its budget for improvements to the publicly owned lands along the Mason River."
      当前,Mason市很少利用附近的Mason河来进行娱乐活动,尽管对该地区居民的几次调查一直指出他们把水上运动(游泳、垂钓和划船)作为他们最喜欢的娱乐形式。由于曾经存在对于这条河水质的投诉,居民一定是因为他们认为河水不够干净才不在这里活动。但这种情况就会改变了:我们地区负责河流管理的部门公布了澄清Mason河的计划。因此,河流的娱乐用途很可能将会增加,因而Mason市的市委有必要增加用于改善Mason河沿岸公共土地的预算。

use the River for recreational activity 论断:政府需要为河边的公共场所的改善增加支出,因为河上娱乐可能会上升。目前居民很少用河进行娱乐,尽管对他们的调查一直说这是他们最喜欢的娱乐方式。由于一直有关于河水质量的抱怨,居民必须避开河水。不过这种情况会有改变,负责我们这个区的机构宣布了清理河的计划。
        前提,河水干净了河上娱乐会上升。尽管论者本地居民一直把水上运动列为他们最喜欢的运动,但由于长时间很少有人使用它,我们并不知道居民是否愿意在M河上进行这种娱乐。M河有可能不适合。
        前提,河水会干净。机构宣布了计划,不一定河水就能干净,污染程度,还有工作效率,都不一定能达到标准。
        结论,公共场所需要改进吗?不一定吧。
结论:这篇社论没有了解当地情况,也不知道前景如何,政府的决策还需考虑。

138.The following report appeared in a memo from the vice president of the Southside Transportation Authority.
    "We should abandon our current five-year plan to purchase additional buses to serve the campus of Southside University, because students there are unlikely to use them. Consider the results of the recent campaign sponsored by the Environmental Club at Southside University: in a program on the campus radio station, the club asked students to call in and pledge that they would commute to school by bus instead of by automobile at least one day per week. Only ten percent of the students called in and pledged. In view of the campaign's lack of success, we can assume that the bus service we currently offer will continue to be sufficient to serve the university."
       我们应该废止当前为服务Southside大学而购买更多巴士的五年计划,因为学生不太可能用到它们。请想一想Southside大学的环保俱乐部最近组织的一次活动的结果:在一次校园广播节目中,他们呼吁学生保证每周至少有一天坐公交车上学而不是坐私家车。只有10%的学生做了保证。鉴于这次活动没有获得成功,我们可以认为现在提供的公交服务将足以满足大学的需求。

Transportation,to serve the campus of Southside University 论断:我们应该放弃目前购买新公交车服务S校的计划。因为该校最近由环境俱乐部资助的活动号召学生签名每周至少坐一天公交车而不开车。但是只有10%的学生响应。从这个活动的不成功我们可以认为目前的公交车服务够用了。
        前提,目前的公交车够用了。论者没有提供有关这方面的资料,既然有计划买,说明目前有可能已经出现了拥挤。如果仍拥挤,坐车的人会少。
        前提,该校学生不喜欢公交车。该校的活动不能得出这样的结论,因为该活动号召学生坐公交车而不开车,针对的是拥有车的学生,没有车的学生当然不会去响应。如果说该一共只有10%的学生有车,那就说明该活动成功,并且坐车的人会上升。
        结论,该校还会有新生入学,人数会上升。
结论:论者没有仔细了解活动以及该校学生状况,

139.The following appeared in the editorial section of an educational
publication.
     "One study at Lee University found that first-semester grades of teenage students who had always attended public, tax-supported schools were slightly lower than the grades of students who had received some home schooling instruction by parents at home, although the grade differences disappeared in the second semester. These results suggest that home schooling is the best way to educate teenage children. Therefore, instead of spending more money on public education, the government should provide financial incentives so that home schooling is an option for more parents. After all, children schooled at home receive more attention, since they are taught by the best possible teacher: a parent who has a high stake in educating them well."
     一次在Lee大学进行的研究发现,那些一直参加由税收支持的公立学校的青少年学生的第一学期成绩比那些曾接受过家长在家中进行的教育的学生低一些,尽管这种分数的差异在第二学期就消失了。这些结果说明,家教是教育青少年的最好方式。因此,政府不应该在公共教育上花更多的钱,而应该提供经济上的激励使家教成为更多家长的选择。不论如何,在家接受教育的孩子受到更多关注,因为教他们的是最好的老师:最希望把他们教育好的家长。

Lee University, 论断:政府应该提供经费鼓励,因为家庭教育很更多父母来说并不是必须的。因为一项在L大学的研究发现,参加公共或税收支持的学校的学生第一年的成绩比其他在家由父母指导的孩子成绩稍差,尽管这种成绩上的差别在第二年会消失。这个结果显示家庭教育是指导孩子最好的方式。毕竟孩子在家里受教育会受到更多的重视,因为他们可能是由最好的老师指导:父母是最可能教育好他们的。
        被忽略的前提,是否所有的父母都能教育好孩子。没文化的怎么办,管不好孩子的怎么办,没时间的怎么办。财政不是原因。
        不一定成立的前提,父母是最可能教育好他们的。引用的论据不足信。首先研究本身是否科学,只在L校,还有孩子们自身的差别,是不是学习不好的才去上课,成绩只是稍差嘛。其次,即使研究结果没问题,但不上课的也不一定在家由父母教,他们可能自学,或是请家庭教师。
        结论,给公众学校更多钱,它可能会对孩子更有帮助。
结论:论者考虑问题不周全,忽略很多重要因为。另外引用的资料也不好好研究,找出学生成绩差异的原因。建议不值得采纳。

140.The following appeared in a report of the Committee on Faculty Promotions and Salaries at Elm City University.
      "During her seventeen years as a professor of botany, Professor Thomas has proved herself to be well worth her annual salary of $50,000. Her classes are among the largest at the university, demonstrating her popularity among students. Moreover, the money she has brought to the university in research grants has exceeded her salary in each of the last two years. Therefore, in consideration of Professor Thomas' demonstrated teaching and research abilities, we recommend that she receive a $10,000 raise and a promotion to Department Chairperson; without such a raise and promotion, we fear that Professor Thomas will leave Elm City University for another college."
     在Thomas教授作为植物学教授的17年里,她证明了自己确实是值50000元年薪的。她的班级是本校最大的班级之一,这说明她在学生中间的受欢迎度。而且,她给学校带来的研究捐助在过去两年中都超过了她的年薪。因此,鉴于Thomas教授已被证实的教学和研究能力,我们建议将她的年薪增加10000元,并提升为系主任;如果没有这些加薪和提升,我们担心Thomas教授将会离开Elm
City大学到别的学校就职。

professor of botany,well worth salary 论断:我们建议给T教授加1万元薪水,并升任系主任。不这样做担心T教授会去另的学校。而在该教授17年的植物学教授的职业经历中,她已证明她非常值目前的年薪。她的课是学校的最大的,说明她在学生中很受欢迎。另外她过去两年每年带给学校的研究经费已经超出了她的薪水。
        前提,T教授可以会去别的学校。论者没有给出资料为什么为有这样的担心。
        前提,T教授值得这样的建议。首先课堂大与受学生欢迎没有必然联系,有可能她的课是必须上的。其次研究经费只在过去两年的情况,不一定以后还会有。教学能力好,不一定能当主任。琐事多,当心少了一个好教授,多了一个无能的主任。这样的情况也许也不只她一人,理由不充足当心引起其他人不满。
        结论,如果有地儿要挖T教授,是否加薪和升职就能挡得住。学生学校的声誉,可以给她改善研究设施和环境。
结论:好的教授不一定是好的主任,好的教授也不会仅因为物质条件决定去留。

141.The following appeared in a newsletter distributed at a recent political rally.
     "Over the past year, the Consolidated Copper Company (CCC) has purchased over one million square miles of land in the tropical nation of West Fredonia. Mining copper on this land will inevitably result in pollution and environmental disaster, since West Fredonia is home to several endangered animal species. But such disaster can be prevented if consumers simply refuse to purchase products that are made with CCC's copper until the company abandons its mining plans."
    去年,Consolidated Copper Company(CCC)在热带国家West Fredonia购买了上百万平方英里的土地。在这些地方采矿将会不可避免地导致污染和环境灾害,因为West Fredonia是很多濒危物种的栖居地。但如果消费者简单地拒绝购买用CCC所生产的铜而制造的产品,直到CCC放弃它的采矿计划就可以避免这种灾害。

political rally,tropical nation 论断:消费者拒绝买CCC的铜制产品使它放弃采铜计划,就能阻止污染和环境灾害。因为过去几年CCC在WF买了上百万平方英里的土地,因为WF是很多濒危动物的家这些铜不可避免的会造成污染和环境灾害。
        被忽略前提:消费者无法辨别产品,因为CCC是采铜企业。
        不一定成立的前提:即使能分辨,CCC还可以把销售点转向其它地方。
        被忽略的前提:CCC放弃计划,还会有其他公司。
        不公平的结论:没有资料表明CCC在带来灾害,可以让他们采取措施。

142.The article entitled 'Eating Iron' in last month's issue of Eating for Health reported that a recent study found a correlation between high levels of iron in the diet and an increased risk of heart disease. Further, it is well established that there is a link between large amounts of red meat in the diet and heart disease, and red meat is high in iron. On the basis of the study and the well-established link between red meat and heart disease, we can conclude that the correlation between high iron levels and heart disease, then, is most probably a function of the correlation between red meat and heart disease.
     上一期的《健康饮食》杂志上刊登的题为《食铁》的文章报导说最近一项研究发现饮食中铁的含量过高与心脏病发病率增加有关联。而且,我们已经知道饮食中大量的牛羊肉和心脏病是有联系的,牛羊肉中铁的含量很高。基于以上研究和牛羊肉与心脏病之间的已知联系,我们可以得出结论,高铁含量与心脏病之间的关联最有可能是牛羊肉与心脏病之间关联的作用。

'Eating Iron' in last month's 论断:红肉和心脏病的关联很可能是含铁量高和心脏病的关联造成的。因为一个研究的成果以及已成立的红肉和心脏病之间的联系。这篇文章说最近的研究发现高含铁量的配餐与升高的心脏病危险有关联。而且饮食中大量的红肉成份和心脏病的联系是已确立的,红肉里的铁含量高。(unoxidated iron.calcium,heroin,rusty iron,compound)
        没有提供资料的前提:红肉与心脏病之间的联系,没数据,没研究,没调查。含量?食用方式?
        不一定成立的前提,即使红肉与心脏病的联系成立,没有资料证明红肉食用多的铁吸收就会多。有可能红肉的铁不容易被吸收的话。
        论据,研究结果不确切,高含铁量与心脏病之间的联系是怎样?铁对身体的影响有很多种,一定铁的摄入对身体是有好处的,非氧化铁,锈铁有区别。
结论:红肉的含的铁是怎样的,不一定会是造成心脏病的。所以不能得出结论红肉与心脏病的联系不是其它因素造成。

143.The following appeared as a letter to the editor of a national newspaper.
     "Your recent article on corporate downsizing* in the United States is misleading. The article gives the mistaken impression that many competent workers who lost jobs as a result of downsizing face serious economic hardship, often for years, before finding other suitable employment. But this impression is contradicted by a recent report on the United States economy, which found that since 1992 far more jobs have been created than have been eliminated. The report also demonstrates that many of those who lost their jobs have found new employment. Two-thirds of the newly created jobs have been in industries that tend to pay above-average wages, and the vast majority of these jobs are full-time." Downsizing is the process in which corporations deliberately reduce the number of their employees.
     你们最近关于美国集体裁员的文章是有误导性的。该文章给人们一种错误的印象,即很多在企业面临严重经济困难的时候裁员而导致失业的有能力的工人通常要用几年的时间找到另一份合适的工作。但这种感觉与最近一次关于美国经济的报告相矛盾,报告发现自1992年以来新增的就业机会数量远超过消失的岗位数量。该报告也指出很多失业人员已经找到了新工作。新增就业机会中有三分之二是那些提供高于平均水平薪酬的企业提供的,而且这些岗位绝大多数是全职工作。

downsizing 论断:这种印象,即有能力的工人因为裁员失业后面临严重经济困难,经常几年找不到合适的工作,是错误的。因为关于美国经济的报道说,从1992年以后新设的工作岗位比消失的多很多。这篇报道还显示很多失去工作的人已经找到了新工作。产业中三分之二的新设岗位比平均工资高,而且绝大多数是全职的。 推论完全建立在这篇报道,但报道没有提供任何数字与证据,无法判断它的真实与准确。先以它真实为前提,论者的推理中有问题。
        新设的岗位多,不一定被裁的工人上,人口在增多,有更多的人要找工作。
        被裁员的工人可能无法得到那三分之二的工高资和全职岗位。因为岗位工种限制、技术限制、年龄限制。
        很多失业人找到工作,不是全部,而且也许用了很多年。失业人数也在增加。

144.According to a poll of 200 charitable organizations, donations of money to nonprofit groups increased by nearly 25 percent last year, though not all charities gained equally. Religious groups gained the most (30 percent), followed by environmental groups (23 percent), whereas educational institutions experienced only a very small increase in donations (3 percent). This poll indicates that more people are willing and able to give money to charities but that funding for education is not a priority for most people. These differences in donation rates must result from the perception that educational institutions are less in need of donations than are other kinds of institutions.
     根据一项针对200个慈善组织的调查,去年对于非营利团体的捐款上升了将近25%,而并不是所有组织都获得了同样的增幅。宗教团体增幅最大(30%),其次为环境组织(23%),而教育机构所获捐款仅有少量增长(3%)。这一调查说明有更多的人愿意而且有能力为慈善组织捐款,但资助教育并不是人们的首选。这种捐款数额上的差异一定是由于人们认为教育机构没有其他组织更需要资助的观念而导致的。

charitable organizations,donations 论断:捐款上升率的差别是认为教育机构没有其它机构需要捐款的印象的结果。根据对200个慈善机构的调查,对非营利机构的捐款去年上升了将近25%,尽管不是所有的机构获得的数额都相等。宗教团体获得最多有30%,然后是环境有23%,而教育机构仅有非常小的上升为3%。这份调查还显示,越来越多的人愿意且能够为慈善捐钱,但是教育不是大多数人的首选。(人们捐钱行业慈善机构收捐款---给具体的行业团体,人们捐钱给教育慈善机收收捐款---给学校)
        前提,教育机构获得的捐款不都从慈善机构来。学校遍地是,教育慈善就那么几家,有很多人直接捐钱到学校。
        论据不足信。教育机构获得的捐款不一定比其他少。没提基数,上升率,不说明问题。总数上升还可能很多。
        证据不说明问题,尽管不是首选,也不一定认为它不需要钱。可能排第二。
结论:不同的人对教育观念不一样,有人可能觉得它需要钱,但是它更关心别的。

145.A new study collected data that shows that people who snore are more likely to gain weight than are people who do not snore. It is well known that many people who snore also stop breathing frequently during the night for a few seconds, a condition called sleep apnea. The interruption of breathing wakes the person-often so briefly that the waking goes unnoticed-and can leave the person too tired during the day to exercise. Anyone who snores, therefore, should try to eat less than the average person and to exercise more.
     一项新的调查所收集的数据显示打呼噜的人比不打呼噜的人更容易长胖。我们知道很多打呼噜的人夜间也经常有数秒钟的呼吸停顿,这种现象被称为睡眠呼吸中断。这种呼吸的停顿导致本人醒来--通常十分短暂以至于不被察觉--并且会使本人在白天很累而无力锻炼。因此,所有打呼噜的人都应该比正常人少吃多锻炼。

snore,gain weight 论断:打鼾的人应该试着比一般人少吃点并多运动。因为一项新的研究收集数据显示打鼾的人比不打的人更容易长胖。大家都知道很多打鼾的人在夜里也会经常有几秒钟的停止呼吸,这叫睡眠窒息状态。这种呼吸的间断会使人醒过来,经常非常短而不被注意到,这会使人在白天感觉特别累而不愿运动。
        前提:打鼾的人不运动。没有证据,没有资料。有可能还运动。
        前提:打鼾的人白天觉得累。也是没有证据。不被察觉的清醒是否会影响睡眠质量。
        证据有问题:一是描述不科学,不知道是否有代表性。二是,别的原因使得长胖,而打鼾是长胖的原因。
结论:即使打鼾会使累,也不应该少吃多运动。都累了,还苦着,身体会坏的。

146.The following appeared in the editorial section of a local newspaper.
   "The librarians in our town's school system have reported that the number of trips that our students make to their school library on a voluntary basis has decreased significantly in recent years. For example, the average seventh-grade student visited the school library five times last year, but four of those visits were part of required classroom activities. This shows that our students are reading less than in the past. To address this problem, our town needs to improve the atmosphere of the libraries so that they will be comfortable places in which to work. If students view the libraries as uncomfortable, then they are unlikely to want to spend much time there."
    我市学校系统的图书馆管理员报告说近年来我们的学生主动去学校图书馆的次数显著下降。举例来说,去年7年级学生平均去图书馆5次,但其中有4次是教学活动所要求的。这表明我们的学生读书比以前少了。为解决这一问题,我市需要改善图书馆的环境使他们适于工作。如果学生认为图书馆不舒服,他们将不太可能想在这里花很多时间。

librarians,students trip 论断:镇上需要改善图书馆的环境,这样使在图书馆非常舒服。因为如果学生觉得图书馆不舒服,他们会不愿意在这儿花很多时间。镇上学校系统的图书馆管理员报告说最近几年学生自愿到学校图书馆的明显少了。比如说,一般七年级的学生去年到学校图书馆五次,但四次是集体活动要求。这表示我们的学生阅读比以前少了。
        前提,图书馆舒服了,学生也不一定会多来。因为没有证据表明学生不来是因为不舒服。
        推论,学生自愿到图书馆少了不一定阅读就少了。学生可能自己买书看的多了,或是去多公共图书馆。
        论据:管理员的报告不一定能说明问题,结论是如何得出来的,是印象,还是登记纪录。而有关七年级学生的例子,也许他们去年的其他活动很多。
结论:学校图书馆情况是这样,不是所有图书馆都需要。

147.The following appeared in an editorial in a business magazine.
    "Although the sales of Whirlwind video games have declined over the past two years, a recent survey of video-game players suggests that this sales trend is about to be reversed. The survey asked video-game players what features they thought were most important in a video game. According to the survey, players prefer games that provide lifelike graphics, which require the most up-to-date computers. Whirlwind has just introduced several such games with an extensive advertising campaign directed at people 10 to 25 years old, the age-group most likely to play video games. It follows, then, that the sales of Whirlwind video games are likely to increase dramatically in the next few months."
     尽管Whirlwind游戏公司的游戏销量在过去两年中下降了,最近一次对于电子游戏玩家的调查显示这种销售趋势可能会逆转。该调查询问游戏玩家对于一个游戏来说他们所认为的最重要特征是什么。根据调查结果,玩家倾向于那些需要最先进的电脑的,具有活灵活现图象的游戏。Whirlwind刚刚面向10-25岁的人群开展了大力的广告活动来推广若干此类游戏,10-25岁是最喜欢玩游戏的年龄层。这说明Whirlwind公司的游戏销量将会在未来几个月中猛增。

Whirlwind video games 论断:WW今后几个月的销售可能会急聚上升。因为一份对电视游戏玩家的调查显示,它的销售趋势会转变。这份调查问玩家们认为游戏的什么特点最重要,他们偏向有生动动画的游戏,这种游戏需要最新的电脑技术。WW刚刚用大规模广告向10到25岁的人群推出了几种这样的游戏,这个年纪的人群最可能玩电游。
        前提:关于10到25岁最可能玩电游,没有数字,没有根据,是瞎猜的。
        WV会卖得好,虽然玩家们认为这是最重要的特点,但并不表示生动了他们就一定买。因为游戏还有其他特点,比如类型、是否刺激、主题等。如果光有技术,思想不足,也不一定卖得好。而且他没有说用的什么技术,我们无法认同是否存在判断失误。
        论据:广告针对10-25岁人群,但调查不是,这两者可能存在偏差。
结论:尽管WV吸引人,可能经济萧条,或整个市场不好。

使用道具 举报

Rank: 2

声望
9
寄托币
139
注册时间
2009-4-2
精华
0
帖子
28
18
发表于 2009-4-26 22:12:15 |只看该作者
148.The following appeared in the editorial section of Monroetown's local newspaper.
     "Mayor Brown was recently re-elected by a clear majority of 52 percent of Monroetown's voters. Her re-election, however, does not show that most people in our town favored Mayor Brown's proposal for tax reduction over that of her opponent, Mr. Greene, who proposed raising taxes to improve education. It has been shown that voters nationwide tend to re-elect people already in office, regardless of candidates' proposals. In fact, a local survey after the election showed most people in Monroetown disagreed with Mayor Brown's proposal. Clearly most people in Monroetown favor improving education and therefore approve of Mr. Greene's proposal despite the fact that they did not vote for him."
      Brown市长最近获得了Monroetown全体选民52%的票而再次当选市长。然而,她的再次当选并不表明我们城市的多数市民更喜欢Brown提出了减税提案,而不是其竞争对手Greene提出的增加税收以促进教育的提案。已有证据表明全国的选民都倾向于再次选举那些已经在位的官员,不论候选人的竞选提案是什么。实际上,这次选举后的一次地方调查显示,Monroetown的多数市民不赞同Brown市长的提案。显然Monroetown的多数市民愿意改善教育,从而支持Greene的提案,尽管他们并没有投他的票。

Mayor,re-elected by a clear majority,education,tax 论断:B被52%的人重选当市长并不证明大多数的人支持她降低税收的建议。因为大多数M镇的人支持G改善教育提高税收的建议,尽管他们没有选他。事实上在选举之后的调查中,大多数M镇的人不同意B市场的建议。而且全国都有趋势重选前任官员。
        前提:M镇的人不支持降低税收。论者搞混了三件事,一是是否改善教育,二是是否降低税收,三是是否需要通过提高税收来改善教育(G的建议)。所以如果M镇的支持改善教育,并不意味着他们支持用提高税收的方式来进行,因而也不代表他们不支持B降低税收的建议。
        论据:尽管调查结果显示人们不同意B的建议,但人们还是有可能同意降低税收的作法。B不只这一个建议吧。另外调查是否有代表性,没有被描述。而且不赞同B不一定就赞同G
        论据,全国性的趋势不一定代表当地的情况。

149.The following is a memorandum from the director of personnel to the president of Get-Away Airlines.
     "Since our mechanics are responsible for inspecting and maintaining our aircraft, Get-Away Airlines should pay to send them to the Quality-Care Seminar, a two-week seminar on proper maintenance procedures. I recommend this seminar because it is likely to be a wise investment, given that the automobile racing industry recently reported that the performance of its maintenance crews improved markedly after their crews had attended the seminar. These maintenance crews perform many of the same functions as do our mechanics, including refueling and repairing engines. The money we spend on sending our staff to the seminar will inevitably lead to improved maintenance and thus to greater customer satisfaction along with greater profits for our airline."
     由于我们的机修师负责检测和维护我们的飞机,Get-Away航空公司应该花钱把他们送去参加一个为期两周的,关于合理维护流程的质量保障研讨会。我推荐这个研讨会是因为这应该是一个明智的投资,因为赛车企业最近报告说他们的维修人员的水平在他们参加完研讨会之后显著提高了。这些维修人员和我们机修师所作的工作有很多是相同的,包括加注燃油和维修引擎。我们用于把员工送去参加研讨会的花费必然会为本公司带来维护水平的提高,并带来更高的客户满意度和更大的利润。

Get-Away Airlines,maintaining our aircraft 论断:花钱送员工参加这个研习班一定会使维修技术提高,从而顾客满意度了利润都会提高。因为赛车产业最近报告说他们员工的维修技术在参加这个研习班后有很显著的提高。这些维修人员有很多和我们机师一样的作用,包括加油和维修发动机。
        员工参加研习班技术一定会提高。没有直接证据。提供的没有说服力。赛车行业和飞机会有很大差别,尽管员工的作用一样,但工作的内容不一样。对他们有用,不一定对飞机有用。
        就算提高也不一定顾客就会满意,(puncuality, the on-board srvice, the ticket price, the luggage handling procedure and even the discount,)(significantly increse it customers or passengers and at the same time cut down its cost,)

150.The following is a letter to the editor of an environmental magazine.
     "The decline in the numbers of amphibians worldwide clearly indicates the global pollution of water and air. Two studies of amphibians in Yosemite National Park in California confirm my conclusion. In 1915 there were seven species of amphibians in the park, and there were abundant numbers of each species. However, in 1992 there were only four species of amphibians observed in the park, and the numbers of each species were drastically reduced. The decline in Yosemite has been blamed on the introduction of trout into the park's waters, which began in 1920 (trout are known to eat amphibian eggs). But the introduction of trout cannot be the real reason for the Yosemite decline because it does not explain the worldwide decline."
     全球两栖动物数量的下降清楚地说明全球空气和水质的污染。在加州Yosemite国家公园对于两栖动物所进行的两次研究证实了我的结论。1915年公园中有7中两栖动物,每种的数量都很丰富。然而到了1992年在公园中只观察到4种两栖动物,并且每种动物的数量都显著下降。Yosemite公园两栖动物数量的下降曾被归因于始于1920年的在公园水域引入鲑鱼的行为(我们知道鲑鱼捕食两栖动物的卵)。但鲑鱼的引入不会是Yosemite两栖动物数量下降的真正原因应为它并不能解释全球范围的数量下降。

decline in the numbers of amphibians worldwide 论断:YNP地区两栖动物的下降不是因为鳟鱼的引入。全球范围两栖动物的数目下降,显示有全球水域和空气的污染。两次关于YNP两栖动物的研究支持我们的结论。在1915年这里有7种两栖动物,每种数目都很多。但是在1992年,这里只看得到四种,每种的数目也都显著下降。这种下降一直被归结为公园水域引入鳟鱼,而这种引入不能解释全球性的下降。 论断中有两个结论,一全球下降,显示全球有污染。二是YNP地区的原因不是因为鳟鱼。但是两个结论论者都没有提供相关的证据,我们既不知是否全球数目有下降,也不知道YNP地区是否还有其他因素。
        两次研究仅能证明YNP地区的数目在下降,但没有得出结论。论者也没有提供任何资料表明该地区存在空气或水域污染。因而不能像论者所以为的那样支持他关于全球数目下降显示全球有污染的结论。(不是污染引起该地区下降)
        是否有全球性的数目下降,又否是存在全球性的污染,论者都没有提供任何有关的证据。而且论者也未能在这两者之间建立因果关系,我们无法得出结论,即数目下降的原因是否是因为污染,因为也无法用以证实YNP地区的下降有可能是因为污染,也就不能排除已建立的可能,即鳟鱼引起了下降。(不是污染引起了全球的下降)
        YNP地区情况完全可以不和全球相同。
结论:论者的推论中有明显的逻辑错误,他将两个仍需要证实的结论,互为彼此推理过程的基础。思维极其混乱。

151.The following is a letter to the editor of the Atticus City newspaper.
     "Former Mayor Durant owes an apology to the city of Atticus. Both the damage to the River Bridge, which connects Atticus to Hartley, and the traffic problems we have long experienced on the bridge were actually caused 20 years ago by Durant. After all, he is the one who approved the construction of the bridge. If he had approved a wider and better-designed bridge, on which approximately the same amount of public money would have been spent, none of the damage or problems would have occurred. Instead, the River Bridge has deteriorated far more rapidly over the past 20 years than has the much longer Derby Bridge up the river. Even though the winters have been severe in the past several years, this is no excuse for the negligence and wastefulness of Durant."
     前任市长Durant应该向Atticus市道歉。连接着Atticus和Hartley两座城市的River Bridge的损害和长期困扰着我们的桥上的交通问题实际上都是20年前因Durant而导致的。无论如何,是他批准了桥梁的建造。如果他能够批准一个差不多花同样的钱的设计更好的方案的话,桥梁的损坏和交通问题都不会出现。但现在,River Bridge在过去20年间的损坏程度比上游另外一座更长的Derby Bridge更加严重。尽管近几年的冬天气候确实比较恶劣,但这也不是为Durant的过失和浪费开脱的借口。

Former Mayor owes an apology to the city 论断:前市长应该为R桥的破坏和长期以来桥上的交通堵塞问题向市民道歉,因为如果他批准用差不多的公共钱建一个更宽设计更好的桥,就不会有破坏和问题发生。现在这个20年前建的桥比另一个时间更长的D桥损毁的速度快得多。尽管过去几年冬天比较冷,但这也不是疏忽和浪费的借口。
        前提,桥是否窄了。桥在20年前建设的,后来车多了,人多了,还有这条道变重要了,但以前可能没这么多。
        前提,桥是否不好。这是从与D桥对比中来,但是位置不同不说明问题,因为水流,使用情况都会不同。另外,天气的寒冷是可能造成损坏的原因
        结论,即使桥不好,可能是因为当时的技术问题和资金问题。

152.The following is a letter to the head of the tourism bureau on the island of Tria.
     "Erosion of beach sand along the shores of Tria Island is a serious threat to our island and our tourist industry. In order to stop the erosion, we should charge people for using the beaches. Although this solution may annoy a few tourists in the short term, it will reduce the number of people using the beaches and will raise money for replenishing the sand. Replenishing the sand, as was done to protect buildings on the nearby island of Batia, will help protect buildings along our shores, thereby reducing these buildings' risk of additional damage from severe storms. And since the areas along the shore will be more attractive as a result, the beaches will be preserved and the area's tourist industry will improve over the long term."
     Tria岛海岸沙滩的侵蚀对于我们岛和我们的旅游业是个严重的威胁。为阻止侵蚀,我们应该对使用海滩的人收费。尽管这一解决方案会在短期内触怒少量游客,它将会减少使用海滩的人数并增加补充沙子的资金。象临近的Batia岛一样补充沙子将会有助于对我们沿岸建筑的保护,从而减少这些房屋在大风暴中受损的危险。并且由于这会导致沿岸地区更具吸引力,海滩将会受到保护,本地区的旅游业将会在长远得到发展。

Erosion of beach sand,tourist industry 论断:我们应该向使用沙滩的人收费,这样可以降低沙滩的使用而且可为填沙攒钱。填沙可以帮助保护沙滩边的房子,降低受严重风暴的特别破坏的危险(隔壁B岛用填沙保护建筑)。(这些都可以防止侵蚀)目前海滩沙的侵蚀是对岛和旅游业的严重危胁,而如果防止侵蚀岸边地区会因此而更吸引人,海滩也会因此保留,而且长远来看旅游业也会提高。
        降低人们使用沙滩不一定能保护沙滩,。 •建筑被保护了不一定就使沙滩更吸引人,因而也不一定能促进旅游业。气候问题,服务问题。
        .Charging people for using the beachers does not prevent sand erosion. There many factors exist. the tourists themself can be one . Moreover, no service in beachers will result in decrease of tourists.
        The preserved sand can do little help to the buildings in severe storms.

153.The following is from an editorial in the Midvale Observer, a local newspaper.
    "Ever since the 1950's, when television sets began to appear in the average home, the rate of crimes committed by teenagers in the country of Alta has steadily increased. This increase in teenage crime parallels the increase in violence shown on television. According to several national studies, even very young children who watch a great number of television shows featuring violent scenes display more violent behavior within their home environment than do children who do not watch violent shows. Furthermore, in a survey conducted by the Observer, over 90 percent of the respondents were parents who indicated that prime-time television-programs that are shown between 7 p.m. and 9 p.m.-should show less violence. Therefore, in order to lower the rate of teenage crime in Alta, television viewers should demand that television programmers reduce the amount of violence shown during prime time."
     20世纪50年代电视开始出现在平常家庭以来,Alta的青少年犯罪发生率也在稳步增加。这种青少年犯罪的增加和电视中暴力节目的增加相对应。根据几次全国调查,那些看了大量含有暴力镜头的节目的很小的儿童比那些不看暴力节目的儿童在家庭环境中表现出更多的暴力行为。而且,在一次由Observer所举行的调查中,超过90%的回应者是那些指出黄金时段的电视节目(晚上7点-9点的节目)应该少播放暴力的家长。因此,为降低Alta青少年犯罪率,电视观众应该要求电视节目制作者减少黄金时段暴力镜头的播出数量。

teenage crime,television 论断:为降低A的少年犯罪率,电视观众应该要求电视节目减少黄金时段里的暴力内容。因为自从电视机进入普通家庭以来,少年犯罪率就一直在上升,电视里的暴力内容也在上升。根据几个国家的研究,甚至很小的孩子看了大量电视上的暴力特定都会在家里表现出比其它孩子更暴力的行为。而且,观察者做的一份调查,90%的被访者是表示黄金时段的电视节目应该少一些暴力内容的父母。
        前提,少年犯罪率的上升是因为电视节目的暴力内容上升,是否有犯罪的少年经常看电视里的暴力。
        前提,暴力并不等于犯罪。
        论据,调查没有科学的描述。

154.The following appeared in the editorial section of a health and fitness magazine.
      "In a study of the effects of exercise on longevity, medical researchers tracked 500 middle-aged men over a 20-year period. The subjects represented a variety of occupations in several different parts of the country and responded to an annual survey in which they were asked: How often and how strenuously do you exercise? Of those who responded, the men who reported that they engaged in vigorous outdoor exercise nearly every day lived longer than the men who reported that they exercised mildly only once or twice a week. Given the clear link that this study establishes between longevity and exercise, doctors should not recommend moderate exercise to their patients but should instead encourage vigorous outdoor exercise on a daily basis."
       在一次关于锻炼对寿命的影响的调查中,医务人员在20年的期间跟踪了500名中年男子。调查对象覆盖了全国若干不同地区的多种职业,他们每年回答一次问卷,在问卷中他们被问道:你锻炼的频率和强度如何?在那些回答了问卷的人中,那些报告说自己几乎每天都参加激烈的户外运动的人比那些报告说自己每周仅仅温和地锻炼一两次的人活的长。鉴于这一调查所建立的长寿与锻炼之间的明显联系,医生不应该建议患者进行温和的运动而应该鼓励他们每天都参加激烈的户外运动。

exercise on longevity 论断:医生不应该建议病人进行适量运动,而应该鼓励每天强化的室外运动。因为一份对500个不同行业不同地区的中年男子长达20年的研究显示,说自己几乎每天都有巨烈室外运动的人比说自己每周一次或两次中等运动的人活得长。
        前提,频繁的巨烈室外运动比中等运动更使人长寿。没有直接证据。而且同时把巨烈和频率放在一起,这是两个概念,而且不一定同时,也许分开会有效。
        论据研究有问题。首先,不同地区,不同职业,人的状况不一样。其次,身体好的人,可能会比其它人更喜欢运动,尤其是一些强度大的运动。第三,只有中年男人,女人怎么办?
结论:医生的病人有病呢!!

155.The following appeared in a letter to the editor of a local newspaper.
     "Too much emphasis is placed on the development of reading skills in elementary school. Many students who are discouraged by the lonely activity of reading turn away from schoolwork merely because they are poor readers. But books recorded on audiocassette tape provide an important alternative for students at this crucial stage in their education, one the school board should not reject merely because of the expense involved. After all, many studies attest to the value of allowing students to hear books read aloud; there is even evidence that students whose parents read to them are even more likely to become able readers. Thus, hearing books on tape can only make students more eager to read and to learn. Therefore, the school board should encourage schools to buy books on tape and to use them in elementary education."
    小学对于阅读能力的培养的强调过分了。很多对于单调的阅读活动不感兴趣的学生仅仅因为他们阅读能力欠佳而放弃学习。但录制在磁带上的教材为学生在教育的关键阶段提供了重要的补充,教委不应该仅仅因为其所需的花费而拒绝它。无论如何,很多研究证实了让学生听大声朗读的教材的作用;甚至还有证据表明那些由家长为他们朗读的学生阅读能力更容易提高。因此,听录制教材只会让学生更乐意阅读和学习。所以,教委应该鼓励学校购买录制的教材并把它们用于基础教育。

Too much emphasis,reading skills in elementary school 论断:学校董事会应该鼓励学校为基础教育买音像书。因为很多被孤单的阅读打击的学生不愿上课仅仅是因为他们阅读不好。但是录在磁带上的书给这个关键年龄段的孩子提供了很重要的后选。学校不能仅因为它贵而拒绝。毕竟很多研究都证实让学生听书的价值。甚至有证据表明父母给他们念书的孩子更愿意成为好的阅读者。所以听磁带上的书不仅使学生更愿阅读且会更愿学。
        前提,音像书会使学生更愿阅读。
        论据有问题,没有提供直接关于磁带的研究,而父母念书的和这是不一样的。
        结论:是否应该鼓励。首先,有多少学生不愿上课是因为阅读?不多就没必要了。其次,花费到底是多少?学校没钱也不应鼓励。

156.The following is taken from an advertisement placed in a weekly business magazine by the Dickens Academy.
     "We distributed a survey to senior management at International Mega-Publishing, Inc. The result of the survey clearly indicates that many employees were well prepared in business knowledge and computer skills, but lacked interpersonal skills to interact gracefully with customers. International Mega-Publishing decided to improve customer satisfaction by sending their newly hired employees to our one-day seminars. Since taking advantage of our program, International Mega-Publishing has seen a sharp increase in sales, an indication that the number of their disgruntled customers has declined significantly. Your company should hire Dickens and let us turn every employee into an ambassador for your company."
     我们对International Mega-Publishing, Inc的高层管理人员进行了一次调查。调查结果清楚地显示很多员工在商业知识和计算机技能方面准备充足,但是缺乏与客户得体沟通的人际交往能力。International
Mega-Publishing决定通过让他们的新员工参加我们为期一天的研讨会来提高客户满意度。在参加了我们的课程之后,International Mega-Publishing的销量急剧上升,这表明不满意的顾客数量显著下降。你们公司应该聘用Dickens来让我们把每名员工变成你们公司的大使.

International Mega-Publishing,interpersonal skills 论断:你们应该雇用我们来使你们的员工成为公司的外交大使。因为自从IM公司送新员工来参加我们一天的培训后,他们的销售争取上升,关于顾客抱怨的数字也明显下降。
        前提,D公司的一天培训能提高员工的交际能力,广告中没有相关的资料证明IM公司的新员工因为这项研究班而有提升。
        前提,IM公司的业绩上升可能不是因为研究班。首先不是因为新员工,其次不是因为人际能力的上升。
        结论,其他工司是否也需要员工有很好的人际交往能力,或者外交大使。

157.The following is a letter to the editor of a local newspaper.
      "As a local merchant, I wish to support the development of a ski resort in the state park north of our township. Along with many other merchants who favor the proposal by Ski the Slopes, Incorporated, I would, of course, experience a growth in my business. But I also know how much more prosperous, not to mention lively and interesting, our community would be if tourism increased. Since the main opposition comes from a few environmentalists* who do not even live in this community, I see no reason to give in to their views. The First National Bank has finally researched the project and agreed to fund it. As a result, I see no reason to delay development of the resort." Environmentalists are people who advocate the preservation of the natural environment.
     作为一个本地商人,我支持在我们城市北部的州立公园建造一个滑雪场。和其他很多支持Ski the Slopes, Incorporated的商人一样,我的生意显然也将会增长。但是我还知道如果旅游业增长了,我们社区将会多么繁荣,还不用说它带来的生机和乐趣。由于主要的反对意见来自一些甚至并不住在本社区的环境主义者*,我认为没有理由向他们的观点屈服。First National Bank最终研究了这个方案并同意资助它。因此,我认为没有理由推迟滑雪场的建造。 *环境主义者是倡导保护自然环境的人。

ski resort 论断:没有理由推迟建设度假村。旅游业上升的话我们的社区会非常繁荣,而且它很有趣。第一国家银行也最后调查了这个计划并同意给资金。由于主要的反对意见还自几个甚至不住在这个社区的环境保护者,我觉得没有理由采纳他们的意见。
        前提,没有证据表明滑雪度假村一定会使旅游业上升。
        度假村可能会带来的负面作用。环保主义者的反对意见,说明建度假村对环境有影响。
        是否有能力建设,有银行提供资金不能保证就能建了。

158.The Trash-Site Safety Council has recently conducted a statewide study of possible harmful effects of garbage sites on the health of people living near the sites. A total of five sites and 300 people were examined. The study revealed, on average, only a small statistical correlation between the proximity of homes to garbage sites and the incidence of unexplained rashes among people living in these homes. Furthermore, although it is true that people living near the largest trash sites had a slightly higher incidence of the rashes, there was otherwise no correlation between the size of the garbage sites and people's health. Therefore, the council is pleased to announce that the current system of garbage sites does not pose a significant health hazard. We see no need to restrict the size of such sites in our state or to place any restrictions on the number of homes built near the sites.
    垃圾场安全委员会最近对于垃圾场对住在附近的居民健康的可能危害进行了一次调查。调查一共检测了5个垃圾场和300位居民。研究发现平均而言,在垃圾场附近居住和这些居民中一种未查明原因的皮炎之间只有很小的相关性。而且,尽管住在最大的垃圾场附近的居民患皮炎的比例确实要高一些,但是垃圾场的规模和人们的健康之间是没有关联的。因此,委员会很高兴宣布现有的垃圾场系统不会对健康产生严重的健康危害。我们认为没有必要限制本州这类垃圾场的规模,或对于在垃圾场周围建造的住宅的数量加以任何限制。

harmful effects on the health of people living near garbage sites 论断:没有必要对垃圾场的面积作限制,也没有必要对垃圾场旁的房屋建筑数目做限制。因为目前的垃圾系统没有造成大的健康危害。最近对州上五个垃圾场和300个住在附近的人的研究显示,住户住在垃圾场边和解释不清的疹一般只有很小的数据关系。而且虽然住在最大的垃圾场旁的人会有较高的轻度疹发生次数,但垃圾场大小和人们的健康也没有关系。 所有基础就是这个调查,但调查有漏洞。
        有无皮疹不能说明是否对健康有害,论者也没有提供为什么用皮疹作唯一参照标准。
        既然最大的垃圾场旁边有较高的发生次数,就说明有可能它对健康的影响比其他的更大。The argue treats the evidence for the occurence of a condition under which a circumstance will occur as the conclusive evidence that a circumstance will not occur.
        目前没有危害,如果不限制危害有可能发生。

159.The nation of Claria covers a vast physical area. But despite wide geographic differences, many citizens are experiencing rising costs of electricity. A recent study of household electric costs in Claria found that families who cooled their houses with fans alone spent more on electricity than did families using air conditioners alone for cooling. However, those households that reported using both fans and air conditioners spent less on electricity than those households that used either fans or air conditioners alone. Thus, the citizens of Claria should follow the study's recommendation and use both air conditioners and fans in order to save money on electricity.
     Claria这个国家覆盖了广泛的地理区域。但尽管存在很大地理差异,很多居民都遇到了电费上涨的经历。最近一项对于Claria家庭用电花费的研究发现,那些单独使用风扇来降温的家庭电费花费比那些单独使用空调降温的家庭要多。然而,那些报告说同时使用风扇和空调的家庭电费花费比单独使用风扇或空调的家庭少。因此,Claria的居民应该遵从这个研究的建议,同时使用空调和风扇来节省电费花费。

rising costs of electricity 论断:C的居民应该同时使用空调和电扇来省电费。因为最近对该国家庭电费的调查发现,仅用电扇降温的家庭比仅用空调降温的家庭电费更高,但是两个都用的家庭花得比它们都少。调查存在的问题:
        前提,家庭用电不只在降温上,还有其他电器。the argue explain the event contributed by many facots with only one reason which is not sufficient to garantee the result.
        前提,两个都用能降低家庭用于降温的电费。各地温度不同,用于此的电费自然不同,该国有vast physical area,这点被忽略了。The argue neglects a very important reason that contribute to the result. •电价问题。

160.As people grow older, an enzyme known as PEP increasingly breaks down the neuropeptide chemicals involved in learning and memory. But now, researchers have found compounds that prevent PEP from breaking neuropeptides apart. In tests, these compounds almost completely restored lost memory in rats. The use of these compounds should be extended to students who have poor memory and difficulty in concentrating-and therefore serious problems in school performance. Science finally has a solution for problems neither parents nor teachers could solve.
     随着人们的衰老,一种已知叫做PEP的酶不断地分解与学习和记忆有关的神经化学物质。但现在研究人员已经发现了一些阻止PEP分解神经化学键的化合物。在试验中,这些化合物几乎完全恢复了老鼠失去的记忆。这些化合物的使用可以扩展到那些记忆力较差和因注意力不能集中而在学习中存在严重问题的学生。科学最终能够解决家长和教师都无法解决的问题。

enzyme known as PEP 论断:这个混和物的应用应该扩大到记忆不好,不容易集中精力的学生身上。因为最近研究者们发现这个混和物可以防止PEP分解某种神经。在试验中,这些混和物几乎使免子恢复了所有记忆。(PEP是一种酶,随着人变老它会逐渐分解某种和学习、记忆有关的神经)
        兔子跟人不一样,实验最终还需要在人身上进行。
        PEP分解神经是否在青少年时期就会发生。
        学生记忆不好和不集中精力的主要原因不是生理原因,懒惰、贪玩是老师家长解决不了的。
        会不会有副作用。

161.In a study of reading habits of Leeville citizens conducted by the University of Leeville, most respondents said they preferred literary classics as reading material. However, a follow-up study conducted by the same researchers found that the type of book most frequently checked out of each of the public libraries in Leeville was the mystery novel. Therefore, it can be concluded that the respondents in the first study had misrepresented their reading habits.
     在一次由Leeville大学所举行的关于Leeville居民阅读习惯的调查中,多数被访者说他们倾向于阅读古典文学。然而,由相同的研究人员随后进行的调查发现Leeville所有公共图书馆中最经常被借阅的书是神秘小说。因此,我们可以得出结论,第一次调查的被访者错误地表达了他们的阅读习惯

reading habits of Leeville citizens 论断:居民不是最爱看古典文学。一项由L校进行的该地区居民阅读习惯研究中,大多数被访者说他们最爱看古典文学。但是他们随后进行的研究又发现该地区每个公共图书馆里最常被借的是神话小说。
        古典文学里有很多都是神话小说。奥德赛,希望神话都是古典小说。 First at all , the mystery novels can also be literary classics. The arguer undercounted the scope of "literary classics" . in fact, all book write by writers past and made great successes in readers can be called "literary classics", there are many mystery novels such as the "Odessey" and "Greek Mysterious story" have long been considered as literary classics. Moreover, each reader has his/her own concept about "literary classics", therefore, the arguer's conclusion about they had misrepresented theri reading habits is unfounded.
        公共图书馆不是唯一的书源。可以买,可以去私人图书馆。有可能是因为公共图书馆里古典小说藏量不丰富,或是神话小说过多而造成的。人们只是顺便借来看看。
        第二次研究时间多长,有可能只是暂时现象。比如流行。

162.A recent study shows that people living on the continent of North America suffer 9 times more chronic fatigue and 31 times more chronic depression than do people living on the continent of Asia. Interestingly, Asians, on average, eat 20 grams of soy per day, whereas North Americans eat virtually none. It turns out that soy contains phytochemicals called isoflavones, which have been found to possess disease-preventing properties. Thus, North Americans should consider eating soy on a regular basis as a way of preventing fatigue and depression.
     最近一次研究显示居住在北美大陆的人患慢性疲劳和慢性抑郁症的数量分别为居住在亚洲大陆居民的9倍和31倍。有趣的是,亚洲人平均每天食用20克大豆,而北美人几乎不吃。人们发现大豆含有一种具有抗病功效的植物化合物单黄酮。因此,北美人应该考虑经常食用大豆作为预防疲劳和抑郁的方法。

North America fatigue,depression,Asia,soy 论断:北美洲的人应该考虑经常吃点黄豆以防止疲劳和消沉。因为黄豆黄有一种叫I的化学成分,有防止得病的特征。最近一份研究发现北美洲的人比亚洲人多9次慢性疲劳和31次慢性消沉。而亚洲人每天会吃20克黄豆,北美人却一点都不吃。
        那种化学成份不一定能防止疲劳和消沉。
        论据有问题,调查没有提供数据,无从判断调查是否能代表两地的情况准确。但从结论来看忽略了一些因素,比如北美人和亚洲人体质有差别,北美人工作压力比亚洲人大等等。
        对亚洲人有效,不一定对北美人有效。

163.The following is taken from the editorial section of the local newspaper in Rockingham.
      "In order to save a considerable amount of money, Rockingham's century-old town hall should be torn down and replaced by the larger and more energy-efficient building that some citizens have proposed. The old town hall is too small to comfortably accommodate the number of people who are employed by the town. In addition, it is very costly to heat the old hall in winter and cool it in summer. The new, larger building would be more energy efficient, costing less per square foot to heat and cool than the old hall. Furthermore, it would be possible to rent out some of the space in the new building, thereby generating income for the town of Rockingham."
    为节省大量支出,Rockingham应该拆除具有百年历史的市政厅并用一些市民所提议的更大更节能的建筑来代替。旧的市政厅过于狭小,无法使政府雇佣的大量员工在里面舒适地工作。而且,旧市政厅冬天采暖和夏天制冷都很花钱。新的更大的市政厅将会更加节能,单位面积所需的采暖和制冷费用都比旧建筑少。还有,我们可以把新建筑的一部分空间出租,从而为Rockingham市增加收入。

century-old town hall 论断:为了省钱,R地该把老大堂推翻建一个更大更有效率的新大堂。因为老的太小无法使其雇用的人舒服地工作。另外它冬天供暖和夏天降温的费用太高。新的每平方尺供热降温的花销要比老的少,另外还可能把新楼里的一些地方租出去,为R创收。
        更大更有效的新楼花销会很大,怎么省钱?
        •新楼更大供暖降温的费用也会高,却没有证据表明它一定有人租它,创收的事儿是说不准的。
        老楼的费用可以有其他办法解决,比如更换设备。
        推翻老楼的负面影响,一历史文化价值,二旅游。

使用道具 举报

Rank: 2

声望
9
寄托币
139
注册时间
2009-4-2
精华
0
帖子
28
19
发表于 2009-4-26 22:14:23 |只看该作者
164.Claitown University needs both affordable housing for its students and a way to fund the building of such housing. The best solution to this problem is to commission a famous architect known for experimental and futuristic buildings. It is common knowledge that tourists are willing to pay money to tour some of the architect's buildings, so it can be expected that tourists will want to visit this new building. The income from the fees charged to tourists will soon cover the building costs. Furthermore, such a building will attract new students as well as donations from alumni. And even though such a building will be much larger than our current need for student housing, part of the building can be used as office space.
      Claitown大学需要为学生提供负担得起的住房和建造这些住房的融资途径。对于这一问题的最好解决办法就是请一位知名建筑师来设计一幢试验性和未来式的建筑。众所周知游客愿意花钱参观这名建筑师的一些建筑,因此我们可以预测游客将想要参观这座新建筑。从游客那里收取的钱将很快收回建楼的成本。而且,这样一座建筑将会吸引校友的捐款和新学生。尽管这样的建筑将比我们当前学生住房所需的大很多,但它的一部分可以用作办公空间。

Claitown University needs housing and way to fund the building 论断:最好的办法是雇用一个有经验有眼力的著名建筑师。因为一般旅游者都愿意花钱看一些建筑师的作品,所以会有旅游者来看新楼。这样这项收入可以很快抵消建楼的费用。另外这个楼会吸引新的学生以及校友的捐款。尽管这个楼会比现在学生住宿需求大,但一部分可以用来办公。
        学生宿舍是否能作旅游点。尽管会有旅游者去看一些大师的建筑物,但是否有人愿意花钱去看学生起居的地方,而且学生会不同意的,因为宿舍是他们的私人空间。这项收入不一定够。学校被视为非赢利机构,任务是培养学生,不能把建楼往商业用途上引
        •没有证据表明会有校友愿意为此捐款,仅仅是猜测而已。
        学生宿舍不太可能适合办公,而且现在的办公空间够了,外面也不见得有人要租。

165.The following appeared in a business magazine.
    "As a result of numerous consumer complaints of dizziness and nausea, Promofoods requested that eight million cans of tuna be returned for testing last year. Promofoods concluded that the cans did not, after all, contain chemicals that posed a health risk. This conclusion is based on the fact that the chemists from Promofoods tested samples of the recalled cans and found that, of the eight chemicals most commonly blamed for causing symptoms of dizziness and nausea, five were not found in any of the tested cans. The chemists did find that the three remaining suspected chemicals are naturally found in all other kinds of canned foods."
     去年由于有大量消费者投诉说产生了眩晕和恶心,Promofoods要求800万罐金枪鱼回收检测。Promofoods下结论说这些罐头根本不含任何有健康危害的化学物质。这一结论基于这样的事实,即Promofoods的化学家测试了回收的罐头样本,并发现8种最常见的导致眩晕和恶心症状的化学物质中,有五种都没有在任何被测试的罐头中被发现。化学家确实发现剩下三种受怀疑的化学物质在所有其他罐头食品中都存在。

dizziness and nausea,tuna can 论断:金枪鱼罐头不含造成健康危险的化学成分。P的结论基础是化学家对收回的罐头样品的测试发现八种最常造成两种病症的化学成份中有五种这些罐头里根本没有,而被发现的另外三种仍有怀疑的化学成份在所有罐头食物里都有。
        八种常见的以外还有其他不常见但同样会造成头晕恶心的。
        三种哪儿都有的可能含量超标。另外食物不同作用也会不同。
        样品选取是否科学,另五种还是有可能存在于其它的里面,或是已售出的罐头的。

166.The following appeared in a local newspaper.
      "People should not be misled by the advertising competition between Coldex and Cold-Away, both popular over-the-counter cold medications that anyone can purchase without a doctor's prescription. Each brand is accusing the other of causing some well-known, unwanted side effect: Coldex is known to contribute to existing high blood pressure and Cold-Away is known to cause drowsiness. But the choice should be clear for most health-conscious people: Cold-Away has been on the market for much longer and is used by more hospitals than is Coldex. Clearly, Cold-Away is more effective."
    人们不应该被任何人不用医生处方就可以买到的OTC药品Coldex和Cold-Away之间的广告战所误导。每个牌子都指责另一种药会导致某种众所周知的不良副作用:Coldex导致血压升高而Cold-Away导致嗜睡。但对于多数关心健康的人来说选择是明显的:Cold-Away比Coldex上市时间更长而且被更多的医院所使用。显然,Cold-Away效果更好。

advertising competition,over-the-counter cold medications 论断:CA更有效。因为CA推出的时间长,并被更多医院使用。无论CA还是C都会引起负作用,CA引起嗑睡,C引起高血压。大家都不应被两家的广告竞争所误导。没有直接证据证明CA更有效,而时间长,用得多不一定代表它就好。
        C是新产品,所以在市场上时间短。但它完全有可能用更好更新的技术成果,以及吸取CA的经验,从而比CA更有疗效。
        更多医院用不能说明好。首先可能是因为商业等其他原因使得医院用CA。其次,既然该种药品不需要医生建议可以柜台买,说明医生的专业知识并不是最好的建议。
结论:C的负作用在大多数人看来可能比较严重,所以限制了它的广泛使用。但是对于治疗感冒它还是有可能比CA有效,所以尤其对于没有高血压危险的人来说,CA不一定比C更有效。

167.A folk remedy* for insomnia, the scent in lavender flowers, has now been proved effective. In a recent study, 30 volunteers with chronic insomnia slept each night for three weeks on lavender-scented pillows in a controlled room where their sleep was monitored. During the first week, volunteers continued to take their usual sleeping medication. They slept soundly but wakened feeling tired. During the second week, the volunteers discontinued their medication. As a result, they slept less soundly than the previous week and felt even more tired. During the third week, the volunteers slept longer and more soundly than in the previous two weeks. This shows that over a short period of time lavender cures insomnia. A folk remedy is usually a plant-based form of treatment common to traditional forms of medicine, ones that developed before the advent of modern medical services and technology.
    一种治疗失眠的偏方--薰衣草花香,现在被证明是有效的。在一次最近的调查中,30名患有慢性失眠的志愿者在三周之内每晚都在一个受监视的控制室内睡在带薰衣草花香的枕头上。在第一周,志愿者继续服用他们常用的安眠药。他们睡得很沉但醒来时很累。在第二周,他们不服用药物。结果与前一周相比他们睡得不那么沉并且感觉更累。在第三周,他们睡得比前两周时间长而且更深。这表明薰衣草在短时间内治愈了失眠。
   *偏方通常是一种在传统医药中常见的植物治疗形式,这些传统医药是在现代医药服务和科技出现之前发展起来的。

lavender cure insomnia 论断:薰衣草对治疗失眠有效。因为最近的研究中,30位有失眠症的志愿者在三个星期里睡在有薰衣草药片的房里。第一个星期志愿者继续服用普通的失眠药,他们睡得很香但是醒来会觉得累。第二个星期,志愿者停药,结果他们睡得不那么香感觉更累。到第三个星期,志愿者睡得比前两上星期都要香要长。推论以研究为基础,但这还只是初步的研究,并不足以得出医学上的结论。因为30位以及三个星期都不足以说明问题。但是抛开这些不考虑,论者的推理过程仍有错误。
        第三个星期志愿者醒来以后的感觉论者没有提供。是比前两星期更加疲劳还是什么。事实上论者提到了志愿者在停药的第二个星期里睡眠非常不好,这有可能是第三个星期睡的时间长的主要原因,因为他们实在太累。所以没有醒来后的感觉作标准,无法评价志愿者的睡眠质量,因为无法知道熏衣草的作用。
        研究本身对志愿者有影响。志愿者三个星期里的睡眠变化与对环境的熟悉有关,第三个星期完全熟悉了,所以不紧张了。 •在短期内的概念不清楚。

168.Typically, as people age, their bone mass decreases, making them more vulnerable to bone fractures. A recent study concludes that the most effective way to reduce the risk of fractures in later life is to take twice the recommended dose of vitamin D and calcium daily. The three-year study followed a group of French women in their eighties who were nursing-home residents. The women were given daily supplements of twice the recommended dose of vitamin D and calcium. In addition, the women participated in a light weightlifting program. After three years, these women showed a much lower rate of hip fractures than is average for their age.
    通常当人们衰老的时候,他们的骨质减少,使他们容易骨折。最近一项研究认为在老年减少骨折危险的最好办法就是每天服用双倍于推荐用量的维生素D和钙。这项为期三年的研究跟踪了一组在养老院生活的80多岁的法国妇女。她们每天被给予两倍于推荐用量的维生素D和钙。而且,这些妇女参加了轻微的举重活动。三年之后,这些妇女髋关节骨折的发生率低于同龄人的平均水平。

age people bone mass decreases,vulnerable to fractures 论断:降低骨折的最有效途径是以后每天吃两次推荐数量的VD和钙。一项对一组看家的八十岁法国女人为期三年的研究发现,这些女人每天吃两次含推荐量的VD和钙营养品。另外她们还参加一个轻度的增肥计划。三年后,这些女人表现出比她们年纪平均要低得多的屁股骨率。论断的代表性还值得讨探,没有提人数,又仅限于某种女人。
        屁股骨折少了,其它骨折没提。
        没有排除其它因素,比如增肥计划,或是她们比以前更加小心了。另外营养品里是不是还有其它成份在起作用。
        没有直接的证据证明VD和钙能够counteract the vulnerability caused by the bone mass decrease.

169.The following appeared in a letter from a department chairperson to the president of Pierce University.
     "Some studies conducted by Bronston College, which is also located in a small town, reveal that both male and female professors are happier living in small towns when their spouses are also employed in the same geographic area. Therefore, in the interest of attracting the most gifted teachers and researchers to our faculty and improving the morale of our entire staff, we at Pierce University should offer employment to the spouse of each new faculty member we hire. Although we cannot expect all offers to be accepted or to be viewed as an ideal job offer, the money invested in this effort will clearly be well spent because, if their spouses have a chance of employment, new professors will be more likely to accept our offers."
     对于同样位于小城市的Bronston学院所作的一些研究发现,不论男性或女性教授,当他们的配偶在相同的城市有工作时,就更乐于在这些小城市生活。因此,为了吸引最有天分的教师和研究人员加入我们的员工队伍并提高我们员工的士气,我们Pierce大学应该为每一个我们所雇佣的新员工的配偶提供就业机会。尽管我们并不期望所有就业机会都会被接受或被看作是理想的工作,我们在这项努力上的投入显然是值得的,因为如果新教授的配偶有就业机会,他们将更乐于在我们学校就职。

professors happier living when spouses in the same geographic area 论断:为吸引最有才能的老师和研究者来我们学校并提高整体员工的士气,我们应该给每位新员工的配偶提供工作机会。因为如果配偶有工作的机会,新教授会更愿意来我们这儿。尽管不可能所有位子都被接受或是看成理想的选择,花在这上的钱也会很值。一个也在小城市的B校的研究发现,男女教授都会更快乐地生活在小镇上,当他们的配偶也在同一个地区工作时。 B校的研究描述不清楚,无法判断是否在一起生活会比不在一起生活更快乐,所以无法判断是否准确。但另外推论中也同样有问题。
        提高士气,愉快的夫妻生活不保证高的士气,校风,工作,工资都很重要
        在一起生活愉快,但没有证据证明配偶的工作地点是教授们选择工作的唯一或重要标准。所以不一定更吸引人。
        如果这笔花费不小,就不能给每个新员工都提供。
        最有才能的?

170.For the past five years, consumers in California have been willing to pay twice as much for oysters from the northeastern Atlantic Coast as for Gulf Coast oysters. This trend began shortly after harmful bacteria were found in a few raw Gulf Coast oysters. But scientists have now devised a process for killing the bacteria. Once consumers are made aware of the increased safety of Gulf Coast oysters, they are likely to be willing to pay as much for Gulf Coast as for northeastern Atlantic Coast oysters, and greater profits for Gulf Coast oyster producers will follow.
     过去五年中,加州的消费者愿意花比Gulf Coast牡蛎贵一倍的价格来购买东北地区的Atlantic Coast牡蛎。这种趋势发生在一些Gulf Coast生牡蛎被查出含有有害细菌之后。但科学家现在已经设计了一套杀灭这些细菌的工艺。一旦消费者发觉Gulf Coast牡蛎的安全性提高了,他们将会愿意付出和东北的Atlantic Coast牡蛎同样的价格来购买它,并为Gulf Coast牡蛎的经营者带来更大的利润。

consumers in California,oysters from the northeastern Atlantic Coast 论断:公众知道有杀死细菌的方法,GC地区的牡蛎生产者就会有更高的利润。因为顾客会觉得安全而愿付与东北牡蛎一样的价格买CG的牡蛎。过去五年加州的消费者愿意付比GC高一倍的钱去买东北牡蛎,这个趋势是从GC牡蛎里发现有害细菌开始的。
        顾客是否会用与现在东北牡蛎一样的价格买GC的。发现GC有细菌使得东北牡蛎价格上涨,没有细菌东北牡蛎价格将来有可能会下降,GC牡蛎价格可能还是这样,尽管可能有不被保证有比目前上升。
        可以杀死细菌也不一定买GC,可能味道,保鲜等等,在发现细菌前就不买它了。
        有人用高于现在的价格买,也不带来赢利。杀菌的花销。
        最后最关键的一点是,reputation is easy to destroy but hard to regain. Even aware of the recent devise to kill the bacteria, consumers very probobaly still afraid of this problem. If they withold their credence to such device, they would still not buy GC oyster. It will maybe a long time and need astrenuous work to conciliate their fear.

171.The following appeared in a memo from the marketing director of Bargain Brand Cereals.
     "One year ago we introduced our first product, 'Bargain Brand' breakfast cereal. Our very low prices quickly drew many customers away from the top-selling cereal companies. Although the companies producing the top brands have since tried to compete with us by lowering their prices, and although several plan to introduce their own budget brands, not once have we needed to raise our prices to continue making a profit. Given our success selling cereal, Bargain Brand should now expand its business and begin marketing other low-priced food products as quickly as possible."
    一年以前我们推出了第一种产品Bargain Brand早餐麦片。我们低廉的价格迅速从一些销量最大的麦片厂商那里吸引了大量客户。尽管这些厂商曾经试图通过降低价格来与我们竞争,并且尽管有一些公司打算推出他们自己的廉价品牌,但我们从未需要通过涨价来持续盈利。基于我们销售麦片的成功,Bargain
Brand应该扩展业务,并且尽快开始推出其他的廉价食品。

'Bargain Brand' breakfast 论断:BB应该尽快拓展业务并开始推出其他低价食品。因为我们卖粮食的成功。一年前我们的第一个产品BB早餐粮食因为很低的价格很快把顾客从其他畅销粮食公司拉了过来。尽管那些公司一直用压低价格与我们竞争,而且还有几家计划推出他们自己的品牌,但我们都还不曾需要提高我们的价格来继续赢利。
        前提是否成功---是否在赢利。没有资料证明在赢利。低价低利润率,而且知名品牌都降价了,如果你提价就会失去顾客。用一个需要证明的,而且可能不存在的原因,解释一个现象。而且这个现象的真实情况有可能会削弱结论。
        论据:顾客选择BB,最主要的可能不是因为低价,而是其他因素。不然别人降价,顾客早回去了。
结论:一个产品因为低价成功,不保证其他市场低价也能成功。尽快拓展业务,还要考虑大的经济环境,和顾客的状况。

172.The Mozart School of Music should obviously be the first choice of any music student aware of its reputation. First of all, the Mozart School stresses intensive practice and training, so that students typically begin their training at a very young age. Second, the school has ample facilities and up-to-date professional equipment, and its faculty includes some of the most distinguished music teachers in the world. Finally, many Mozart graduates have gone on to be the best known and most highly paid musicians in the nation.
     Mozart音乐学校显然应该是所有注意到其声誉的学音乐学生的第一选择。首先,Mozart学校强调集中的练习和训练,因而学生通常在很小的时候就开始接受训练。其次,学校拥有充足的师资和最先进的专业器具,其员工包括一些全球最著名的音乐教师。最后,很多Mozart的毕业生已经成为全国最有名而且收入最高的音乐家。

Mozart School 论断:M校应该显然是任何知道其他声誉的音乐学生的首选。因为,1、M校对练习和培训非常重视,所以一般学生在非常小的时候就开始训练。2、学校有大量最先进的教学设备,而且它很多教师都是世界上最好的。3、很多M的毕业生是全国最知名且大多数收入非常高的音乐家。学校好,学费贵不贵?另外给出的三个原因都有问题。
        早期开始大量练习。首先这样不一定好,比如抑制学生想象力等等,这是教育界一直比较争论的。其次对于学生来说,严格的训练可能会吓着他们而不愿去。第三,很早期开始训练,多早,年纪稍大的岂不是不行了。坦率说,这是一条weaken. •先进设备和好的老师。先进设备给不给学生用,好的老师还教不教学。
        音乐家的问题,没给数字和证据,无法想信其客观。

173.The following is a memorandum issued by the publisher of a newsmagazine, Newsbeat, in the country of Dinn.
     "Our poorest-selling magazine issues over the past three years were those that featured international news stories on their front covers. Over the same period, competing news-magazines have significantly decreased the number of cover stories that they devote to international news. Moreover, the cost of maintaining our foreign bureaus to report on international news is increasing. Therefore, we should decrease our emphasis on international news and refrain from displaying such stories on our
magazine covers."
     过去三年中我们销量最低的几期杂志是在封面上刊登了国际新闻故事的那几期。在同一时期,与我们竞争的几种新闻刊物显著减少了刊登在封面上的国际新闻数量。而且,我们用于维持国外部报导国际新闻的费用正在增加。因此,我们应该减少对于国际新闻的强调并且避免把这类消息刊登在杂志封面上。

poorest-selling magazine 论断:我们应该减少对国际新闻的重视,并控制这样的文章出现在杂志封面。因为过去三年我们卖的最不好的杂志是重点且封面报道是国际新闻的。同时,竞争对手已大量削减对国际新闻报道的数目。另外,我们维持驻外站的费用在升高。
        被忽略的前提:读者是否不喜欢看国际新闻。没有证据。关于销售问题,有可能是报道写得的不好,或是杂志发行出了问题,都不能说明国际新闻本身不吸引读者。
        驻外站的费用是否还会上升,上升是否还能承受。
        竞争对手减少对国际新闻的关注,但却不知其销量是否上升。而且有可能是他们力不从心了,这样这个市场全是我们的了。

174.The following recommendation was made by the president and administrative staff of Grove College, a private institution, to the college's governing committee.
    "We recommend that Grove College preserve its century-old tradition of all-female education rather than admit men into its programs. It is true that a majority of faculty members voted in favor of coeducation, arguing that it would encourage more students to apply to Grove. But eighty percent of the students responding to a survey conducted by the student government wanted the school to remain all female, and over half of the alumni who answered a separate survey also opposed coeducation. Keeping the college all-female, therefore, will improve morale among students and convince alumni to keep supporting the college financially."
     我们建议Grove学院保留其已有百年历史的女校传统,而不是允许录取男性。确实有大部分员工投票赞成男女同校,认为这会使更多的学生申请Grove。但由学生会所组织的一次调查中,有80%的被访学生要求学校维持女校形式,并且在回应了一次单独调查的校友中超过一半的人也反对男女同校。因此,维持女校形式将促进学生的精神状态并且让校友继续对学院进行财政资助。

century-old tradition of all-female education 论断:维持全女生,会提高学生的士气并让校友继续支持学校的财政。因为学生会做的调查中80%的被访者希望学校维持全女性教育,一半以上的校友在另一个调查中也反对联合教育。尽管大多数的教师投票赞同联合教育,认为这样可以使更多的学生申请我们。
        前提,联合教育会降低士气减少校友支持,没有证据。
        论据,两校调查都没有具体的总数,调查本身的可性度不高。而且可能会有其他问题使得他们没有说实话,比如女生可能都比较害羞。而且同意改变就意味着对现状不满意,有人会影响这种心理而不说实话。
        比较扩大申请带来的收入和校友捐资减少。
结论:尽管学生真不喜欢联合教育,学校也应该从教育的目的考虑,不能一味顺从学生。社会是男女共存的,联合教育可能比全女生教育更合适。有可能80%就已经说明学生有问题了。

175.The following appeared in a letter to the school board in the town of Centerville.
     "All students should be required to take the driver's education course at Centerville High School. In the past two years several accidents in and around Centerville have involved teenage drivers. Since a number of parents in Centerville have complained that they are too busy to teach their teenagers to drive, some other instruction is necessary to ensure that these teenagers are safe drivers. Although there are two driving schools in Centerville, parents on a tight budget cannot afford to pay for driving instruction. Therefore an effective and mandatory program sponsored by the high school is the only solution to this serious problem."
     Centerville高中的所有学生都应该被要求参加驾驶员教育课程。在过去两年中,Centerville周围有几起交通事故涉及了青少年驾驶者。由于Centerville的一些家长抱怨说他们太忙没有时间教他们的孩子驾驶,必须有一些其他的课程来保证这些孩子成为安全的驾驶者。尽管在Centerville已经有两所驾校,手头不宽裕的家长无法负担驾校学费。因此由学校组织的有效的强制性课程是解决这一严重问题的唯一方案。

All students should take the driver's education 论断:所有的学生必须参加C校的驾驶课程。因为由高中资助的有效且必须参加的驾驶教育课程是唯一解决这个严重的少年驾驶问题的办法。过去两年里C及其周围有几起有少年司机有关的事故。由于很多父母都抱怨他们太忙没时间教孩子开车,所以需要有其它方式保证这些孩子是安全驾驶员。虽然C有两所教校,但穷父母供不起驾驶课。
        前提,是否唯一的办法:其它办法,不是所有的父母都供不起,学校教开车没有驾校的专业。
        论据,是否需要开驾驶课。出的事帮是否是C地区的,事故原因是否是因为这些少年。又是否是因为他们缺乏指导,有可能他们已经非常熟练了。there is a adage saying:"skilled swimmer gets killed in water。
结论:为什么得是所有学生,有的学生可能根本就不想开车。

176.The following is a memorandum from the sales director to the president of the Healthy-and-Good food company.
    "A recent study indicates that Venadial, a new margarine currently produced only in the country of Alta, actually reduces cholesterol levels. Derived from the resin of pine trees, Venadial works by activating a metabolic response that is not yet well understood. However, cholesterol levels fell ten to fifteen percent among participants in the study who consumed Venadial daily, and the risk of heart attack by one-third. In addition, the new margarine is so popular that stores in Alta are unable to keep it on their shelves. Therefore, if our company obtains the exclusive right to sell Venadial internationally, our profits are sure to increase substantially within a very short time."
    最近一次研究表明,当前仅在Alta生产的新的人造黄油Venadial实际可以降低胆固醇指标。Venadial从松树树脂中提炼而来,它通过一种现在还未完全明确的代谢反应发生作用。然而,在那些每天食用Venadial的研究对象中,胆固醇指标下降了10-15%,心脏病发病率下降了三分之一。而且,这种新的人造黄油非常受欢迎以至于在Alta的商店中往往一抢而光。因此,如果本公司取得在Venadial的全球独家代理权,我们的利润肯定能够在很短的时间内显著增长。

a new margarine 论断:如果我们公司获得销售V的国际专营权,我们的利润一定会在很短时间内有显著增长。因为V能降低胆固醇,而且这个新的产品非常受欢迎,以至V的产地A的商店跟不上供应。一份研究显示V,从松树的上炼取树脂,可以促进新陈代谢的某种反应目前还不是很清楚。但在研究中,每天吃V的人胆固醇含量下降了10%到15%,且心脏病危险少三分之了。
        前提,A地销量好,不一定全球就好。有的国家胆固醇含量没那么引人关注,有的地方只用real butter.
        获得专营权没用,既然是炼松脂,会有其它公司也能生产类似的产品。甚至用其他原料生产同样效果的产品。
        负作用的问题没有考虑。response is not yet well understood. •论据的有效性不明。

177.The following is a letter that recently appeared in the Oak City Gazette, a local newspaper.
     "Membership in Oak City's Civic Club-a club whose primary objective is to discuss local issues-should continue to be restricted to people who live in Oak City. People who work in Oak City but who live elsewhere cannot truly understand the business and politics of the city. It is important to
restrict membership to city residents because only residents pay city taxes and therefore only residentsunderstand how the money could best be used to improve the city. At any rate, restricting membership in this way is unlikely to disappoint many of the nonresidents employed in Oak City, since neighboring Elm City's Civic Club has always had an open membership policy, and only twenty-five nonresidents have joined Elm City's Club in the last ten years."
    Oak City的市民俱乐部主要目的是讨论本地事务,其成员资格应该继续被限制在居住在Oak City的市民。那些在Oak City工作但在别的地方居住的人无法真正理解本城市的政治和经济。把成员资格限制在本市居民非常重要,因为只有居民付城市税因而也只有他们才知道这些钱如何使用才能促进城市发展。无论如何,以这种方式限制成员资格不太可能使很多在Oak City工作的外地人失望,因为邻近城市Elm City的市民俱乐部一直采取开放成员资格的政策,但在过去10年中只有25名外地人加入了Elm City的市民俱乐部。

Membership in Oak City's Civic Club 论断:O城的市民俱乐部应该继续只让住在O城的居民参加。因为只有居民付城市税,所以他们知道钱要怎么花才最好地促进这个城市,而在O城工作但住别地儿的人不可能真正懂得这个城市的商业和政策。另外限制也不会让非本城居民的员工失望,因为隔壁E城不限制,但是过去十年里只有25个非居民会员参加。
        前提,不住在本城就不懂得本城。不交税就关心本城,没有证据。另外由于这些人有不同城市的经历,应该好好利用。不然只局限于本地,可能眼界狭窄。
        不会让非本地员工失望,论据不足信。E城可能是小城,没多少外地人关心和工作,但O可能经济要发展一些,吸引了很多外地人。进行讨论就是一个很好的表现。

178.The following appeared in the annual report from the president of the National Brush Company.
     "In order to save money, we at the National Brush Company have decided to pay our employees for each brush they produce instead of for the time they spend producing brushes. We believe that this policy will lead to the production of more and better brushes, will allow us to reduce our staff size, and will enable the company factories to operate for fewer hours-resulting in savings on electricity and security costs. These changes will ensure that the best workers keep their jobs and that the company will earn a profit in the coming year."
    为节省开支,我们National Brush公司决定不再按员工生产刷子的工时支付工资,而是按他们生产刷子的数量来支付。我们相信这一政策将会导致刷子产量和质量的提高,将会减少我们的员工数量,并将会使工厂能够减少运营时间从而导致电费和保安费用的节省。这些改革将保证那些最好的工人留在这里工作,公司将会在来年有更多的盈利。

National Brush Company 论断:我们应该由按员工产B个数算钱,而不再按时间算钱。因为我们相信这个政策会带来更多更好的产生,可以让我缩减员工人数,而且生产时间减少可以节省电和安全的开销。这些变化将保证最好的工作留住工作,公司也将在明年获得赢利。
        被忽略的,这种方式是否适合:按时间算是一个方便的方法,按个数算得有很多人去计每个工人的工作量。另外制作不可能由一个工人全部完成,stream lining production,这种方式都有问题。
        更多更好的生产:更多有可能,但质量肯定下降。工人求数量多嘛。质量不好,就有可能遭来投诉,销量下降,利润就没了。get no profit but deficit.
        省电省安全:工人求速度,可能会导致更多的安全事故。
        工人,不是最好的,是最快的,还有可能是最会投机取巧的。

179.The following is a memorandum written by the director of personnel to the president of the Cedar Corporation.
     "It would be a mistake to rehire the Good-Taste Company to supply the food in our employee cafeteria next year. It is the second most expensive caterer in the city. In addition, its prices have risen in each of the last three years, and it refuses to provide meals for people on special diets. Just last month three employees complained to me that they no longer eat in the cafeteria because they find the experience 'unbearable.' Our company should instead hire Discount Foods. Discount is a family-owned local company and it offers a varied menu of fish and poultry. I recently tasted a sample lunch at one of the many companies that Discount serves and it was delicious-an indication that hiring Discount will lead to improved employee satisfaction."
     继续雇佣Good-Taste公司在明年为我们员工餐厅提供饮食将是一个错误。它是本市价格第二贵的餐饮服务商。而且,其价格在过去三年中持续上涨,并且它拒绝为那些有特殊饮食要求的人提供服务。上个月有三名员工向我投诉说他们再也不会在员工餐厅用餐了,因为他们发现那种经历"无法忍受"。我们公司应该雇佣Discount Foods。Discount是一个本地的家族企业,它提供鱼类和禽类的多种菜单。我最近在由Discount提供服务的公司之一品尝了一份样品,味道很好,这说明雇佣Discount将会带来更高的员工满意度。

food in employee cafeteria 论断:公司应该用D公司,重新雇用G是个错误。因为G是这个城市第二贵的,他们的价格过去三年里一直在涨,而且不给饮食有特殊要求的供餐。上个月就有三个员工向我抱怨他们不再在食堂吃饭了,因为经历不可忍受。雇用D,D是一个当地家庭公司,它的菜单有各种各样的鱼和猪肉。我最近在一个他们服务的公司吃了个品尝午餐,味道很不错——这是一个用他们会提高员工满意程度的标志。
        前提G不好,three employee and just last month does not make sense. Maybe for their insatiable contentment or just happenchances. Perhaps a survey over this topics should be carried before any conclusion arrived.
        D好,证据不够。首先,the arguer's gustation, however just once, cannot represent that of the whole compnay's. It is very possible that the majority of employees reject the food served by D. 其次,"Delicious" does not equal nutrition, which is of more importance in judging the food service.
        价格问题,G说了干嘛不提D的,它可能也挺贵的。而且既然是家小公司,hygiene and amout quality is problematic.

使用道具 举报

Rank: 2

声望
9
寄托币
139
注册时间
2009-4-2
精华
0
帖子
28
20
发表于 2009-4-26 22:15:55 |只看该作者
180.The following is a recommendation from the personnel director to the president of Acme Publishing Company.
     "Many other companies have recently stated that having their employees take the Easy Read Speed-Reading Course has greatly improved productivity. One graduate of the course was able to read a five-hundred-page report in only two hours; another graduate rose from an assistant manager to vice president of the company in under a year. Obviously, the faster you can read, the more information you can absorb in a single workday. Moreover, Easy Read costs only $500 per employee-a small price to pay when you consider the benefits to Acme. Included in this fee is a three-week seminar in Spruce City and a lifelong subscription to the Easy Read newsletter. Clearly, Acme would benefit greatly by requiring all of our employees to take the Easy Read course."
     很多其他公司最近指出他们的员工参加了Easy Read的速读课程之后生产效率显著提高。这个课程的一名毕业生能够在两个小时之内读完长达500页的报告,另一名毕业生在一年内从助理经理上升到了副总裁。显然,你的阅读速度越快,在一天之内所能吸收的信息就越多。而且,Easy Read的学费只有每人500元,当考虑到它给Acme带来的效益的时候这就是个小数目。这个费用包括在Spruce City举行的为期三周的研讨会和Easy Read新闻刊物的终生赠阅。显然,Acme通过要求所有员工参加Easy Read的课程将会极大地受益。

Easy Read Speed-Reading Course, Publishing Company 论断:A让所有员工参加快速阅读课程会收获很大。因为很多其它工司说他们参加这个课程的员工生产力提高很大,这个课程的一个毕业生两个小时里能五百页的报告,另一个毕业生在一年之内就从经理助理升到了副总裁。显然,读得越快,你一个工作日里获得的信息就越多。另外这个课程花费只有五百元,这与从其受到的利比是非常低的。它包括在云杉城三个星期的课程和一生的快速阅读新闻信。
        前提收获大,证据有问题。首先,两个人的例子不说明问题,这个课程的大多数学员怎么样?第二五百页那人,看的是什么内容,又记住了多少。第三当副总的那人,他升职的原因可能跟阅读完全没有关系。
        前提,对我们是否有用。公司不同,员工不同,工作内容不同。如果我们的员工阅读已经很好了,就不用了吧。
结论:有用也不用所有人都去呀,还有一些岗位根本用不着快速阅读。而且还得要员工都带薪放三个星期的假。

181.From a letter to the editor of a city newspaper.
     "One recent research study has indicated that many adolescents need more sleep than they are getting, and another study has shown that many high school students in our city are actually dissatisfied with their own academic performance. As a way of combating these problems, the high schools in our city should begin classes at 8:30 A.M. instead of 7:30 A.M., and end the school day an hour later. This arrangement will give students an extra hour of sleep in the morning, thereby making them more alert and more productive. Consequently, the students will perform better on tests and other assignments, and their academic skills will improve significantly."
     最近一项研究表明,很多青少年需要更多的睡眠,另外一项研究表明我们市很多高中学生对于自己的学习成绩不满意。作为解决这些问题的途径,我市的高中应该在早上8:30开始上课,而不是7:30,并且推迟一小时放学。这种安排将允许学生在早上多睡一个小时,从而使他们更加清醒和高效。因此,学生在测验和其他作业中将表现得更好,他们的学习水平将会显著提高。

adolescents need more sleep 论断:高中早上课应该推迟一个小时开始,下午推后一小时下课。这个安排能让学生早上多一个小时睡眠,从而使他们更有效率更精神。学生的测验和作业表现会提高,学术能力也会有很大进步。因为最近的研究显示很多青少年需要比现在更多的睡眠,而且另一项研究也显示我们城市的高中学生对自己的学校表现不满意。
        被忽略的前提,是否睡得多学校表现就会好,并没有这样的证据。而关于不满意自己的学生,也没说是因为自己睡得少,他们可能是对自己有高要求,要不就是觉得自己挺懒的。还有就是觉得老师不好。
        前提,这样做学生睡眠是否会加长。早上迟一小,可能多了一小时睡眠,但学生可能晚上晚睡一小时。这样睡眠时间是一样的。而且还有看电视。
结论:睡眠问题解决了,不一定就会学术能力提高。

182.Butter has now been replaced by margarine in Happy Pancake House restaurants throughout the southwestern United States. Only about 2 percent of customers have complained, indicating that 98 people out of 100 are happy with the change. Furthermore, many servers have reported that a number of customers who still ask for butter do not complain when they are given margarine instead. Clearly, either these customers cannot distinguish margarine from butter, or they use the term "butter" to refer to either butter or margarine. Thus, to avoid the expense of purchasing butter, the Happy Pancake House should extend this cost-saving change to its restaurants in the southeast and northeast as well.
    Happy Pancake House在美国西南部的餐厅用人造黄油代替了天然黄油。只有大约2%的顾客曾经投诉,这说明100个人中有98人对于这种替换是乐于接受的。而且,很多服务生报告说很多仍然要了天然黄油的顾客在被给予人造黄油的时候并没有投诉。显然,这些顾客要么分不清天然黄油和人造黄油,要么是用"天然黄油"这个词汇来指天然黄油或人造黄油。因此,为避免购买天然黄油的花费,Happy
Pancake House应该把这项节省措施推广到在东南部以及东北部的餐厅。

Butter has now been replaced by margarine 论断:H饭店应该在东南部地区和东北部地区也用人造黄油换成黄油这项省钱的改变。因为在美国西南部地区H城的这种改变,只引起了2%的顾客抱怨,这显示98%的人对这种变化挺高兴。另外很多服务商报告说很多仍然说要黄油的顾客,当他们得到的是人造黄油时也没有说什么。很清楚,要么是他们分辨不出来,要么就是他们用黄油这个名称称呼这两种产品。
        前提,顾客没意见。2%的顾客抱怨,不代表98%的就高兴。有人不在乎,有人在乎但不说,但下次会不再光顾。
        证据,要黄油的顾客拿到人造的不说什么,不表示他们分辨不出来,没资料显示顾客看不出来,或是不在乎。他们怕麻烦,或是不得己。但下次还是不去了。
结论:各地不一样,有些地方的人特别在乎人造的问题。还有地方不一样,货的成本可能也会有区别。

183.Many employees of major United States corporations are fearful that they will lose their jobs in the near future, but this fear is largely unfounded. According to a recent study, a majority of companies expected to make new hires in the coming year, while fewer companies expected to lay off employees. In addition, although it is very disturbing to be laid off, the proliferation of programs and of workshops designed to improve job-finding skills has made being laid off far less painful than it once was.
      很多美国大公司的员工担心他们在不久的将来可能失业,但这种担心是没有依据的。根据最近的一项研究,大部分公司准备在来年雇佣新员工,而更少的公司准备裁员。而且,尽管下岗确实很麻烦,但一些帮助人们提高求职技巧的培训项目的增加使下岗远没有以前那么痛苦。

employees are fearful lose their jobs in the near future 论断:害怕失去工作是没有根据的。因为根据一份最近的研究,大多数的公司准备在明年增加新雇佣员工,而更少的公司司准备裁员。另外尽管被裁员很让人烦恼,提高找工作的技巧的课程的增加也使失去工作没以前痛苦。根据一份调查,调查问题多多。
        大多数公司准备新雇人,而小部分公司有裁员计划。新雇人不证明不开除人,事实上是对老员工的一种潜在开除的危险。公司得控制一定规模。另外明年有计划招人,也不证明未来几年不裁人。小部分公司裁员人数可能很多,它们可能都是大公司,一裁好几千,而小公司招人才几十而己。
        调查的人说了实话。首先没有科学描述,其次准备裁员即意味着效益不好,有depression发生,公司不会愿意透露。
        课程的效果,效果不好找工作还是很费劲。而且在两项工作之间的日子会有多长,也还是让人disturbing.

184.In the Bayhead Public Library, books that are rarely borrowed continue to take up shelf space year after year, while people who want to read a recent novel frequently find that the library's only copy is checked out. Clearly, the library's plan to replace books that are borrowed no more than once a year with sufficient copies of more recent books will solve this problem. The protest we have heard since this plan was made public has come from a small, and thus unrepresentative, group of some thirty people and so should therefore be ignored.
      在Bayhead公共图书馆,那些很少被借阅的图书仍然长期占据着书库空间,而想要读最新小说的读者往往发现图书馆唯一的一本已经被借出了。显然,图书馆用充足的最新书籍来代替一年中被借阅不超过一次的书籍的计划将解决这一问题。这项计划公布后我们所听到的抗议来自一个30来人的很小的,因而也不具有代表性的团体,所以可以被忽略。

Public Library, books rarely borrowed take up shelf space year after year 论断:图书馆准备把每年借出次数不多的书换成多几份较新的书能够解决,有些书占书架,有些人们想借的书不够的问题。我们听到的反对意见是,因为这项计划已对公众宣布,来自一个小的没有代表性的30多个人的队伍,所以可以被忽略。
        前提,能解决问题,首先新书迟早也成旧书,如果因为旧而被借的少,那么这些书将来也会借的人少。其次,常被借的书以及所谓新书有可能多是期刊,流行畅销书,很快就会失去意义,而很多旧书虽然一年里借出的次数不多,但每年都会有人借,包括以后。
        会带来的问题,这样做的结果是图书馆的藏数种类减少。因为图书馆的空间是有限的。
        一部分人的意见也不能忽略,虽然对外公布也有很多人可能不知道,另外很多人采用沉默表达反抗,30个人已经很多了。或许他们还很有权威。

185.The following appeared in a letter from the owner of the Sunnyside Towers apartment building to its manager.
     "One month ago, all the showerheads on the first five floors of Sunnyside Towers were modified to restrict the water flow to approximately 1/3 of its original force. Although actual readings of water usage before and after the adjustment are not yet available, the change will obviously result in a considerable savings for Sunnyside Corporation, since the corporation must pay for water each month. Except for a few complaints about low water pressure, no problems with showers have been reported since the adjustment. Clearly, restricting water flow throughout all the 20 floors of Sunnyside Towers will increase our profits further."
     一个月之前,Sunnyside塔楼最低的五层的所有淋浴喷头被调节成水压只有以前的大约三分之一。尽管在调节之后用水量的确切读数还没有出来,但这种变革显然将为Sunnyside公司节省大量的花费,因为公司必须每月为所用的水付费。除了关于低水压的几起投诉,在调节之后没有发生关于淋浴喷头的问题的报告。显然,在Sunnyside塔楼所有的20层都限制水压将会增加我们未来的利润。

showerheads 论断:对20层的S塔楼限制水流,能够进一步提高我们的利润。因为这样可以为公司节省很多钱。一个月前,下面五层楼的淋浴喷头被调整,限制了三分之一的水流量。虽然调整后具体的用水指数还不知道,这个变化显然会为公司节约大量的钱。除了一些关于水压的抱怨外,没有别的关于喷头的抱怨。
        前提,是否节水了。既然还不知道具体的指数就不排除用水量上升的可能。因为水流变小,用水时间就会加长。另外由于水压的冲洗作用没了,这种时间上的补偿很有可能会特别长。
        前提,没人抱怨。首先刚一个月就有了抱怨。其次楼层越高对水压要求越到。第三楼上的人可能特殊,他们用水特别多,特别喜欢大流量的洗澡。时间长必然引起楼主不高兴。
结论:利润,收入得靠卖房,有可能是租房,水费只是其中很小一部分成本。

186.The following is a recommendation from the director of personnel to the president of Professional Printing Company.
      "In a recent telephone survey of automobile factory workers, older employees were less likely to report that having a supervisor present increases their productivity. Among workers aged 18 to 29, 27 percent said that they are more productive in the presence of their immediate supervisor, compared to 12 percent for those aged 30 or over, and only 8 percent for those aged 50 or over. Clearly, if our printing company hires mainly older employees, we will increase productivity and save money because of the reduced need for supervisors."
      在最近一次对汽车工厂工人的电话调查中,年纪大一些的员工更少报告说有管理员在场会提高他们的生产效率。在18到29岁的员工中,27%的人说当他们的顶头上司在场时更有效率,相比之下,30岁及以上的工人只有12%,50岁及以上的工人只有8%这样认为。显然,如果我们印刷公司主要雇佣年级大一些的员工的话,我们的劳动生产率将会提高,并且节省开支,因为对于管理员的需求将会减少。

older employees were less likely having a supervisor 论断:我们印刷公司主要雇用老员工的话,将提高生产率并省下雇指导员的钱。因为一份对汽车工人的电话调查里,老的员工比较少报告当指导员在现场时会提高他们的生产力。
        前提,省钱。老工人一般要求的工资都比年轻人高,指导员的钱省下来了,工资却得长。
        前提,提高生产力。没有证据证明年轻人的生产力比老工人的低,不管在不在场的问题。而且年轻人一般体力好,少得病,少乱七八糟的事。
        论据,太弱。首先,电话调查查的范围,各有多少人,是否准确。其次,调查里没指导员时年轻人的生产力还有可能比老工人高,尽管有人在场时会更高。
结论:两家公司工作不一样,员工要求不一样,没有可比性。

187.The following appeared as part of an article in a health magazine.
    "A new discovery warrants a drastic change in the diets of people living in the United States. Two scientists have recently suggested that omega -3 fatty acids (found in some fish and fish oils) play a key role in mental health. Our ancestors, who ate less saturated fat and more polyunsaturated fat, including omega -3 fatty acids, were much less likely to suffer from depression than we are today. Moreover, modern societies-such as those in Japan and Taiwan-that consume large quantities of fish report depression rates lower than that in the United States. Given this link between omega
-3 fatty acids and depression, it is important for all people in the United States to increase their consumption of fish in order to prevent depression."
     一项新发现表明生活在美国的居民饮食将会发生显著变化。两名学者最近指出omega-3脂肪酸(它在一些鱼类以及鱼油中被发现)在精神健康方面有很重要的作用。我们的祖先食用更少的饱和脂肪和更多的多重不饱和脂肪,包括omega-3脂肪酸,他们患抑郁症的可能性比我们现在要低很多。而且,食用大量鱼类的现代社会--比如象日本和台湾那样的社会--抑郁症的上报率比在美国要低。基于以上omega-3脂肪酸和抑郁症的联系,让所有的美国人增加鱼类的食用量以防止抑郁症是很重要的。

omega -3 fatty acids (found in some fish and fish oils) 论断:对所有美国人来说更多吃鱼来防止消沉很重要。因为两个科学家最近说O酸这种在一些鱼和鱼油里发现的东西对心理健康很有影响。我们的先人就少吃饱和脂肪多吃不饱和脂肪,其中就包括O酸,他们就比我们今天少消沉。另外,一些现代社会比如大量食用鱼的日本和台湾,他们报告消沉的论就比美国少。
        前提,吃鱼防止消沉。没有证据,两个科学家是说在某些鱼里发现的O酸,不是所有的鱼里都含有O酸。概念偷换。
        证据,科学家的证据值得怀疑:先人与我们社会不同、经济状况不同、生活节奏不同,人际关系复杂度不同,当然不能比。其次:台湾和日本,那等小国情况相对美国可能会简单一些。也没有资料证明他们少消沉就是因为吃了鱼。其它情况问题,比如经济状况,社会问题等等。
结论:干嘛所有人呀,有人对鱼敏感呢,还有些人吃得也太多了吧。

188.A new report suggests that men and women experience pain very differently from one another, and that doctors should consider these differences when prescribing pain medications. When researchers administered the same dosage of kappa opioids-a painkiller-to 28 men and 20 women who were having their wisdom teeth extracted, the women reported feeling much less pain than the men, and the easing of pain lasted considerably longer in women. This research suggests that kappa opioids should be prescribed for women whenever pain medication is required, whereas men should be given other kinds of pain medication. In addition, researchers should reevaluate the effects of all medications on men versus women.
      一项新报告表明,男性和女性对于疼痛的感受是有显著差异的,医生在开止痛药方的时候应该考虑到这种差异。当研究者把相同剂量的kappa opioids--一种止痛药--分发给智齿刚刚被拔除的28名男子和20名女子的时候,女性报告的她们感受的痛楚要比男性小的多,而且止痛的时间女性更长。这一研究说当需要止痛药时,应该给女性服用kappa opioids,而应该给男性服用其他的止痛药。而且,研究人员应该重新评估所有药品对于男性以及女性的效用。

men and women experience pain very differently from one another 论断:只要止痛药需要就要给女的开K,而男的应该开别的药。另外研究者应该对所有药对于男女不同的效果重新评估。因为一份新的报告显示,男女对痛的经历非常不同,医生应该考虑给他们开不同的止痛药。研究中给在拨智齿的28个男人和20个女人开同样剂量的K,女的比男的少说痛,而且药效女的也明显长一些。
        前提,K对女的有效,对男的没效。拨牙一项不能代表所有的情况。
        前提,男女对疼痛的经历不同,没有直接证据证明。
        论据,问题多。首先,人太少,没有代表性。其次,也许不是性别带来的不同,而是其他体质上的问题。没有说是否每一个女人都比每一个男人少说痛。第三,心理影响。报告的痛有心理作用,而医学关心的止痛是生理问题。不排除女的意志力坚强,所以少说痛。
        就算男女对K的经历不一样,没有理由存在需要对所有的药都重新评估。

189.The following is a recommendation from the dean at Foley College, a small liberal arts college, to the president of the college.
     "Since college-bound students are increasingly concerned about job prospects after graduation, Foley College should attempt to increase enrollment by promising to find its students jobs after they graduate. Many administrators feel that this strategy is a way for Foley to compete against larger and more prestigious schools and to encourage students to begin preparing for careers as soon as they enter college. Furthermore, a student who must choose a career path within his or her first year of college and who is guaranteed a job after graduation is more likely to successfully complete the coursework that will prepare him or her for the future."
    鉴于要读大学的学生越来越关注毕业后的就业前景,Foley学院应该尝试通过保证其学生在毕业后找到工作来增加就读人数。很多管理者感到这一策略是Foley学院与更大、历史更悠久的学校竞争,并鼓励学生在一入校就开始职业准备的途径。而且,一个在大学一年纪就必须选择一条职业道路,并且被保证毕业后有工作的学生更可能成功地完成那些为他们的未来作准备的课程。

increase enrollment by promising to find jobs after graduate. 论断:F校应该通过许诺毕业后给学生找工作来提高注册率。因为想上大学的学生越来越关心毕业后工作的前景。很多管理者都认为这个是一个和牛校竞争并鼓励学生一进大学就为工作作准备的好策略。另外,必须在大学第一年就选择就业途径以及被保证毕业后有工作的学生,会更可能成功地完成为未来作准备的功课。
        前提,学生喜欢。也没有资料证明学生愿意一进校就确定将来的工作。尽管学生越来越关心工作前景,但并没有资料证明这种关心会影响到他们对于学校的选择。
        前提,和牛校竞争。学校为学生提供什么样的教育才是最关键的,标准是学生将来的能力如何。但是不会像论者说的那样,一开始就定了工作的学生可能没有野心和积极性去学习。而且失去多种可能性的发展,禁固了学生,更不如牛校的了。
结论:能提高注册率。学校提供的工作学生可能不愿接受。

190.The following is a letter to the editor of the Glenville Gazette, a local newspaper.
    "Over the past few years, the number of people who have purchased advance tickets for the Glenville Summer Concert series has declined, indicating lack of community support. Although the weather has been unpredictable in the past few years, this cannot be the reason for the decline in advance ticket purchases, because many people attended the concerts even in bad weather. Clearly, then, the reason for the decline is the choice of music, so the organizers of the concert should feature more modern music in the future and should be sure to include music composed by Richerts, whose recordings Glenville residents purchase more often than any other contemporary recordings. This strategy will undoubtedly increase advance ticket purchases and will increase attendance at the concerts."
      过去几年中,购买了Glenville夏季音乐会的预售票的人数减少了,表明它缺乏社会支持。尽管过去几年的天气确实有些变化无常,但这并不是预售票销售减少的原因,因为很多人即使在坏天气下仍然参加音乐会。显然,销量下降的原因是音乐的选择,因此音乐会的组织者应该在未来提供更多的现代音乐,并保证演奏更多的Richerts作曲的作品,Glenville的居民购买他的唱片的数量显著多于其他的当代音乐唱片。这一策略无疑将会增加预售票的销量并增加音乐会的上座率。

advance tickets for the Concert series 论断:音乐会的组织者今后应该更多选择当代音乐并保证有R的音乐,当地人比其它的当代唱片更经常买他的,这个策略无疑会提高预售票的购买,并提高音乐会的出席率。因为不是天气不可预测影响了预售票的发售,所以下降是音乐的选择问题。过去几年系列音乐会预售票的下滑,显示社会区没有支持。
        前提,社会不支持。预售票下滑,可能出席率仍然很高。当天售票的多了。只是不愿早买票了,谁知道还有没有别的事
        前提,不是天气的原因。就算的确下滑,还有其他问题存在。首先没能有效排除天气的影响,有可能人们不愿意浪费,所以后来就不再买预售的票了。而且也有可能那几场特别重要。另外,音乐会的质量问题,演奏技术问题都有可能。
        是音乐选择的问题。没有证据关于人们不喜欢以前音乐会上的曲目。也没有人说喜欢现代音乐,R的唱片卖得好,不一定适合现场演奏,也不一定人们就还会去听。

191.The following is a letter from a professor at Xanadu College to the
college's president.
     "The development of an extensive computer-based long-distance learning program will enhance the reputation of Xanadu College. This program would allow more students to enroll in our courses, thereby increasing our income from student tuition. Traditional courses could easily be adapted for distance learners, as was shown by the adaptation of two traditional courses for our distance learning trial project last year. Also, by using computer programs and taped lectures, faculty will have fewer classroom obligations and more time to engage in extensive research, thereby enhancing the reputation of Xanadu."
    发展一套大型计算机远程教学系统将会提高Xanadu学院的声誉。这一系统将使更多学生报名参加我们的课程,从而提高我们的学费收入。传统教学可以很容易地改编成远程教学内容,这已经被去年我们远程教学实验系统的两个传统课程的改编所证实。而且,通过使用计算机程序和录音教学,员工可以减少去教室的次数并有更多的时间从事大规模的科研活动,从而提高Xanadu的声誉。

long-distance learning enhance the reputation of College. 论断:发展计算机远程教育会提高学校的声誉。因为这个课程可以让更多的学生上我们的课从而增加学费收入,另外传统课程也容易适应远程的学习者,这已经被去年试点的两个课程证实。同样,计算机和磁带讲座,老师就不用去教室,而有更多的时间去搞研究,从而提高学校声誉。
        前提,老师有更多时间作研究。还是有学生要去课堂上课,老师的这部分时间并不一定会节省下来。而且时间多了,老师也不一定作研究。
        前提,学费收入会上升。首先没有证据证明有学生来上。其次可能会有在校的转去远程,从而减少这部分收入。
        传统课程转成远程容易。首先没有证据说试点的两个课程很成功。其次,课程不一样。
        可能会带来的负作用:学有共同学习的气氛,老师和学生没有互动。搞不好学校声誉会下降。

192.The following is a letter to the editor of the Roseville Gazette.
    "Despite opposition from some residents of West Roseville, the arguments in favor of merging the townships of Roseville and West Roseville are overwhelming. First, residents in both townships are confused about which authority to contact when they need a service; for example, the police department in Roseville receives many calls from residents of West Roseville. This sort of confusion would be eliminated with the merger. Second, the savings in administrative costs would be enormous, since services would no longer be duplicated: we would have only one fire chief, one tax department, one mayor, and so on. And no jobs in city government would be lost-employees could simply be reassigned. Most importantly, the merger will undoubtedly attract business investments as it did when the townships of Hamden and North Hamden merged ten years ago."
     尽管West Roseville的一些居民持反对意见,支持合并Roseville和West Roseville的呼声还是占了上风。首先,两地区的居民经常搞不清在需要服务的时候应该和哪个政府联系;举例来说,Roseville的警署接到过很多West Roseville居民的电话。这种混乱可以通过合并来消除。其次,管理开支将会得到很大节省,因为服务不再重复:我们将只有一个消防局,一个税务局,一个市长,以及其他等等。市政府的工作岗位并不会减少,雇员可以被重新委任。最重要的是,合并无疑会吸引更多的商务投资,就像Hamden和North Hamden在十年前合并时所发生的那样。

merging the townships of Roseville and West Roseville 论断:尽管一些WR的居民反对,合并R和WR的好处还是占上风的。首先两镇的居民现在在需要服务的时候总不清楚到底该找谁,合并后就不会有这事儿了。第二,节省下来的管理开支会很庞大,因为服务不再双份。另外政府的工作也不会少,员工可以简单地重新分配。最重要的是,合并无疑会吸引商业投资,十年前H和NH合并的时候就是这样的。
        例子没有代表性。而且其他解决的办法很简单。
        开支不一定会节省。虽然工作是单份的,但会是庞大的。而且这样的后果是使机构更复杂,可能会降低工作效率。
        人员重新分配。既然工作不再双份,必然岗位减少,需要员工减少。这一条与第二条矛盾。另外就算你不矛盾,工作内容也会影响。
        例子不说明问题。时间不同,地点不同,没有可能比性。另外他们的投资吸引可能是因为经济的发展和其他措施,也许与合并没有关系。

193.The Department of Education in the state of Attra recommends that high school students be assigned homework every day. Yet a recent statewide survey of high school math and science teachers calls the usefulness of daily homework into question. In the district of Sanlee, 86 percent of the teachers reported assigning homework three to five times a week, whereas in the district of Marlee, less than 25 percent of the teachers reported assigning homework three to five times a week. Yet the students in Marlee earn better grades overall and are less likely to be required to repeat a year of school than are the students in Sanlee. Therefore, all teachers in our high schools should assign homework no more than twice a week, if at all.
     Attra州的教育部建议高中学生应该每天都被布置家庭作业。然而最近一项对于全州的数学和理化高中教师的调查却对每天布置家庭作业的作用提出了疑问。在Sanlee地区,86%的教师报告说他们每周布置3-5次家庭作业,而在Marlee地区,只有少于25%的教师报告说每周布置3-5次作业。而Marlee地区的学生综合成绩更好,而且比起Sanlee的学生更不容易留级。因此,我们高中的所有教师每周布置作业都不应该超过两次。

high school students be assigned homework every day 论断:我们的高中老师每星期布置的作业应该不超过两次,如果非得有的话。因为最近全州对数学和理科老师的调查显示,在S区86%的老师每周布置三到五次作业,M区则不到25%的老师说每周三到五次作业。但是M的学生比S的学生少留级。
        前提,留级的少教学效果就好。留级是个极端现象,不能说明教学效果,它与各校的政策有关系。有可能M在这方面的标准比较松。
        论据有问题:首先两个学校的老师和学生的能力没有考虑进来,如果说这方面的差异很大,教学效果就不是作业多少带来的。其次,数学和理科两门功课不能代表所有课程。第三,M的作业数目不确定,有可能M有很多功课是要学生长期完成的。
•结论:各校情况都不一样。不超过两次的概念是怎么来的,不一定就好。

194.A recent study suggests that people who are left-handed are more likely to succeed in business than are right-handed people. Researchers studied photographs of 1,000 prominent business executives and found that 21 percent of these executives wrote with their left hand. So the percentage of prominent business executives who are left-handed (21 percent) is almost twice the percentage of people in the general population who are left-handed (11 percent). Thus, people who are left-handed would be well advised to pursue a career in business, whereas people who are right-handed would be well advised to imitate the business practices exhibited by left-handers.
     最近一项研究发现左撇子比右撇子更可能在商业活动中取得成功。研究者研究了1000名著名商业经理并发现这些经理有21%的人用左手写字。因而左撇子的著名商业经理的比例(21%)几乎是总体人群中左撇子比例(11%)的两倍。因此,左撇子应该被建议寻求商业方面的职业,而右撇子应该被建议模仿左撇子的商业行为。

left-handed more likely to succeed than are right-handed people 论断:左捌子应该去从商,右捌子模仿左捌子的商业经历。因为最近的研究显示左捌子比右捌子更可能从商成功。研究者通过照片对1000名商业巨头作分析,发现21%的人用左手写字。这个比例几乎是普通人中间左捌子的两倍。
        左捌子从商成功率高。虽然商业成功人士中的左捌子比例比一般人高,但是从商的人中间有多少是左捌子?如果从商的人中有一半是左捌子,那么右捌子显然比左捌子成功率更高。
        右捌子要学左捌子。没有资料说左捌子比右捌子更成功,为什么要他们学。
        论据有问题,且不论医学依据是什么,照片有可能摆拍呀。另外混淆左捌子与用左手写字的人,有些人两只手都能写字,有些人用左手写字,但别的事都主要是右手。

195.The following is a letter from an editor at Liber Publishing Company to the company's president.
    "In recent years, Liber has unfortunately moved away from its original mission: to publish the works of regional small-town authors instead of those of big-city authors. Just last year, 90 percent of the novels we published were written by authors who maintain a residence in a big city. Although this change must have been intended to increase profits, it has obviously backfired, because Liber is now in serious financial trouble. The only way to address this problem is to return to our original mission. If we return to publishing only the works of regional small-town authors, our financial troubles will soon be resolved."
       近年来,Liber很不幸地偏离了其原始使命:出版地区性小城镇作家的作品而不是那些大城市作家的作品。就在去年,我们出版的小说中有90%都是由那些在大城市拥有一处住所的作家写的。尽管这一改变的目的一定是为了增加利润,但它显然事与愿违,因为Liber现在正处于严重的财政问题之中。解决这一问题的唯一途径就是回到我们的原始使命。如果我们回复到只出版地区性小城镇作家的作品,我们的财政问题将很快得到解决。

publish small-town authors instead of those of big-city 论断:恢复只出本地小镇作者的书财政问题很快就会解决。因为去年90%的小说都是大城市的作者的,我们现在处于严重的财财政麻烦当中。
        前提,我们现在的财政麻烦是因为去年90%的小大城市小说作者的书造成。没有资料证明这些书卖得不好。成本上升,书市不好都可能。
        前提,工作重心转移的问题。论据里的问题:小说是否是出版社的大头。如果95%的书都不是小说,这90%就不能说明我们的工作重心发生了转移。
        被忽略的前提:是否小镇上的作者出书量还有那么多。
结论:没有证据证明只出小镇的作者书就会解决财政问题。没说有人愿意看呀。而且拒绝大城市作家的书,这会带来问题。

使用道具 举报

Rank: 2

声望
9
寄托币
139
注册时间
2009-4-2
精华
0
帖子
28
21
发表于 2009-4-26 22:19:43 |只看该作者
180.The following is a recommendation from the personnel director to the president of Acme Publishing Company.
     "Many other companies have recently stated that having their employees take the Easy Read Speed-Reading Course has greatly improved productivity. One graduate of the course was able to read a five-hundred-page report in only two hours; another graduate rose from an assistant manager to vice president of the company in under a year. Obviously, the faster you can read, the more information you can absorb in a single workday. Moreover, Easy Read costs only $500 per employee-a small price to pay when you consider the benefits to Acme. Included in this fee is a three-week seminar in Spruce City and a lifelong subscription to the Easy Read newsletter. Clearly, Acme would benefit greatly by requiring all of our employees to take the Easy Read course."
     很多其他公司最近指出他们的员工参加了Easy Read的速读课程之后生产效率显著提高。这个课程的一名毕业生能够在两个小时之内读完长达500页的报告,另一名毕业生在一年内从助理经理上升到了副总裁。显然,你的阅读速度越快,在一天之内所能吸收的信息就越多。而且,Easy Read的学费只有每人500元,当考虑到它给Acme带来的效益的时候这就是个小数目。这个费用包括在Spruce City举行的为期三周的研讨会和Easy Read新闻刊物的终生赠阅。显然,Acme通过要求所有员工参加Easy Read的课程将会极大地受益。

Easy Read Speed-Reading Course, Publishing Company 论断:A让所有员工参加快速阅读课程会收获很大。因为很多其它工司说他们参加这个课程的员工生产力提高很大,这个课程的一个毕业生两个小时里能五百页的报告,另一个毕业生在一年之内就从经理助理升到了副总裁。显然,读得越快,你一个工作日里获得的信息就越多。另外这个课程花费只有五百元,这与从其受到的利比是非常低的。它包括在云杉城三个星期的课程和一生的快速阅读新闻信。
        前提收获大,证据有问题。首先,两个人的例子不说明问题,这个课程的大多数学员怎么样?第二五百页那人,看的是什么内容,又记住了多少。第三当副总的那人,他升职的原因可能跟阅读完全没有关系。
        前提,对我们是否有用。公司不同,员工不同,工作内容不同。如果我们的员工阅读已经很好了,就不用了吧。
结论:有用也不用所有人都去呀,还有一些岗位根本用不着快速阅读。而且还得要员工都带薪放三个星期的假。

181.From a letter to the editor of a city newspaper.
     "One recent research study has indicated that many adolescents need more sleep than they are getting, and another study has shown that many high school students in our city are actually dissatisfied with their own academic performance. As a way of combating these problems, the high schools in our city should begin classes at 8:30 A.M. instead of 7:30 A.M., and end the school day an hour later. This arrangement will give students an extra hour of sleep in the morning, thereby making them more alert and more productive. Consequently, the students will perform better on tests and other assignments, and their academic skills will improve significantly."
     最近一项研究表明,很多青少年需要更多的睡眠,另外一项研究表明我们市很多高中学生对于自己的学习成绩不满意。作为解决这些问题的途径,我市的高中应该在早上8:30开始上课,而不是7:30,并且推迟一小时放学。这种安排将允许学生在早上多睡一个小时,从而使他们更加清醒和高效。因此,学生在测验和其他作业中将表现得更好,他们的学习水平将会显著提高。

adolescents need more sleep 论断:高中早上课应该推迟一个小时开始,下午推后一小时下课。这个安排能让学生早上多一个小时睡眠,从而使他们更有效率更精神。学生的测验和作业表现会提高,学术能力也会有很大进步。因为最近的研究显示很多青少年需要比现在更多的睡眠,而且另一项研究也显示我们城市的高中学生对自己的学校表现不满意。
        被忽略的前提,是否睡得多学校表现就会好,并没有这样的证据。而关于不满意自己的学生,也没说是因为自己睡得少,他们可能是对自己有高要求,要不就是觉得自己挺懒的。还有就是觉得老师不好。
        前提,这样做学生睡眠是否会加长。早上迟一小,可能多了一小时睡眠,但学生可能晚上晚睡一小时。这样睡眠时间是一样的。而且还有看电视。
结论:睡眠问题解决了,不一定就会学术能力提高。

182.Butter has now been replaced by margarine in Happy Pancake House restaurants throughout the southwestern United States. Only about 2 percent of customers have complained, indicating that 98 people out of 100 are happy with the change. Furthermore, many servers have reported that a number of customers who still ask for butter do not complain when they are given margarine instead. Clearly, either these customers cannot distinguish margarine from butter, or they use the term "butter" to refer to either butter or margarine. Thus, to avoid the expense of purchasing butter, the Happy Pancake House should extend this cost-saving change to its restaurants in the southeast and northeast as well.
    Happy Pancake House在美国西南部的餐厅用人造黄油代替了天然黄油。只有大约2%的顾客曾经投诉,这说明100个人中有98人对于这种替换是乐于接受的。而且,很多服务生报告说很多仍然要了天然黄油的顾客在被给予人造黄油的时候并没有投诉。显然,这些顾客要么分不清天然黄油和人造黄油,要么是用"天然黄油"这个词汇来指天然黄油或人造黄油。因此,为避免购买天然黄油的花费,Happy
Pancake House应该把这项节省措施推广到在东南部以及东北部的餐厅。

Butter has now been replaced by margarine 论断:H饭店应该在东南部地区和东北部地区也用人造黄油换成黄油这项省钱的改变。因为在美国西南部地区H城的这种改变,只引起了2%的顾客抱怨,这显示98%的人对这种变化挺高兴。另外很多服务商报告说很多仍然说要黄油的顾客,当他们得到的是人造黄油时也没有说什么。很清楚,要么是他们分辨不出来,要么就是他们用黄油这个名称称呼这两种产品。
        前提,顾客没意见。2%的顾客抱怨,不代表98%的就高兴。有人不在乎,有人在乎但不说,但下次会不再光顾。
        证据,要黄油的顾客拿到人造的不说什么,不表示他们分辨不出来,没资料显示顾客看不出来,或是不在乎。他们怕麻烦,或是不得己。但下次还是不去了。
结论:各地不一样,有些地方的人特别在乎人造的问题。还有地方不一样,货的成本可能也会有区别。

183.Many employees of major United States corporations are fearful that they will lose their jobs in the near future, but this fear is largely unfounded. According to a recent study, a majority of companies expected to make new hires in the coming year, while fewer companies expected to lay off employees. In addition, although it is very disturbing to be laid off, the proliferation of programs and of workshops designed to improve job-finding skills has made being laid off far less painful than it once was.
      很多美国大公司的员工担心他们在不久的将来可能失业,但这种担心是没有依据的。根据最近的一项研究,大部分公司准备在来年雇佣新员工,而更少的公司准备裁员。而且,尽管下岗确实很麻烦,但一些帮助人们提高求职技巧的培训项目的增加使下岗远没有以前那么痛苦。

employees are fearful lose their jobs in the near future 论断:害怕失去工作是没有根据的。因为根据一份最近的研究,大多数的公司准备在明年增加新雇佣员工,而更少的公司司准备裁员。另外尽管被裁员很让人烦恼,提高找工作的技巧的课程的增加也使失去工作没以前痛苦。根据一份调查,调查问题多多。
        大多数公司准备新雇人,而小部分公司有裁员计划。新雇人不证明不开除人,事实上是对老员工的一种潜在开除的危险。公司得控制一定规模。另外明年有计划招人,也不证明未来几年不裁人。小部分公司裁员人数可能很多,它们可能都是大公司,一裁好几千,而小公司招人才几十而己。
        调查的人说了实话。首先没有科学描述,其次准备裁员即意味着效益不好,有depression发生,公司不会愿意透露。
        课程的效果,效果不好找工作还是很费劲。而且在两项工作之间的日子会有多长,也还是让人disturbing.

184.In the Bayhead Public Library, books that are rarely borrowed continue to take up shelf space year after year, while people who want to read a recent novel frequently find that the library's only copy is checked out. Clearly, the library's plan to replace books that are borrowed no more than once a year with sufficient copies of more recent books will solve this problem. The protest we have heard since this plan was made public has come from a small, and thus unrepresentative, group of some thirty people and so should therefore be ignored.
      在Bayhead公共图书馆,那些很少被借阅的图书仍然长期占据着书库空间,而想要读最新小说的读者往往发现图书馆唯一的一本已经被借出了。显然,图书馆用充足的最新书籍来代替一年中被借阅不超过一次的书籍的计划将解决这一问题。这项计划公布后我们所听到的抗议来自一个30来人的很小的,因而也不具有代表性的团体,所以可以被忽略。

Public Library, books rarely borrowed take up shelf space year after year 论断:图书馆准备把每年借出次数不多的书换成多几份较新的书能够解决,有些书占书架,有些人们想借的书不够的问题。我们听到的反对意见是,因为这项计划已对公众宣布,来自一个小的没有代表性的30多个人的队伍,所以可以被忽略。
        前提,能解决问题,首先新书迟早也成旧书,如果因为旧而被借的少,那么这些书将来也会借的人少。其次,常被借的书以及所谓新书有可能多是期刊,流行畅销书,很快就会失去意义,而很多旧书虽然一年里借出的次数不多,但每年都会有人借,包括以后。
        会带来的问题,这样做的结果是图书馆的藏数种类减少。因为图书馆的空间是有限的。
        一部分人的意见也不能忽略,虽然对外公布也有很多人可能不知道,另外很多人采用沉默表达反抗,30个人已经很多了。或许他们还很有权威。

185.The following appeared in a letter from the owner of the Sunnyside Towers apartment building to its manager.
     "One month ago, all the showerheads on the first five floors of Sunnyside Towers were modified to restrict the water flow to approximately 1/3 of its original force. Although actual readings of water usage before and after the adjustment are not yet available, the change will obviously result in a considerable savings for Sunnyside Corporation, since the corporation must pay for water each month. Except for a few complaints about low water pressure, no problems with showers have been reported since the adjustment. Clearly, restricting water flow throughout all the 20 floors of Sunnyside Towers will increase our profits further."
     一个月之前,Sunnyside塔楼最低的五层的所有淋浴喷头被调节成水压只有以前的大约三分之一。尽管在调节之后用水量的确切读数还没有出来,但这种变革显然将为Sunnyside公司节省大量的花费,因为公司必须每月为所用的水付费。除了关于低水压的几起投诉,在调节之后没有发生关于淋浴喷头的问题的报告。显然,在Sunnyside塔楼所有的20层都限制水压将会增加我们未来的利润。

showerheads 论断:对20层的S塔楼限制水流,能够进一步提高我们的利润。因为这样可以为公司节省很多钱。一个月前,下面五层楼的淋浴喷头被调整,限制了三分之一的水流量。虽然调整后具体的用水指数还不知道,这个变化显然会为公司节约大量的钱。除了一些关于水压的抱怨外,没有别的关于喷头的抱怨。
        前提,是否节水了。既然还不知道具体的指数就不排除用水量上升的可能。因为水流变小,用水时间就会加长。另外由于水压的冲洗作用没了,这种时间上的补偿很有可能会特别长。
        前提,没人抱怨。首先刚一个月就有了抱怨。其次楼层越高对水压要求越到。第三楼上的人可能特殊,他们用水特别多,特别喜欢大流量的洗澡。时间长必然引起楼主不高兴。
结论:利润,收入得靠卖房,有可能是租房,水费只是其中很小一部分成本。

186.The following is a recommendation from the director of personnel to the president of Professional Printing Company.
      "In a recent telephone survey of automobile factory workers, older employees were less likely to report that having a supervisor present increases their productivity. Among workers aged 18 to 29, 27 percent said that they are more productive in the presence of their immediate supervisor, compared to 12 percent for those aged 30 or over, and only 8 percent for those aged 50 or over. Clearly, if our printing company hires mainly older employees, we will increase productivity and save money because of the reduced need for supervisors."
      在最近一次对汽车工厂工人的电话调查中,年纪大一些的员工更少报告说有管理员在场会提高他们的生产效率。在18到29岁的员工中,27%的人说当他们的顶头上司在场时更有效率,相比之下,30岁及以上的工人只有12%,50岁及以上的工人只有8%这样认为。显然,如果我们印刷公司主要雇佣年级大一些的员工的话,我们的劳动生产率将会提高,并且节省开支,因为对于管理员的需求将会减少。

older employees were less likely having a supervisor 论断:我们印刷公司主要雇用老员工的话,将提高生产率并省下雇指导员的钱。因为一份对汽车工人的电话调查里,老的员工比较少报告当指导员在现场时会提高他们的生产力。
        前提,省钱。老工人一般要求的工资都比年轻人高,指导员的钱省下来了,工资却得长。
        前提,提高生产力。没有证据证明年轻人的生产力比老工人的低,不管在不在场的问题。而且年轻人一般体力好,少得病,少乱七八糟的事。
        论据,太弱。首先,电话调查查的范围,各有多少人,是否准确。其次,调查里没指导员时年轻人的生产力还有可能比老工人高,尽管有人在场时会更高。
结论:两家公司工作不一样,员工要求不一样,没有可比性。

187.The following appeared as part of an article in a health magazine.
    "A new discovery warrants a drastic change in the diets of people living in the United States. Two scientists have recently suggested that omega -3 fatty acids (found in some fish and fish oils) play a key role in mental health. Our ancestors, who ate less saturated fat and more polyunsaturated fat, including omega -3 fatty acids, were much less likely to suffer from depression than we are today. Moreover, modern societies-such as those in Japan and Taiwan-that consume large quantities of fish report depression rates lower than that in the United States. Given this link between omega
-3 fatty acids and depression, it is important for all people in the United States to increase their consumption of fish in order to prevent depression."
     一项新发现表明生活在美国的居民饮食将会发生显著变化。两名学者最近指出omega-3脂肪酸(它在一些鱼类以及鱼油中被发现)在精神健康方面有很重要的作用。我们的祖先食用更少的饱和脂肪和更多的多重不饱和脂肪,包括omega-3脂肪酸,他们患抑郁症的可能性比我们现在要低很多。而且,食用大量鱼类的现代社会--比如象日本和台湾那样的社会--抑郁症的上报率比在美国要低。基于以上omega-3脂肪酸和抑郁症的联系,让所有的美国人增加鱼类的食用量以防止抑郁症是很重要的。

omega -3 fatty acids (found in some fish and fish oils) 论断:对所有美国人来说更多吃鱼来防止消沉很重要。因为两个科学家最近说O酸这种在一些鱼和鱼油里发现的东西对心理健康很有影响。我们的先人就少吃饱和脂肪多吃不饱和脂肪,其中就包括O酸,他们就比我们今天少消沉。另外,一些现代社会比如大量食用鱼的日本和台湾,他们报告消沉的论就比美国少。
        前提,吃鱼防止消沉。没有证据,两个科学家是说在某些鱼里发现的O酸,不是所有的鱼里都含有O酸。概念偷换。
        证据,科学家的证据值得怀疑:先人与我们社会不同、经济状况不同、生活节奏不同,人际关系复杂度不同,当然不能比。其次:台湾和日本,那等小国情况相对美国可能会简单一些。也没有资料证明他们少消沉就是因为吃了鱼。其它情况问题,比如经济状况,社会问题等等。
结论:干嘛所有人呀,有人对鱼敏感呢,还有些人吃得也太多了吧。

188.A new report suggests that men and women experience pain very differently from one another, and that doctors should consider these differences when prescribing pain medications. When researchers administered the same dosage of kappa opioids-a painkiller-to 28 men and 20 women who were having their wisdom teeth extracted, the women reported feeling much less pain than the men, and the easing of pain lasted considerably longer in women. This research suggests that kappa opioids should be prescribed for women whenever pain medication is required, whereas men should be given other kinds of pain medication. In addition, researchers should reevaluate the effects of all medications on men versus women.
      一项新报告表明,男性和女性对于疼痛的感受是有显著差异的,医生在开止痛药方的时候应该考虑到这种差异。当研究者把相同剂量的kappa opioids--一种止痛药--分发给智齿刚刚被拔除的28名男子和20名女子的时候,女性报告的她们感受的痛楚要比男性小的多,而且止痛的时间女性更长。这一研究说当需要止痛药时,应该给女性服用kappa opioids,而应该给男性服用其他的止痛药。而且,研究人员应该重新评估所有药品对于男性以及女性的效用。

men and women experience pain very differently from one another 论断:只要止痛药需要就要给女的开K,而男的应该开别的药。另外研究者应该对所有药对于男女不同的效果重新评估。因为一份新的报告显示,男女对痛的经历非常不同,医生应该考虑给他们开不同的止痛药。研究中给在拨智齿的28个男人和20个女人开同样剂量的K,女的比男的少说痛,而且药效女的也明显长一些。
        前提,K对女的有效,对男的没效。拨牙一项不能代表所有的情况。
        前提,男女对疼痛的经历不同,没有直接证据证明。
        论据,问题多。首先,人太少,没有代表性。其次,也许不是性别带来的不同,而是其他体质上的问题。没有说是否每一个女人都比每一个男人少说痛。第三,心理影响。报告的痛有心理作用,而医学关心的止痛是生理问题。不排除女的意志力坚强,所以少说痛。
        就算男女对K的经历不一样,没有理由存在需要对所有的药都重新评估。

189.The following is a recommendation from the dean at Foley College, a small liberal arts college, to the president of the college.
     "Since college-bound students are increasingly concerned about job prospects after graduation, Foley College should attempt to increase enrollment by promising to find its students jobs after they graduate. Many administrators feel that this strategy is a way for Foley to compete against larger and more prestigious schools and to encourage students to begin preparing for careers as soon as they enter college. Furthermore, a student who must choose a career path within his or her first year of college and who is guaranteed a job after graduation is more likely to successfully complete the coursework that will prepare him or her for the future."
    鉴于要读大学的学生越来越关注毕业后的就业前景,Foley学院应该尝试通过保证其学生在毕业后找到工作来增加就读人数。很多管理者感到这一策略是Foley学院与更大、历史更悠久的学校竞争,并鼓励学生在一入校就开始职业准备的途径。而且,一个在大学一年纪就必须选择一条职业道路,并且被保证毕业后有工作的学生更可能成功地完成那些为他们的未来作准备的课程。

increase enrollment by promising to find jobs after graduate. 论断:F校应该通过许诺毕业后给学生找工作来提高注册率。因为想上大学的学生越来越关心毕业后工作的前景。很多管理者都认为这个是一个和牛校竞争并鼓励学生一进大学就为工作作准备的好策略。另外,必须在大学第一年就选择就业途径以及被保证毕业后有工作的学生,会更可能成功地完成为未来作准备的功课。
        前提,学生喜欢。也没有资料证明学生愿意一进校就确定将来的工作。尽管学生越来越关心工作前景,但并没有资料证明这种关心会影响到他们对于学校的选择。
        前提,和牛校竞争。学校为学生提供什么样的教育才是最关键的,标准是学生将来的能力如何。但是不会像论者说的那样,一开始就定了工作的学生可能没有野心和积极性去学习。而且失去多种可能性的发展,禁固了学生,更不如牛校的了。
结论:能提高注册率。学校提供的工作学生可能不愿接受。

190.The following is a letter to the editor of the Glenville Gazette, a local newspaper.
    "Over the past few years, the number of people who have purchased advance tickets for the Glenville Summer Concert series has declined, indicating lack of community support. Although the weather has been unpredictable in the past few years, this cannot be the reason for the decline in advance ticket purchases, because many people attended the concerts even in bad weather. Clearly, then, the reason for the decline is the choice of music, so the organizers of the concert should feature more modern music in the future and should be sure to include music composed by Richerts, whose recordings Glenville residents purchase more often than any other contemporary recordings. This strategy will undoubtedly increase advance ticket purchases and will increase attendance at the concerts."
      过去几年中,购买了Glenville夏季音乐会的预售票的人数减少了,表明它缺乏社会支持。尽管过去几年的天气确实有些变化无常,但这并不是预售票销售减少的原因,因为很多人即使在坏天气下仍然参加音乐会。显然,销量下降的原因是音乐的选择,因此音乐会的组织者应该在未来提供更多的现代音乐,并保证演奏更多的Richerts作曲的作品,Glenville的居民购买他的唱片的数量显著多于其他的当代音乐唱片。这一策略无疑将会增加预售票的销量并增加音乐会的上座率。

advance tickets for the Concert series 论断:音乐会的组织者今后应该更多选择当代音乐并保证有R的音乐,当地人比其它的当代唱片更经常买他的,这个策略无疑会提高预售票的购买,并提高音乐会的出席率。因为不是天气不可预测影响了预售票的发售,所以下降是音乐的选择问题。过去几年系列音乐会预售票的下滑,显示社会区没有支持。
        前提,社会不支持。预售票下滑,可能出席率仍然很高。当天售票的多了。只是不愿早买票了,谁知道还有没有别的事
        前提,不是天气的原因。就算的确下滑,还有其他问题存在。首先没能有效排除天气的影响,有可能人们不愿意浪费,所以后来就不再买预售的票了。而且也有可能那几场特别重要。另外,音乐会的质量问题,演奏技术问题都有可能。
        是音乐选择的问题。没有证据关于人们不喜欢以前音乐会上的曲目。也没有人说喜欢现代音乐,R的唱片卖得好,不一定适合现场演奏,也不一定人们就还会去听。

191.The following is a letter from a professor at Xanadu College to the
college's president.
     "The development of an extensive computer-based long-distance learning program will enhance the reputation of Xanadu College. This program would allow more students to enroll in our courses, thereby increasing our income from student tuition. Traditional courses could easily be adapted for distance learners, as was shown by the adaptation of two traditional courses for our distance learning trial project last year. Also, by using computer programs and taped lectures, faculty will have fewer classroom obligations and more time to engage in extensive research, thereby enhancing the reputation of Xanadu."
    发展一套大型计算机远程教学系统将会提高Xanadu学院的声誉。这一系统将使更多学生报名参加我们的课程,从而提高我们的学费收入。传统教学可以很容易地改编成远程教学内容,这已经被去年我们远程教学实验系统的两个传统课程的改编所证实。而且,通过使用计算机程序和录音教学,员工可以减少去教室的次数并有更多的时间从事大规模的科研活动,从而提高Xanadu的声誉。

long-distance learning enhance the reputation of College. 论断:发展计算机远程教育会提高学校的声誉。因为这个课程可以让更多的学生上我们的课从而增加学费收入,另外传统课程也容易适应远程的学习者,这已经被去年试点的两个课程证实。同样,计算机和磁带讲座,老师就不用去教室,而有更多的时间去搞研究,从而提高学校声誉。
        前提,老师有更多时间作研究。还是有学生要去课堂上课,老师的这部分时间并不一定会节省下来。而且时间多了,老师也不一定作研究。
        前提,学费收入会上升。首先没有证据证明有学生来上。其次可能会有在校的转去远程,从而减少这部分收入。
        传统课程转成远程容易。首先没有证据说试点的两个课程很成功。其次,课程不一样。
        可能会带来的负作用:学有共同学习的气氛,老师和学生没有互动。搞不好学校声誉会下降。

192.The following is a letter to the editor of the Roseville Gazette.
    "Despite opposition from some residents of West Roseville, the arguments in favor of merging the townships of Roseville and West Roseville are overwhelming. First, residents in both townships are confused about which authority to contact when they need a service; for example, the police department in Roseville receives many calls from residents of West Roseville. This sort of confusion would be eliminated with the merger. Second, the savings in administrative costs would be enormous, since services would no longer be duplicated: we would have only one fire chief, one tax department, one mayor, and so on. And no jobs in city government would be lost-employees could simply be reassigned. Most importantly, the merger will undoubtedly attract business investments as it did when the townships of Hamden and North Hamden merged ten years ago."
     尽管West Roseville的一些居民持反对意见,支持合并Roseville和West Roseville的呼声还是占了上风。首先,两地区的居民经常搞不清在需要服务的时候应该和哪个政府联系;举例来说,Roseville的警署接到过很多West Roseville居民的电话。这种混乱可以通过合并来消除。其次,管理开支将会得到很大节省,因为服务不再重复:我们将只有一个消防局,一个税务局,一个市长,以及其他等等。市政府的工作岗位并不会减少,雇员可以被重新委任。最重要的是,合并无疑会吸引更多的商务投资,就像Hamden和North Hamden在十年前合并时所发生的那样。

merging the townships of Roseville and West Roseville 论断:尽管一些WR的居民反对,合并R和WR的好处还是占上风的。首先两镇的居民现在在需要服务的时候总不清楚到底该找谁,合并后就不会有这事儿了。第二,节省下来的管理开支会很庞大,因为服务不再双份。另外政府的工作也不会少,员工可以简单地重新分配。最重要的是,合并无疑会吸引商业投资,十年前H和NH合并的时候就是这样的。
        例子没有代表性。而且其他解决的办法很简单。
        开支不一定会节省。虽然工作是单份的,但会是庞大的。而且这样的后果是使机构更复杂,可能会降低工作效率。
        人员重新分配。既然工作不再双份,必然岗位减少,需要员工减少。这一条与第二条矛盾。另外就算你不矛盾,工作内容也会影响。
        例子不说明问题。时间不同,地点不同,没有可能比性。另外他们的投资吸引可能是因为经济的发展和其他措施,也许与合并没有关系。

193.The Department of Education in the state of Attra recommends that high school students be assigned homework every day. Yet a recent statewide survey of high school math and science teachers calls the usefulness of daily homework into question. In the district of Sanlee, 86 percent of the teachers reported assigning homework three to five times a week, whereas in the district of Marlee, less than 25 percent of the teachers reported assigning homework three to five times a week. Yet the students in Marlee earn better grades overall and are less likely to be required to repeat a year of school than are the students in Sanlee. Therefore, all teachers in our high schools should assign homework no more than twice a week, if at all.
     Attra州的教育部建议高中学生应该每天都被布置家庭作业。然而最近一项对于全州的数学和理化高中教师的调查却对每天布置家庭作业的作用提出了疑问。在Sanlee地区,86%的教师报告说他们每周布置3-5次家庭作业,而在Marlee地区,只有少于25%的教师报告说每周布置3-5次作业。而Marlee地区的学生综合成绩更好,而且比起Sanlee的学生更不容易留级。因此,我们高中的所有教师每周布置作业都不应该超过两次。

high school students be assigned homework every day 论断:我们的高中老师每星期布置的作业应该不超过两次,如果非得有的话。因为最近全州对数学和理科老师的调查显示,在S区86%的老师每周布置三到五次作业,M区则不到25%的老师说每周三到五次作业。但是M的学生比S的学生少留级。
        前提,留级的少教学效果就好。留级是个极端现象,不能说明教学效果,它与各校的政策有关系。有可能M在这方面的标准比较松。
        论据有问题:首先两个学校的老师和学生的能力没有考虑进来,如果说这方面的差异很大,教学效果就不是作业多少带来的。其次,数学和理科两门功课不能代表所有课程。第三,M的作业数目不确定,有可能M有很多功课是要学生长期完成的。
•结论:各校情况都不一样。不超过两次的概念是怎么来的,不一定就好。

194.A recent study suggests that people who are left-handed are more likely to succeed in business than are right-handed people. Researchers studied photographs of 1,000 prominent business executives and found that 21 percent of these executives wrote with their left hand. So the percentage of prominent business executives who are left-handed (21 percent) is almost twice the percentage of people in the general population who are left-handed (11 percent). Thus, people who are left-handed would be well advised to pursue a career in business, whereas people who are right-handed would be well advised to imitate the business practices exhibited by left-handers.
     最近一项研究发现左撇子比右撇子更可能在商业活动中取得成功。研究者研究了1000名著名商业经理并发现这些经理有21%的人用左手写字。因而左撇子的著名商业经理的比例(21%)几乎是总体人群中左撇子比例(11%)的两倍。因此,左撇子应该被建议寻求商业方面的职业,而右撇子应该被建议模仿左撇子的商业行为。

left-handed more likely to succeed than are right-handed people 论断:左捌子应该去从商,右捌子模仿左捌子的商业经历。因为最近的研究显示左捌子比右捌子更可能从商成功。研究者通过照片对1000名商业巨头作分析,发现21%的人用左手写字。这个比例几乎是普通人中间左捌子的两倍。
        左捌子从商成功率高。虽然商业成功人士中的左捌子比例比一般人高,但是从商的人中间有多少是左捌子?如果从商的人中有一半是左捌子,那么右捌子显然比左捌子成功率更高。
        右捌子要学左捌子。没有资料说左捌子比右捌子更成功,为什么要他们学。
        论据有问题,且不论医学依据是什么,照片有可能摆拍呀。另外混淆左捌子与用左手写字的人,有些人两只手都能写字,有些人用左手写字,但别的事都主要是右手。

195.The following is a letter from an editor at Liber Publishing Company to the company's president.
    "In recent years, Liber has unfortunately moved away from its original mission: to publish the works of regional small-town authors instead of those of big-city authors. Just last year, 90 percent of the novels we published were written by authors who maintain a residence in a big city. Although this change must have been intended to increase profits, it has obviously backfired, because Liber is now in serious financial trouble. The only way to address this problem is to return to our original mission. If we return to publishing only the works of regional small-town authors, our financial troubles will soon be resolved."
       近年来,Liber很不幸地偏离了其原始使命:出版地区性小城镇作家的作品而不是那些大城市作家的作品。就在去年,我们出版的小说中有90%都是由那些在大城市拥有一处住所的作家写的。尽管这一改变的目的一定是为了增加利润,但它显然事与愿违,因为Liber现在正处于严重的财政问题之中。解决这一问题的唯一途径就是回到我们的原始使命。如果我们回复到只出版地区性小城镇作家的作品,我们的财政问题将很快得到解决。

publish small-town authors instead of those of big-city 论断:恢复只出本地小镇作者的书财政问题很快就会解决。因为去年90%的小说都是大城市的作者的,我们现在处于严重的财财政麻烦当中。
        前提,我们现在的财政麻烦是因为去年90%的小大城市小说作者的书造成。没有资料证明这些书卖得不好。成本上升,书市不好都可能。
        前提,工作重心转移的问题。论据里的问题:小说是否是出版社的大头。如果95%的书都不是小说,这90%就不能说明我们的工作重心发生了转移。
        被忽略的前提:是否小镇上的作者出书量还有那么多。
结论:没有证据证明只出小镇的作者书就会解决财政问题。没说有人愿意看呀。而且拒绝大城市作家的书,这会带来问题。

使用道具 举报

Rank: 2

声望
9
寄托币
139
注册时间
2009-4-2
精华
0
帖子
28
22
发表于 2009-4-26 22:22:00 |只看该作者
196.Sadly, widespread negative images of businesspeople have been created in large part by television. Consider the fact that, although they make up a mere 10 percent of the characters in dramatic roles on television, businesspeople are responsible for about one-fifth of all the crime on television shows. In fact, in a recent survey of television producers, only 35 percent of the television roles for businesspeople were viewed as positive ones.
     很不幸的,商业人士广为人知的负面形象很大程度上是由电视造成的。请考虑如下事实:尽管商业人士在电视剧角色中只占10%,他们却占了电视节目中所有犯罪的大约五分之一。实际上,在最近一次对于电视节目制作者的调查中,电视角色中的商业人士只有35%被看作是正面人物。

widespread negative images of businesspeople created by television. 论断:商人的负面印象主要是电视造成的。因为尽管电视剧里有只有10%的角色是商人,但是电视里的罪犯大约五分之一是商人。事实上,最近的一份对电视制作人的调查中,只有35%的电视商人角色被看成是有积极意义的。
        没有观众的资料,他们没说自己对于商人的负面印象是从电视上来的。事实上还有很多的途径。
        电视里的商人也不都是负面的。首先罪犯问题,五分一是商人,那么罪犯的比例是多少?如果只有1%的角色是罪犯,那么只有千分二的角色是商人罪犯,那么也就是99%的商人角色都不是罪犯。另外,关于制作人的调查,首先描述不科学,不知是否准确,其次35%是积极的,并不代表剩下的65%就是负面的。

197.The following appeared as part of a recommendation made by a faculty member to the president of a large university.
     "Never once in our 150-year history as a university have we clarified our objectives. How, then, can we hope to adapt as an institution to the new challenges facing higher education. As a first step in this evolutionary process, therefore, we should send out questionnaires asking faculty members why they teach, asking students what they want from this university, and asking former students what they gained from their own education here. When the replies come in, we can tabulate them and formulate an official statement of our educational mission. This will surely result in improved programs at our university."
    在我们学校150年的历史中,没有一次明确过我们的目标。这样的话我们如何能够成为一个面对高等教育新挑战的学校呢?因此作为进行改革的第一步,我们应该发出调查问卷,问问员工为何从教,问问学生他们想从学校得到什么,并且问问以前的学生他们从本校的教育得到了什么。当我们得到反馈后,我们就可以作出统计并形成教育任务的官方声明。这一定会带来本校教学的提高。

Never once our university have we clarified our objectives 论断:我们应该发送问题调查,因为这项调查肯定能提高我们学校的课程。因为我们150年来都不知道自己的目标是什么。
        有了目标就能提高水平。
        发了调查就能确定下目标。
        学生、老师和校友的目标合适。

198.There is a general idea that a translation always fails to preserve some of the qualities that distinguish the original work-i.e., that 'something always gets lost in translation.' Writers, critics, and the general reading public unthinkingly accept this clichè. But this belief is unwarranted: translators are sometimes distinguished authors themselves, and some authors may even translate their own works. As the translator pointed out in the preface to an English version of Dante's works, the violin and the piano make different sounds, but they can play what is recognizably the same piece of music.
    一种普遍观点认为翻译作品总是不能保留某些原作独特的特征--即"译作总会丢掉一些东西"。作家,评论家和普通读者不加思考地接受了这种陈词滥调。但这种观点是没有依据的:有时翻译者本人就是独特的作者,有些作家甚至会翻译自己的作品。就像译者在Dante作品的英译版前言中所指出的那样,小提琴和钢琴发出不同的声音,但它们都能演奏同样为人所知的音乐作品。

translation fails to preserve the qualities that distinguish 论断:认为翻译失去原作精华的观念是没有根据的。因为翻译家有时本身就是出色的作家,而一些作家也会翻译自己的作品。D的英文版的翻译者在前言里说,小提琴和钢琴有不同的声音,但它们能演奏感觉一样的音乐作品。这些例子都有问题。
        翻译家本身是出色的作家。是作家,但不能表示它能很好地理解原著里的一些细微之处。这是生活环境、观念、文化所造成的。
        作家自己翻译。用一个自己不是很熟悉的语言,不一定能把内心中的东西表达出来。没有资料表示这些作家对于自己的翻译很满意。
        小提琴和钢琴的类比不合适。他们照着同一个乐谱演奏,而翻译却是在两种文字中转换。而且,小提琴和钢琴演奏出来的韵味肯定不一样。

199.There is a general idea that waiters and waitresses are more likely to receive larger gratuities from large groups of people. A recent research study suggests this is not true. The researchers examined the relationship between the size of tips in restaurants and the number of meals charged on the bill. They found that, while most tips were around 15 percent, the minimum percentage considered appropriate, people dining alone tipped consistently more (19 percent) and those dining in groups of four or more tipped considerably less (13 percent) than this 15 percent standard. These results strongly suggest that people dining in a group are less likely to feel personally responsible for leaving an adequate or generous tip.
     一种普遍观点认为服务生更容易从群体就餐者那里得到更多的小费。最近一项研究表明并不是这样的。研究者调查了餐厅中收到的小费数量与就餐人数的关系。他们发现大多数小费约为15%,这是被认为恰当的最小比例;一个人就餐的小费往往较多(19%)而四个或更多人一起就餐时小费要比15%的标准值少的多(13%)。这一结果清楚地表明成群就餐的人较少感到应该留下足够的或慷慨的小费。

gratuities from large groups of people 论断:普遍认同的服务员在服务一群人收到的小费多的观念是不正确的。因为一群人用餐的时候人们个人比较不愿为留下足够的慷慨小费。一项调查发现,大多数小费都是在15%,这是觉得合适的最小量。而一个吃饭的时候小费一般都多,而四五个人的时候比这还少。
        前提,一群人吃饭的时候小费给得少。虽然比例少于15%,但他们花销大,虽然总额比人少的时候多。
        证据:菜单上的meals和吃饭人的多少没有一定比例关系,同时和最后的金额也没有一定的关系。所以方法不对。
        有人隐瞒不报。

200.Statistics collected from dentists indicate that three times more men than women faint while visiting the dentist. This evidence suggests that men are more likely to be distressed about having dental work done than women are. Thus, dentists who advertise to attract patients should target the male consumer and emphasize both the effectiveness of their anesthetic techniques and the sensitivity of their staff to nervous or suffering patients.
     从牙医那里获得的统计数据表明在看牙医时男性昏厥的次数是女性的三倍。这一证据说明男性在接受牙科治疗时比女性更容易感到痛楚。因此,那些做广告来吸引患者的牙医应该定位于男性顾客,并同时强调他们的麻醉技术和他们的职员对于紧张和痛苦患者的敏感度。

three times more men than women faint while visiting the dentist 论断:打广告吸引病人的牙医应该把目标放在男顾客身上,并强调他们麻醉技术的效果以及员工对病人紧张或痛苦的敏感。因为从牙医那儿收集的数据表明,看牙医晕的男人比女人多三倍,这显示男人比女人接受牙治疗时更紧张。
        前提,男人比女人晕得多。男女病人的各自总数没有提,有可能男人牙病的多。而且也没有提病症,有可能男人的牙问题比女人的严重。
        男人比女人看牙医容易紧张。女人可能也紧张,但不是晕,还有其它的表现,比如说心里,还有的干脆不去。
        结论,打广告:没人说医生敏感就不紧张,得技术好。不关女病人,万一不看牙医的女人多呢。

201.The citizens of Forsythe have adopted healthier lifestyles. Their responses to a recent survey show that in their eating habits they conform more closely to government nutritional recommendations than they did ten years ago. Furthermore, there has been a fourfold increase in sales of food products containing kiran, a substance that a scientific study has shown reduces cholesterol. This trend is also evident in reduced sales of sulia, a food that few of the healthiest citizens regularly eat.
     Forsythe的居民选择了更健康的生活方式。他们对于最近一项调查的回答显示,他们的饮食习惯比十年前更加贴近政府的营养建议。而且,含有kiran的食品销量增长了四倍,在一次科学研究中发现kiran是一种能够降低胆固醇水平的物质。这种趋势同样也被sulia的销量下降所证实,sulia是那些最健康的居民极少经常食用的食品。

conform more closely to government nutritional recommendations 论断:F的市民现在采用了更健康的生活方式。因为在最近的一次调查中,他们反应出较十年前更接近政府营养建议的饮食习惯。另外,含科学证明可降胆固醇的K的食品销量增长了四倍,而且S的销量下降了也是这个趋势的一个证据,很少有最健康的居民经常食用这种食物。
        前提:饮食更健康。证据的问题,第一,接近政府的营养建议,由于不知道这个建议的具体内容,所以无法判断这种情况是否就更健康了,这只能说当地人的饮食方式有意或无意地接近一个普遍接受的饮食习惯,但是是否符合当地的情况,还不得知。第二,K的上升不一定是因为人们关心健康或是关心胆固醇,这类食品也许存在其他特征吸人们,而且这类食品中也可能还含有其他不健康的万分。第三,S的下降与人们是否更健康也无关,没有证据证明食用它会导致不健康。
结论:饮食更健康不一定生活方式就健康,还有运动,心理等等因素。

202.Humans arrived in the Kaliko Islands about 7,000 years ago, and within 3,000 years most of the large mammal species that had lived in the forests of the Kaliko Islands had become extinct. Yet humans cannot have been a factor in the species' extinctions, because there is no evidence that the humans had any significant contact with the mammals. Further, archaeologists have discovered numerous sites where the bones of fish had been discarded, but they found no such areas containing the bones of large mammals, so the humans cannot have hunted the mammals. Therefore, some climate change or other environmental factor must have caused the species' extinctions.
    大约7000年前人类到达了Kaliko岛,在3000年内曾经生活在Kaliko岛的树林中的大型哺乳动物绝大多数已经灭绝了。然而人类并不是导致这些物种灭绝的因素,因为没有证据表明人类与这些哺乳动物有很多接触。而且,考古学家发现一些有大量鱼骨被抛弃的场所,而他们并没有发现存在大型哺乳动物骨头的类似场所,因而人类并没有猎杀这些哺乳动物。因此,一定是一些气候上的变化或其他环境因素导致了这些物种的灭绝。

Humans arrived 7,000 years ago,within 3,000 years mammal extinct. 论断:天气变化或其他环境因素导致了种类灭绝。因为人不可能是灭绝的一个因素。没有证据表明人和哺乳动物有很多接触。另外考古学家发现了很多带有鱼骨的地方,但没有发现含有大型哺乳动物的骨头的地方,所以人不可能捕杀哺乳动物。
        前提,不是人造成的灭绝。没有接触也能造成灭绝,破坏食物链,污染等等。
        前提,人没有捕杀哺乳动物。没有发现有遗弃骨头的地方,不证明不猎杀。有可能只带肉回来,或是把骨头做成了装饰品。There still a logical error made by the arguer by treating a lack of proof that something is the case as consituting sufficient proof that it is not the case.
结论:就算不是人造成的,人也脱不了干系。(cannot have been a factor??)

203.The following appeared in a newspaper feature story.
   "At the small, nonprofit hospital in the town of Saluda, the average length of a patient's stay is two days; at the large, for-profit hospital in the nearby city of Megaville, the average patient stay is six days. Also, the cure rate among patients in the Saluda hospital is about twice that of the Megaville hospital. The Saluda hospital has more employees per patient than the hospital in Megaville, and there are few complaints about service at the local hospital. Such data indicate that treatment in smaller, nonprofit hospitals is more economical and of better quality than treatment in larger, for-profit hospitals."
    在Saluda镇的小型非盈利医院,患者平均逗留时间是两天;在邻近的Megaville市的大型盈利医院,患者平均逗留时间为6天。而且,Saluda医院患者的治愈率大约是Megaville医院的两倍。Saluda平均每个患者对应的医务人员的数量比Megaville医院多,而且地方医院关于服务的投诉也较少。这些数据表明小型非盈利医院的治疗比大型盈利医院更加经济,质量更高。

nonprofit hospital 论断:小的非赢利医院的治疗比大的赢利医院更经济质量也更好。因为S的非营利医院病人的平均住院天数是两天,附近M市大的营利医院的平均住院天数是六天。另外,S的疗愈率是M的两倍。S每位医生看的病人也比M的多,抱怨也少。
        前提,治疗质量。没有直接的对比数字,无法得出。提供的住院天数没有意义。重病的人一般去大医院,而且由于S是非营利的,对资源使用控制并不完全视病情需要,不排除有可能病人没好就让人走人。
        前提,经济。乍一看有可能,因为S不赢利,M赢利,不过如果把治疗效果考虑进来,情况就要复杂一些。提供的每位医生看病人的比较没有意义。因为S没有钱支付更多的人员工资,所以每个人看的病人多,每位病人得到的治疗少,但又不能保证医生水平会比M高,所以这一点是不被保证的。
        抱怨,因为不知道两家医院接待病人的情况所以无从比较。另外,S非营利可能,病人也就不抱怨了,或者是穷人,不愿惹麻烦。而M就不一样了,因为营利,人们对它会挑剔一些。

204.The following appeared as a letter to the editor of a farming publication.
     "With continuing publicity about the need for healthful diets, and with new research about the harmful effects of eating too much sugar, nationwide demand for sugar will no doubt decline. Therefore, farmers in our state should use the land on which they currently grow sugar cane to grow peanuts, a food that is rich in protein and low in sugar. Farmers in the neighboring country of Palin greatly increased their production of peanuts last year, and their total revenues from that crop were quite high."
    随着对于健康饮食需求的持续报导,以及关于食用过多的糖对于健康危害的新研究,全国对于糖的需求无疑将会下降。因此,本州的农场主应该把他们现在种植蔗糖的土地用于种植花生,花生富含蛋白质而糖的含量低。邻近国家Palin的农夫去年极大增加了花生的产量,他们在这种作物上的收入相当可观。

harmful effects of eating too much sugar 论断:我们州种糖蔗的地应该用来种花生。因为对健康饮食的宣传,以及关于吃太多粮的坏作用的研究会带来全国性的糖需求无疑的下降。隔壁P国去年提高了花生的产量,他们粮食收入很高。
        前提,糖的需求会下降。没有直接证据,关于健康与研究问题不一定会有这样的结果。首先,没人说现在人的糖摄入量再长就不健康,其次,不健康人就不吃了吗,那烟的销量为什么还年年攀升。
        前提,应该种花生。没有直接证据证明我们种花生会赚钱。隔壁国的证据是false analogy.首先,天气不同,环境不同,搞不好我们这儿种不出果来。其次,技术问题,人家可能前两年技术有了新突破才想到增加种植的。第三,没有证据说他们种花生赚钱了,粮食嘛,还有其他呢。
结论:就算糖不赚钱了,换种花生也会很麻烦。另外不种花生还可以种别的呀。  

205.The following appeared in a recommendation from the president of Amburg's Chamber of Commerce.
     "Last October the city of Belleville installed high intensity lighting in its central business district, and vandalism there declined almost immediately. The city of Amburg has recently begun police patrols on bicycles in its business district but the rate of vandalism there remains constant. Since high intensity lighting is apparently the most effective way to combat crime, we should install such lighting throughout Amburg. By reducing crime in this way, we can revitalize the declining neighborhoods in our city."
     去年10月Belleville市在其中央商业区安装了高照度灯光,那里的破坏公物的行为几乎立即减少。Amburg市最近开始在其中央商业区安排警察骑自行车巡逻,但破坏公物的发生率并没有变化。由于高照度灯光显然是震慑犯罪的最有效途径,我们也应该在Amburg全市安装这种灯光。通过以这种方式减少犯罪,我们可以使本市重新繁荣起来。

intensity lighting and vandalism 论断:降低犯罪后,我们可以扭转我们城市人际居住环境的下降,我们应该在全A装这样的灯。因为它显然是打击犯罪的最有效途径。去年十月B城在商业区中心这样作了,肆意破坏活动几乎立刻下降。A城最近开始警察骑自行车在商业区巡逻,但是破坏率还是没有变。
        前提,装灯会降底犯罪率。没有直接证据证明这样会有效。隔壁B城的降低了肆意破坏行为,但A可能根本就没有这类的犯罪,而另外抢动强奸到挺多。
        论据也有问题,因为没有证据说是灯起的作用,它们可能同时还有其他措施,比如新法规,比如警察增多等等。
结论:犯罪降低了,neighood也不一定就提高。还有其他因素可能导致,比如经济萧条等等。

206.The following appeared in a letter to the editor of the Parkville Daily Newspaper.
     "Throughout the country last year, as more and more children below the age of nine participated in youth-league softball and soccer, over 80,000 of these young players suffered injuries. When interviewed for a recent study, youth-league softball players in several major cities also reported psychological pressure from coaches and parents to win games. Furthermore, education experts say that long practice sessions for these sports take away time that could be used for academic activities. Since the disadvantages apparently outweigh any advantages, we in Parkville should discontinue organized athletic competition for children under nine."
      去年在全国范围内,随着越来越多的9岁以下儿童参加少年棒球和足球比赛,有超过80000的少年选手受伤。在最近一次研究所作的采访中,很多大城市的少年棒球选手报告说存在来自教练和家长要求赢得比赛的心理压力。而且,教育专家指出这些运动项目长时间的训练占据了本应用于学习的时间。由于不利因素明显超过了有利因素,我们Parkville应该停止组织9岁以下儿童的体育比赛。

children below nine in softball injuries 论断:P应该停止组织九岁以下孩子的体育竞赛。因为它的坏处大于它的好处。去年当全国有越来越多的九岁下孩子参加垒球和足球的青年队时,这些年青的运动员有越过8万人受伤。最近的一份调查看,一些大城市的垒球青年队队员说有来自教练和父母要赢的压力。另外,教育专家说这些体育活动长时间的训练占用了应该用于学术活动的时间。
        前提,九岁下孩子参加体育活动受伤。没有证据。既没说这8万人里有没有九岁下儿童,也没有说8万人是否比以前就多了。
        前提,体育比赛让孩子有心里压力。证据来自几个大城市,说的人可能针对的是高水平的比赛。没有证据说P城里九岁下的孩子也有这种压力。
        前提,占用学术时间。首先没有证据证明P城九岁下的孩子不参加体育活动就会增加自己的学术时间。其次,没有证据说这些参加体育活动的孩子学校表现不好。
        结论,忽视体育比赛的好处,何来好与坏的对比。

207.It is known that in recent years, industrial pollution has caused the Earth's ozone layer to thin, allowing an increase in the amount of ultraviolet radiation that reaches the Earth's surface. At the same time, scientists have discovered, the population of a species of salamander that lays its eggs in mountain lakes has declined. Since ultraviolet radiation is known to be damaging to delicate tissues and since salamander eggs have no protective shells, it must be the case that the increase in ultraviolet radiation has damaged many salamander eggs and prevented them from hatching. This process will no doubt cause population declines in other species, just as it has in the salamander species.
     我们知道近年来工业污染导致地球臭氧层变薄,使到达地球表面的紫外线辐射增加。同时,学者发现在山区湖泊产卵的某种蜥蜴的数量下降了。由于紫外线辐射能够破坏脆弱的组织并且蜥蜴的卵没有保护壳,因此一定是紫外线辐射的增加破坏了很多蜥蜴的卵并阻止它们孵化。这一过程无疑也会导致其他物种数量的下降,就像发生在这种蜥蜴身上的一样。

pollution has caused ozone layer to thin 论断:紫外线照射的增加一定破坏了很多S蛋并阻止了它们的孵化,而且这个过程无疑也会造成其他种类数目的下降。因为工业污染已造成地球臭氧层变薄从而增加了到达地球表面的紫外线的量,同时科学家发现一种在山上的湖里产孵的S的数目在下降。紫外线已知对S蛋精致的组织有破坏,而S蛋无保护壳。。
        忽视的前提,是否所有的S都在下降,一种在下降,不一定所有的都在下降。不排除只有它下降,而其他种在上升。
        前提,这种S数目的下降是因为蛋的孵化受到了影响。没有直接的证据证明现在孵化出来的S,或是说小S比以前少了。还有其它原因会造成数目下降,比如人为破坏环境和捕杀造成的成年S下降。explain result contributed by many causes with just one reason which can not garantee the result.
        结论,紫外线使得S蛋不能孵化。虽然S蛋没有保护壳,而紫外线对于S的组织有破坏作用,但都不被保证一定会使得严重到不能孵化。另外其它动物不一定会一样。

208.The following appeared in a memorandum from the planning department of an electric power company.
"Several recent surveys indicate that homeowners are increasingly eager to conserve energy and manufacturers are now marketing many home appliances, such as refrigerators and air conditioners, that are almost twice as energy-efficient as those sold a decade ago. Also, new technologies for better home insulation and passive solar heating are readily available to reduce the energy needed for home heating. Therefore, we anticipate that the total demand for electricity in our area will not increase, and may
decline slightly. Since our three electric generating plants in operation
for the past 20 years have always met our needs, construction of new
generating plants should not be necessary."
      一些最近的调查表明,房主越来越强烈地希望节省能源,并且生产商现在正在推出很多比十年前的电器几乎节能两倍的家用电器,比如冰箱和空调。而且,更好的房屋隔热和被动式太阳能采暖的新技术已经可以用于减少家庭采暖所需的能源。因此,我们预计我们地区的用电需求总量不会增加,而可能有轻微下降。由于我们的已经运作了20年的三座发电站总能够满足需求,我们无需建造新的发电厂。

homeowners eager to conserve energy 论断:建新的电厂不需要。因为三家电厂在过去20年里都满足了我们的需求,而我们的用电总量有可能不会上升并有希望会小幅下降。几项最近的调查显示人们越来越希望保持能源,而制造商的家用电器也比十年前节电能力提高了两倍。另外新的技术使家庭隔缘更好,日光照射等也使得家庭供暖的用电需求会下降。
        前提,家用电量不会上升。没有直接的证据表明现在的用电指数的走势。给的证据不足以保证这一点。首先,人们的愿望和新的家用电器的节能不能保证这样的结果。已经有的是否愿意换新的,而原来没有的买新的,必然就增加用电量。其次供暖用电下降,其他用电会上升。
        前提,三家电厂现在还能满足需求。没有直接证据,而过去满足不代表现在还能满足。
        结论,不必建新的。干嘛不以备后用,城市发展,工业发展用电可能会上升。

209.The following recommendation was made by the Human Resources Manager to the board of directors of the Fancy Toy Company.
      "In the last three quarters of this year, under the leadership of our president, Pat Salvo, our profits have fallen considerably. Thus, we should ask for her resignation in return for a generous severance package. In Pat's place, we should appoint Rosa Winnings. Rosa is currently president of Starlight Jewelry, a company whose profits have increased dramatically over the past several years. Although we will have to pay Rosa twice the salary that Pat has been receiving, it will be well worth it because we can soon expect our profits to increase considerably."
     在今年的过去三个季度里,在我们总裁Pat Salvo的领导下,利润显著下降。因此,我们应该给予她优厚的补偿并要求她辞职。我们应该任命Rosa Winnings来代替她的职位。Rosa当前是Starlight Jewelry的总裁,这家公司的利润在过去几年中显著增长。尽管我们要付给Rosa两倍于我们付给Pat的工资,但这将是值得的因为我们马上可以看到利润的显著增长。

Fancy Toy Company 论断:我们应任命R,并要求P辞职。因为尽管R的薪水是P的两倍,但他会使我们的利润迅速显著上升。而在过去3个季度里,现任总裁P的领导下,我们的利润下降明显。R现在是S珠宝店的头,这家公司的利润在过去几年里一直急剧增加。
        被忽略的前提,R会来这儿。这儿效益这么不好。
        前提,R能使我们的利润迅速上升。没有直接的证据证明R对我们公司的情况了解并有很好的建议。而关于他在珠宝店的业绩不说明问题:首先行业不同,其次珠宝店的成绩不是他一人的,员工的作用。 •
        前提,P的能力不行。首先3个季度的工作没有说服力。其次,利润下滑不是他一个人的原因。可能是以前留下的隐患。第三,没准下滑是暂时的,他采取了一些措施,效果在长期。
•结论:R薪水是P的两倍,利润能增长多少却不知道。

210.The following is a letter to the editor of a news magazine.
     "Clearly, the successful use of robots on missions to explore outer space in the past 20 years demonstrates that robots could be increasingly used to perform factory work more effectively, efficiently, and profitably than human factory workers. The use of robots in factories would offer several advantages. First, robots never get sick, so absenteeism would be reduced. Second, robots do not make mistakes, so factories would increase their output. Finally, the use of robots would also improve the morale of factory workers, since factory work can be so boring that many workers would be glad to shift to more interesting kinds of tasks."
     显然,过去20年中机器人在探索外层空间任务中的成功使用证明,机器人可以比人类工作人员更有效、更有益地用于进行很多工作。工厂中机器人的使用将带来若干好处。首先,机器人从不生病,从而旷工将会减少。其次,机器人不会出错,因此工厂产量将会增加。最后,使用机器人同样也会提高工厂工人的精神状态,因为工厂工作有时如此枯燥以至于很多工人将会乐于转换到更有趣的任务。

use of robots on missions to explore outer space 论断:机器人探索外空间在过去20年里的成功使用表明机器人在工厂里的更多使用能比工人更有好,更有效,利润更多。因为它有几个好处:第一机器人不得病,所以旷工会减少。第二机器人不会犯错,所以产量能上升。最后机器人的使用还会提高工作的士气,因为工厂工作很枯燥,工作会愿意换去做其他更有意思的活。
        被忽略的前提:有无用于工厂使用的机器人。外空间探索与工厂工作是两回事,软硬件要求都不一样。
        所提到的三个好处:第一,机器人会出故障,不但停工还得花钱修。第二,机器人只会执行命令,出现异常没有主动性和创造性地解决问题,带瑕的产品不能防止。第三,没有工人说现在的工作枯燥,换了工作后也还人枯燥。另外有没工作给他们,如果因为机器人而被解雇的风险到是挺大。
结论:利润不一定上升。买机器人的钱,维修的钱。技术工人的钱等等。不是所以有的行业都适合。

211.The following appeared in a memorandum to faculty from the academic vice president of Waymarsh University.
     "So that we can better accomplish Waymarsh University's academic goals, we should adopt the job-opportunity (job-op) program offered at Plateau Technical College and strongly encourage all students at Waymarsh to participate in it. The success of the job-op program at Plateau is evident: over the past two years, more than 75% of the freshmen at Plateau have enrolled in the optional job-op program. Moreover, at Plateau, the grades of job-op students are consistently higher than those of other students, 90% of the job-op students receive job offers within a month after their graduation, and most former job-op students report much success in their careers."
      为使我们更好的完成Waymarsh大学的教学目标,我们应该采纳Plateau技术学院所提供的就业机会(job-op)课程,并积极鼓励所有Waymarsh的学生参加该课程。Plateau的job-op课程的成功就是证据:在过去两年中,Plateau75%的一年级学生选修了job-op课程。而且,在Plateau参加job-op课程的学生成绩要比其他学生高很多,90%的参加job-op的学生在毕业后一个月内得到了就业机会,多数以前的job-op学生报告了在他们职业中的成功。

job-op program 论断:为了更好地完成W大学的学术目标,我们应该采用P校提供的工作机会项目,并强烈鼓励W大学的学生参加。P校的这个项目的成功是有证据的:过去两年里P校75%以上的一年级学生都参加了这个可选择的项目。而P校这个项目的毕业生比其他学生的分数高很多,而且90%毕业后一个月内就收到了工作offer,大多数以前这个项目的学生也报告说职业生涯很成功。
        前提,工作项目会受学生欢迎。没有证据表明W校的学生会喜欢,而P校的经历没有参照性。学校不一样,学生不一样。
        前提,P校的这个项目成功。第一,75%的一年级学生参加,但他们还会退出,新生嘛,可以理解,什么都不了解嘛。第二,分数比别人高,不一定是项目的功劳,而收到OFFER的问题,什么工作?W的学生或许大多数还没毕业就能收到。第三,谁愿意说自己工作不成功呀。
结论:W的学术目标是什么?让学生找工作?还是研究,培养四有新人。

212.The following appeared in a memorandum from the new president of the Patriot car manufacturing company.
     "In the past, the body styles of Patriot cars have been old-fashioned, and our cars have not sold as well as have our competitors' cars. But now, since many regions in this country report rapid increases in the numbers of newly licensed drivers, we should be able to increase our share of the market by selling cars to this growing population. Thus, we should discontinue our oldest models and concentrate instead on manufacturing sporty cars. We can also improve the success of our marketing campaigns by switching our advertising to the Youth Advertising agency, which has successfully promoted the country's leading soft drink."
     在过去,Patriot汽车的车型已经过时了,我们的汽车没有竞争对手的车卖的好。但现在,由于我国很多地区的新驾驶员数量飞速增加,我们可以通过向这一日渐增长的人群出售车辆来增加我们的市场份额。因此,我们应该停止生产最老的型号而集中力量生产新颖的汽车。我们也可以通过把我们的广告交给Youth广告公司来促进市场推广的成功,这家公司曾经为本国销量最大的软饮料进行过成功的促销。

body styles of Patriot cars have been old-fashioned 论断:我们应该停止最旧款的车型而集中制造运动型轿车,我们应该用Y广告公司。因为我们的车型已经过时,车也没有对手的卖得好。不过现在很多地区新拿驾照的人数迅速上升,我们应该可能通过向这个增长部分的人群售车提高利润。另外,我们还可能换用Y广告公司促进销售的成功,他们已成功地推销了这个国家最主要的软饮料。
        前提:运动型轿车会有市场。没有直接证据证明,新拿驾照的人喜欢运动型车。而且运动型车也有新款和旧款。
        前提,新拿驾照的人会买车。没有直接证据证明。他们可能借、买旧车以及租车。
        前提:Y广告推销成功了饮料,但他们可能不了解汽车市场。
结论:尽管没有对手卖得好,旧款车不一定就销量就不行。而且不一定是款型的问题,性能,舒适以及价格都有可能。而且销售成功,广告只是一个因素。

使用道具 举报

Rank: 2

声望
9
寄托币
139
注册时间
2009-4-2
精华
0
帖子
28
23
发表于 2009-4-26 22:23:46 |只看该作者
213.The following appeared in a memorandum from the owner of Armchair Video, a chain of video rental stores
     "Because of declining profits, we must reduce operating expenses at Armchair Video's ten video rental stores. Raising prices is not a good option, since we are famous for our special bargains. Instead, we should reduce our operating hours. Last month our store in downtown Marston reduced its hours by closing at 6:00 P.M. rather than 9:00 P.M. and reduced its overall inventory by no longer stocking any film released more than two years ago. Since we have received very few customer complaints about these new policies, we should now adopt them at all other Armchair Video stores as our best strategies for improving profits."
    由于利润下降,我们必须压缩Armchair Video十家录像租赁店的营业开支。涨价并不是一个好选择,因为我们就是因特价销售而知名的。因此我们应该缩短营业时间。上个月我们位于闹市区Marston的店营业时间从以前的9点缩减到下午6点,并通过不再保留发行两年以上的电影来减少总库存量。由于我们所收到的客户关于这些新政策的投诉很少,我们应该在所有Armchair Video的商店实施这些政策作为增加利润的最好策略。

reduce operating expenses at ten video rental stores 论据:在全A的十家店里缩短营业时间和减少库存是得高利润的最好办法。因为利润的下降我们必须减少十家音像租赁店的开支。而提高价格不是一个好的办法,因为我们是以特价闻名。相反过去一个月我们在M的店缩短了三个小时的营业时间和整个库存,收到的抱怨很少。
        前提,缩短时间和库存是好办法。没有具体对全部的论证,给出的M店证据没有说服力。首先,没有提M过去一个月里的利润是不是上升。其次,有人不满也不抱怨,刚一个月就有人抱怨,可能接下来就不来了。第三,一个店的情况不能套用到所有上。
        被忽略的,减少营业时间和库存,即意味着把更多的顾客让给竞争对手。
结论:节约开销不等于利润提高。干嘛说是最,其它方法还有呀。提高价格不一定不行,如果成本实在下不来,可以适当提价。

214.In each city in the region of Treehaven, the majority of the money spent on government-run public school education comes from taxes that each city government collects. The region's cities differ, however, in the value they place on public education. For example, Parson City typically budgets twice as much money per year as Blue City does for its public schools-even though both cities have about the same number of residents. It seems clear, therefore, that Parson City residents care more about public school education than do Blue City residents.
     在Treehaven地区的每个城市,政府用于公立学校教育的开支大部分都是 从政府征收的税收而来的。然而,该地区不同城市对公共教育的重视程度是不同的。举例而言,Parson市用于公立学校的预算通常是Blue市的两倍,尽管两城市居民数量基本相同。因此,Parson市的居民显然比Blue市居民更关注公立学校教育。

each city in the region of Treehaven 论断:P城的居民比B城更关心公立学校的教育。P城每年拨到公立学校的财政经费是B城的两倍,尽管两个城市的居民人数相同。
        •B城收入可能比P城少五倍,这样他的教育经费所占的比例是P城的近三倍。
        P城的学生是B城的四倍,每个学生摊到的是只有B城的一半。
        钱花得多不一定就更关心,政府的态度与居民的态度不一定一致。

215.The following appeared in a letter to the Grandview City Council from a local business leader.
     "During last year's severe drought, when the water supply in the Grandview city reservoir fell to an extremely low level, the city council imposed much more rigid water-rationing rules. But just after these rules were imposed, industrial growth in the area declined. This clearly shows that the new rationing rules have hurt industry in Grandview. Therefore, to promote the health of the local economy, the city council should now stop water rationing."
     在去年严重的干旱中,当Grandview市水库的储水量降至极低水平时,市委发布了更加严格的供水配额条例。但就在这些条例实施以后,该地区的工业增长减缓了。这显然表明新的供水配额条例对Grandview的工业造成了破坏。因此,为促进当地经济健康发展,市委现在应该停止供水配额。

water supply in reservoir fell to an extremely low level 论断:为促进健康的当地经济,政府现在应该停止对水进行限量。因为它伤害了当地的工业,就在这个办法实施没多久,当地的工业增长就下降了。在去年最干旱的时候,水库水降到了最低点,政府就实施了太严的限量供水措施。
        前提,是限制用水导制工业增长下降。the arguer treats events happened after something as the consequence of that. industrial growth declined does not equal industrial declined, maybe the local industrial might have reached a plateau.
        前提:工业增长不等于健康经济。不限制用水可能会导致其它问题。
结论:不一定要停止,可以适当放松。even some of these water-rationing rules did harm the industrial ,no evidence shows that city coucil should stop water rationing. they can just changes the rigidity of these rules.

216.The following appeared in a magazine article about planning for retirement.
     "Because of its spectacular natural beauty and consistent climate, Clearview should be a top choice for anyone seeking a place to retire. As a bonus, housing costs in Clearview have fallen significantly during the past year, and real estate taxes remain lower than those in neighboring towns. Nevertheless, Clearview's mayor promises many new programs to improve schools, streets, and public services. Retirees in Clearview can also expect excellent health care as they grow older, since the number of physicians in the area is far greater than the national average."
       由于Clearview的天然景色和温和气候,它应该成为那些为退休后寻找生活地的人的首选。其中一项好处是,Clearview的房价在去年显著下降,房地产税一直比邻近城市低。而且,Clearview的市长承诺了很多新方案来改善学校、街道和公共服务。Clearview的退休人员也可以在老年时享受到出色的医疗服务,因为该地区的医生数量高于全国平均水平。

planning for retirement 论断:C是人们找地方退休的首选。因为当地风光优美,气候常年稳定,而作为退休金,住房在当地的费用去年也有很显著的下降,房地产税收一直比隔壁几个镇低。不过,市长许诺了新的项目改善学校、街道和公共服务。退休在C还有好的健康服务,因为这儿医生的数目比全国平均水平高。
        如果C真的很适合退休居住,当地一定吸引了不少老人,尤其是退休后搬来居的。听听他们的意见是有说服力的,另外老年人需要同相年纪的伴,这是非常重要的。没有这方面的信息,论断的说服力非常弱。
        住房的费用在去年下降,并不证明它比其它地方低,而且今后是否还会上升也不被保证。另外住房费用低以及房产税比隔壁几个镇低,让人不得不担心一个问题,就是当地经济可能比较弱。虽然经济发达并不是老年人最关心的,但与此相关的社会问题会不会存在,比如高失业律带来的社会不稳定等等,另外还有一些服务会不会缺。
        许诺的项目并不真正存在,要多少年也说不定。而且学校和街道问题可能老年人并不关心。 •医生的水平比数目更重要。

217.The following appeared in a brochure promoting the purchase of local franchises for a national chain of gyms.
     "Now is the time to invest in a franchise so that you can profit from opening one of our gyms in your town. Consider the current trends: Power-Lift Gyms are already popular among customers in 500 locations, and national surveys indicate increasing concern with weight loss and physical fitness. Furthermore, last year's sales of books and magazines on personal health totaled more than $50 million, and purchases of home exercise equipment almost doubled. Investing now in a Power-Lift Gym franchise will guarantee a quick profit."
     现在是投资于Power-Lift Gym特许权的良好时机从而你可以在你们城市开设一家体育馆。考虑一下当前的潮流:Power-Lift Gym已经在500个地点的消费者中相当流行,全国调查表明对于减肥和健身的关注越来越多。而且,去年关于个人健康的书籍和杂志销售总额超过了5千万元,家庭健身器具的购买量几乎翻了一番。现在投资Power-Lift Gym特许权将保证快速致富。

purchase of local franchises for a national chain of gyms. 论断:投资P的联锁经营权能保证很快营利。因为有这样的趋势:P已经在五百个地方的消费者中很流行了,而且全国性的调查显示对减肥和体形健康的越来越关注。另外,去年在个人健康方面的书和杂志一共卖了5千万,而家庭健身设备几乎翻了一倍。
        前提:是否所有的P都在营利。
        既然五百多个地方都已经有P,市场是否还有空间允许新的投资进入。
        论据,这些论据只能说明在全国范围内人们越来越关心体形健康,除此之外没有任何意义。第一是投资人当地也许没有这样的趋势。第二是关心体形健康的人不保证都会去体育馆建身。这家体育馆的主营项目是什么我们并不知道,另外家用设备翻番说明人们有在家里锻炼的趋势,而不是去体育馆。

218.The following appeared in a memorandum from the president of Hyper-Go Toy Company.
     "Last year, sales of our Fierce Fighter toy airplane declined sharply, even though the toy had been a top seller for three years. Our customer surveys show that parents are now more worried about youthful violence and are concerned about better education for their children. Therefore, to maintain profits we should discontinue all our action toys and focus exclusively on a new line of educational toys. Several other toy companies have already begun marketing educational toys and report sales increases last year of 200 percent. And since the average family income is growing, sales of new Hyper-Go toys should also increase."
     去年,我们Fierce Fighter玩具飞机的销量锐减,尽管该玩具连续三年都非常畅销。我们的消费者调查显示,家长对于青少年暴力越来越担忧,并且关注他们的孩子是否能得到更好的教育。因此,为继续盈利我们应该停止生产所有的动作玩具并专门生产一系列新的教育玩具。若干其他玩具公司已经开始推广教育玩具并且报告说去年的销量增长了200%。由于平均家庭收入正在上涨,新的Hyper-Go玩具销量也会上升。

toy airplane declined sharply 论断:H新的玩具销售也会上升。因为家庭的收入在上升,而父母越来越担心小孩子的暴力问题以及给孩子好的教育,所以我们并停止所有动作玩具的销售并只集中在只教育玩具上。其它几家玩具公司已经开始推出教育玩具而且销售已被报告在去年增长了200%。
        因为家庭平均收在上升,新玩具的销售也会上升。如果事情这么简单,为什么老玩具的销售会有忽剧的下降?新玩具的设计以及它将来的促销,还有经济情况,都会影响它的销售。
        是否该停止生产旧玩具。家长担心暴力问题,并不证明这就是旧玩具下降的主要因素,因为没有证据证明家长现在不给孩子买动作玩具,或是少买。首先玩具与暴力的关联并不太,而且不是所有的动作玩具都与暴力相关。论者应该多考虑一下是否有其他问题,比如设计上有先进的类似产品出现,市场饱和问题等等。
        是否该只集中在教育玩具上。首先没有有关此类玩具销售比例上升的证明,所以不能证明虽然家长越来越关心教育,今后就会多买教育玩具,事实上家长一直都很关心教育。其次,其他公司推出此类玩具销售上升的问题,与这些公司的玩具设计、推广以及管理等问题都有关系,很有可能他们同时还推出了其他动作玩具,销售也在上升。另外,其他公司的销售增长如此之快,得考虑一下会不会竞争已经开始激烈,对于新的进入者的门槛会进一步提高。

219.The following appeared in a memorandum from a vice president of the Megamart department store chain.
      "For the third year in a row, the average household income in our country has risen significantly. That prosperity means that families are likely to be spending more time and money on leisure activities. Megamart stores should therefore concentrate on enlarging and promoting its line of products typically used in leisure activities: athletic and outdoor equipment, televisions, gourmet cooking equipment, and luggage and travel accessories."
      在连续三年中,我国家庭平均收入显著上升。这种繁荣意味着家庭可能会花更多的时间和金钱于休闲活动。因此Megamart商店应该致力于扩大和推广通常用于休闲活动的商品:包括运动和户外装备,电视,美食烹调设备,箱包和旅行用具。

household income in our country has risen significantly 论断:M店应该集中扩大推广用于休闲活动的产品线。因为我们国家已经连续三年家庭平均年收上升。这种繁荣意味着家庭更可能花多的时间和钱在休闲活动上。
        首先要质疑的是家庭年平均收入增长的实质,同时是否有通货膨胀?如果物价上长,这种收入上升根本不意味着任何变化。而且这种增长是否会持续。
        收入上升与休闲活动之间的因果关系不知如何得来?想想如果与此同时工作量加上,人们生活越来越紧张,又怎么可能去休闲。
        还要考虑一下目前该市场的情况如何,是否有出现供过于求。

220.The following appeared in an article in a magazine for writers.
     "A recent study showed that in describing a typical day's conversation, people make an average of 23 references to watching television and only 1 reference to reading fiction. This result suggests that, compared with the television industry, the publishing and bookselling industries are likely to decline in profitability. Therefore, people who wish to have careers as writers should acquire training and experience in writing for television rather than for print media."
     最近一次研究显示当描述日常对话的时候,人们平均有23次提到看电视而只有一次提到读小说。这一结果说明与电视行业相比,出版和书籍销售行业的盈利能力可能会下降。因此,想要以作家为职业的人应该接受为电视而不是为印刷媒体写作的训练和经验。

people make references to television and reading fiction. 论断:想当作家的人应该获得为电视写作的训练经验而不是为印刷媒体。因为最近一份研究显示人们日常的谈话平均引用电视和小说的比例为23:1。这个结果表示,与电视产业相比,出版和书的销售利润可能都在下降。
        前提,出版和销售的利润在下降。没有任何有关的数字。而所谓的调查只讲了小说,而印刷媒体中有很多种,不只是小说。电视的利润也不一定就是在上升。
        论据,该调查非常不可信,首先描述不科学。其次,比较的是电视和小说。但是电视节目内容很广,比如新闻、或是其他类似issue的东西。这些因为与人们生活比较接近,所以容易被谈论到,但这些都不是作家可以写的。第三,电视因为其媒介的特性,一般为很多人同时收看,而小说阅读是相对个人的活动,所以不太容易被谈论到。但人们的谈论与他们主要接触什么样的媒体并没有直接的联系。
结论:事实上有很多东印刷媒体的东西并改编成了电视,当初压根没想它的。

221.The following appeared in the editorial section of a student newspaper.
    "In a recent survey, most students who were studying beginning Russian gave higher course-evaluation ratings to classes taught by non-native Russian speakers than to classes taught by native Russian speakers. The reason that the non-native speakers were better teachers of Russian is easy to see: the non-native speakers learned Russian later in life themselves, and so they have a better understanding of how the language can be taught effectively. Therefore, in order to improve instruction for all languages and also save money, our university should hire non-native speakers as language instructors instead of trying to find and recruit native speakers."
     在一次最近的调查中,学习初级俄语的学生给予非俄国教师所教的课的评价要高于给予俄国教师所教的课。非俄国人作为俄语教师更加优秀的原因是显而易见的:非俄国人在长大以后自己学习俄语,因此他们对于这种语言如何教授将更加有效有着更深刻的理解。因此,为提高所有语言课程的质量并节省开支,我们大学应该雇佣非本国人来担任语言教师而不是试图寻找和雇佣本国人。

non-native Russian speakers 论断:为提高语言教育并省钱,我们大学应该雇用非母语者而不是招收新的母语者。因为最近的调查中,大多刚开始学习俄语的学生里,由母语不是俄语的人教出来的比母语是俄语的人教出来的分数高。这个原因很明显:因为非母语的人是自己在后来的生活中学会的,所以他们更了解一门语应该怎么教有效果。
        前提,这样做会省钱,没证据。
        前提,非母语的教的比母语的好。引用的例子片面。初级学生的效果差别,不证明语言教学应该用非母语的。应该再考虑到中级学生和高级学生,这时候母语教师的价值就显出来了。
        论据:有问题。首先可能两部分学生的能力本身有差别。其次有可能是个别的现象。

222.The following article appeared in a recent issue of a college newspaper.
       "Among all students who graduated from Hooper University over the past five years, more physical science majors than social science majors found permanent jobs within a year of graduation. In a survey of recent Hooper University graduates, most physical science majors said they believed that the prestige of Hooper University's physical science programs helped them significantly in finding a job. In contrast, social science majors who found permanent employment attributed their success to their own personal initiative. Therefore, to ensure that social science majors find permanent jobs, Hooper University should offer additional social science courses and hire several new faculty members who already have national reputations in the social sciences."
     在过去五年从Hooper大学毕业的所有学生中,物理学专业的毕业生在毕业一年内找到固定工作的人数要多于社会科学的毕业生。在一次对于Hooper大学最近的毕业生的调查中,多数物理学专业的学生说他们认为Hooper大学物理学科的声望在他们找工作的过程中起到了极大的作用。相比之下,找到固定工作的社会科学毕业生则把他们的成功归因于他们自己的能动性。因此,为保证社科毕业生找到固定工作,Hooper大学应该提供更多的社科课程并雇佣一些已经在社科领域获得全国声誉的新教员。

more physical science majors 论断:为保证社会学系的学生找到身生职业,H大学应该再多提供社会科学课程并雇用几位新的已在全国该领域有名望的教师。因为过去五年里H大学有更多的理科学生比社会学系的学生在毕业后一年里找到了终身职业。在一份对毕业生的最近调查中,大多数理科生都说他们相信H在理科课程的名望在很大程度上帮助了他们找工作。而相反,社会学系找到终身职业的人把这种成功归结于他们自己的个人创新。
        前提,H校目前的社会学系不好。没有直接证据。
        前提,H校社会学系的学生不好找工作。论据有问题。首先毕业后一年里是否找到终生工作不说明问题,学生刚刚迈入,可能有学生不愿接受。其次,找工作与职业市场有关系,可能理科学生本来就比社会学的工作多。第三,学生把找工作上的成功归因,可能理科学生比较谦虚,而社会学的学生比较实诚。
结论:多提供课程和找新的有名望的老师并不能保证更多的社会学系的学生能找到终身职业。

223.The following appeared as an editorial in one of Coleville's city newspapers.
      "Even though a high percentage of Coleville City's businesses failed last year, we who live in Coleville City should keep in mind the fact that the Coleville region has attracted a great many new businesses over the last three years. It is well known that new businesses are, on average, much
more likely to fail than are long-established ones, so the business failures should not be considered a sign of poor economic health. Indeed, many analysts regard the presence of a significant number of new companies among a region's businesses as a sign of economic health. Thus Coleville City appears to be in good, not poor, economic shape."
     尽管Coleville市有很大比例的商业去年失败了,居住在Coleville市的人们应该记住这一事实:Coleville地区在过去三年中吸引了大量的新商业。众所周知平均而言,新商业比有一定历史的行业更容易失败,因此去年的商业失败不应该被看作是经济状况变坏的迹象。相反,很多分析家把一个地区商业中的大量新公司的存在看作是经济健康的标志。因此Coleville市的经济状况是良好的。

high percentage of businesses failed last year 论断:C城目前的经济形式很好,不坏。因为尽管去年C城有很高的商业失败比例,但我们应该认识到C城在过去三年里吸引了很多新的公司。大家都知道新的公司一般比老公司更容易失败,所以商业失败不应该看作是经济不好的标志。事实上,很多分析家认为一个地区有相当多数量的新公司是健康经济的标志。
        前提,吸引了很多新公司。过去三年里吸引了很多新公司,但没有证据证明现在仍有新公司进来。没有任何有关现在经济好的直接支持信息。所以在去年一年里倒闭了很多公司,说明问题已经存在。
        前提:倒闭的是新公司。没有证据证明倒闭的公司都是新公司,可能有大量老公司也倒闭了。
结论:如果说分析家认出现多数量的新公司是经济健康的标志,那分析家也说大量公司倒闭是经济不好的标志,论者没有提到。所以从论断内容得不出这样的结论。

224.The following appeared in a corporate planning memo at ABC Cereal Company, the makers of Better Bran cereal.
     "Sales of Better Bran have declined in recent years, for reasons that management has now identified. First, Better Bran is a cereal with high sugar content, and recent research studies have found that most consumers say they are concerned about the amount of sugar added to their breakfast cereal. Second, the price of Better Bran has increased by 5 percent in each of the last three years. Therefore, to increase our company's profits we need to reduce the amount of sugar in Better Bran and lower Better Bran's price."
    Better Bran的销量在最近几年下降了,领导层现已查明了原因。首先,Better Bran是一种含糖量较高的麦片,最近的研究发现,多数消费者说他们关注早餐麦片中所添加的糖的量。其次,Better Bran的价格在过去三年上涨了5%。因此,为增加公司的利润我们应该减少Better Bran中的含糖量并降低其价格。

ABC Cereal Company 论断:为提高公司利润,我们需要降低食品中的糖含量以及食品的价格。因为BB的销售最近几年一直在下滑。原因有一,BB糖含量高,而最近的研究发现大多数的消费者说他们关心在谷类早餐中加的糖。第二,过去三年里BB的价格每年升高5%。
        没有证据证明BB的下滑是因为顾客考虑到糖的问题。而引用的调查是说消费者关心在谷类早餐里加的糖,但论者没有说BB是不是早餐食品。所以说消费者对于糖的关心,与BB不一定相关。
        价格上升很自然,很多商品的价格都有每年的趋势。原因有很多,比如商品更受欢迎了,比如说人们的购买力上升了,还比如说市场有通货的问题。
        结论,忽视的问题有,口味没人喜欢了,有现了新的竞争者。简单降低糖和价格不能保证利润上升。

225.The following appeared as part of a letter to the editor of a local newspaper.
    "During her three years in office, Governor Riedeburg has shown herself to be a worthy leader. Since she took office, crime has decreased, the number of jobs created per year has doubled, and the number of people choosing to live in our state has increased. These trends are likely to continue if she is reelected. In addition, Ms. Riedeburg has promised to take steps to keep big companies here, thereby providing jobs for any new residents. Anyone who looks at Ms. Riedeburg's record can tell that she is the best-qualified candidate for governor."
     在州长Riedeburg在位的三年中,她证明了自己是一个合格的领导者。自从她就任以后,犯罪减少了,每年创造的就业机会数量翻了一番,选择在本州居住的人数也增加了。如果她再次当选,这种趋势很可能将会继续下去。而且,Riedeburg承诺说要采取措施使大公司留在这里,从而为新居民提供就业机会。任何目睹了Riedeburg工作成就的人都会认为她是州长的最佳人选。

worthy leader, trends are likely to continue if she is reelected. 论断:R是最合格的统治者候选人。因为她在任的三年里,R证明了自己是个有价值的领导者,犯罪下降,每年新的工作机会翻倍,越来越多的人选择在我们州居住。如果她连任,这样的趋势将继续。另外,R还许诺会采取措施留住几家大公司,这样能给新的居民提供工作。
        前提,R连任,情况会保持。没有证据。论者没有提供任何有关她在未来的任期里将要采取的措施。
        前提,是R让情况好起来的。把目前的好情况都归功给了R,对其他部门官员来说非常不公平。
        前提,大公司给新居民提供工作。没有证据证明这些大公司还能容下新员工,如果不裁员的话。其次,留下,是否已经说明这些公司要离开。

226.The following appeared as part of a memo from the manager of an automobile manufacturing company.
     "Because the demand for our automobiles is expected to increasedramatically, we need to open a new manufacturing plant as soon as possible in order to continue to thrive. Our marketing projections indicate that 80 million people will want to buy our automobiles, yet our existing plant can only produce 40 million automobiles. The new plant can be opened on a part-time basis, with workers from our existing site rotating responsibilities, until an operational staff can be trained. A major airplane manufacturer was extremely successful using this part-time rotating strategy when it opened its new plant five years ago."
     由于对我们生产的汽车的需求预计将有飞速增长,我们需要尽快开设一处新的制造工厂以保持繁荣。我们的市场计划指出有8千万人将要买我们的车,而我们现有的工厂只能生产4千万辆。新的工厂可以部分时间运转,让我们现有工厂的职工轮班工作,直到新的员工被培训出来。一家大型飞机制造公司在五年前开设了新工厂的时候使用了这种非全日轮班策略,取得了极大的成功。

part-time rotating strategy 论断:我们需要尽快开一家新分厂以继续繁荣,新的厂可以采用以兼职为基础,用现在的工人轮岗,直到一个可操作的员工训练完。因为市场记录显示有8千万人要买我们的汽车,但我们现有的厂子只能生产4千万。五年前一家飞机制造商用这种兼职轮岗的方式取得了极大的成功。
        前提,市场真的供不应求。首先projection的准确性没有被保证,因为论者没有提供任何资料交待。其次,想买不等于一定会买,但由于经济或其它原因不一定就会买。
        开新厂的开支非常庞大,如果供不应求的局面只是暂时的,将是很浪费,不是继续繁荣。
        现有员工轮岗,会加重员工的工作强度。它的成功不被保证。而飞机制造与汽车并不相同,而且它们可能还采用了一些其他的方式。

227.The following appeared as an editorial in a local newspaper.
       "In order to attract visitors to Central Plaza downtown and to return the plaza to its former glory, the city should prohibit skateboarding there and instead allow skateboarders to use an area in Monroe Park. At Central Plaza, skateboard users are about the only people one sees now, and litter and defaced property have made the plaza unattractive. In a recent survey of downtown merchants, the majority supported a prohibition on skateboarding in the plaza. Clearly, banning skateboarding in Central Plaza will make the area a place where people can congregate for fun or for relaxation."
      为使闹市区中央广场吸引更多的观光者并回复该广场以前的繁荣,我们市应该禁止在那里玩滑板,让滑板爱好者使用Monroe公园地区活动。在中央广场,人们唯一可以看到的就是玩滑板的人,废弃物和损毁的公物使广场失去了往日的吸引力。在最近一次对闹市区商户的调查中,大多数人都支持禁止在广场玩滑板。显然,在中央广场禁止滑板将会使该地区成为人们可以聚集起来娱乐和休闲的场所。

prohibit skateboarding 论断:禁止滑冰C广场能成为人们可以聚会和放松的场所。因为现在C广场,滑冰的人几乎是唯一能看到的使用者,而且纸屑垃圾使得广场很难看。最近的一项对市中心商人的调查,大多数人支持对滑冰的禁止。而且滑冰的人可以去M公园里的场地。
        •前提:是滑冰的人把广场弄得没有吸引力的。论者没有提供任何有关人们为什么不爱去广场的原因,很有可能他把因果关系倒置了,滑冰的人选择广场,是因为广场上没有人。这样,没有滑冰的人,人们也不会去广场。
        论据:谁说纸屑一定是滑冰的人扔的。
        商人的调查没有说服力。商人只关心自己的利润,并不能代表大多数市民。也许他们认为滑冰的人碍事,比如挡住了他们的店面,或者他们想在别的什么地方开一个收费的冰场。
        使用公园里的场地滑冰有可能存在问题。场地是否适合,是否很远等等。

228.The following appeared in a newsletter from a political organization.
      "In order to promote economic growth in the city, city residents should vote 'yes' on the state government's proposal to build a new expressway linking the outlying suburbs directly to the city center. A direct link to the city center will enable downtown businesses to receive deliveries more frequently, so that downtown retailers will no longer run out of stock and city manufacturers will not be affected by shortages of materials. Booming businesses will attract qualified workers from all over the state, workers who will be able to take advantage of the new expressway to commute to work in our city. In addition to these advantages, hundreds of workers will be employed to build the expressway, further stimulating the local economy!"
     为促进本市的经济增长,市民应该对州政府关于建造直接连接郊区和市中心的新高速路的提案投赞成票。到市中心的直接连接将使中心商业区的货运更加频繁,从而中心区的零售商就不会再断货,市内的生产商也不会受到材料短缺的影响。繁荣的商业将会从整个州吸引合格的工人,工人可以利用新的高速路在本市上班。除了这些好处,上百的工人将被雇佣修筑这条道路,进一步促进地方经济。

a new expressway linking suburbs directly to the city center 论断:市里的居民应该赞成政府建新高速联结市中心和郊区的建议。因为这可以使市中心更经常地收到送货,这样市里的零售商就不会再缺货,制造商也不会被材料短缺困扰。繁荣的商业会吸引全州合格的工人,他们可以享受新高速上下班出入我们城市。另外,雇用几百名工人修高速,更进一步剌激当地经济。
        前提,现在是否需要一条新的高速。没有证据证明当前的交通不能满足经济的需要,而提到的缺货缺原料问题,也没有证明一定是存在的,更没有证据证明它们是因为与交通的问题造成的。
        前提,有了高速会繁荣经济。没有证据证明出入城市方便了,就会有很多人愿意每天开车上下班地来到这座城市工作。城市的经济发展才会有吸引力。而对于用几百名工人修路的问题,首先修路只是一个短期的项目,一年半年结束后,工人的工作仍然没有着落。其次,修路将不可避免地占用现有的路,可能会有人的房屋、商店要被折,这会影响市民的士气。

229.The following appeared as part of a memo from the manager of a hazardous-waste disposal company.
       "Our new plan will help us better protect our staff against exposure to toxic chemicals: we are fitting each new safety suit with an alarm that will sound in the main control booth when the suit is punctured. The control booth will notify the managers on duty, who will then take the necessary steps to ensure employees' safety. In addition to the obvious safety-related benefits, the fast reaction time will ensure minimal work stoppage and thus result in increased worker productivity and company profitability."
        我们的新计划将帮助我们更好地保护员工避免有毒化学物质的影响:我们在每一件安全服上安装了报警装置,当安全服被刺穿时在总监控室将会发出警报。监控室将会通知值班经理,他将采取必要步骤来保证员工的安全。除了这些显而易见的安全方面的好处,第一反应时间将会保证最小的工作停顿,从而带来工人效率和公司利润的提高。

hazardous-waste disposal company 论断:我们的新计划将帮助我们更好地保护员工不受有毒化学品的侵害,而且它带来的最快反就也将保证最少的工作被停止从而提高工人的生产率和公司的利润。这个计划是:给每个新安全服装上闹钟,当安全服被刺破时,主控室里就会响,然后会通知当班的经理,他就能采取需要的措施保证员工安全。
        前提,更好地保护。首先论者没有提供有关现有的措施,以及可能接触到的主要有毒物品的特征,我们无法根据论断得出这样的结论。而从对新措施的描述来看,在当班经理采取步骤时,安全服已经被刺破了一段时间,因为主控室首先要找到当班经理,而经理再找到员工,这段时间看起来不会太短,而员工已经接触到了有毒物。
        结论,利润。没有证据证明现在因为安全问题引起了很多停产,所以不知道是否提高安全就能使生产率上升。而利润关系到的问题更多,比如成本,新的计划的花销将很大程度上影响这一点。

使用道具 举报

Rank: 2

声望
9
寄托币
139
注册时间
2009-4-2
精华
0
帖子
28
24
发表于 2009-4-26 22:24:24 |只看该作者
230.The following appeared in a recommendation from the planning department of the city of Transopolis.
     "Ten years ago, as part of a comprehensive urban renewal program, the city of Transopolis adapted for industrial use a large area of severely substandard housing near the freeway. Subsequently, several factories were constructed there, crime rates in the area declined, and property tax revenues for the entire city increased. To further revitalize the city, we should now take similar action in a declining residential area on the opposite side of the city. Since some houses and apartments in existing nearby neighborhoods are currently unoccupied, alternate housing for those displaced by this action will be readily available."
      十年前,作为一项综合城市改造项目的一部分,Transopolis市为了工业用途在快速路旁边改造了一大片严重低于标准的住房。随后,那里建立了若干工厂,犯罪率降低了,全市资产税收入增加。为进一步使城市重新繁荣,我们应该在城市另一侧的正在衰落的居住区采取类似的行动。由于现有邻近社区的一些住宅和公寓无人居住,这一行动产生的搬迁户的住所将有所保障。

substandard housing near the freeway 论断:为一进步搞活城市,我们应该在城市的居民正在减少的城市另一边采用同样的行动(把地用作工业)。因为那儿的一些房子和公寓目前没人住,这个计划需要的备用房已经有了。十年前,做为整个郊区搞活计划的一部分,T城把高速边的一些低质量房用于工业。结果那么建了很多工厂,犯罪率也下降了,整个城市的财税收入也上升了。
        前提,这边有地用于工业。没有证据证明这边有足够的空间发展工业,或者说发展工业适合该地区。如果说该地区附近有很多疗养院,sanitorium or park, 建工业就会影响该地区的环境。
        论据,论者引用十年前另一边的经验,但第一,时间相隔十年,经验是否能被借鉴值得怀疑。第二,另一边可能有大量质量不好的房屋,这些房屋迟早要被推倒用做别的用途,但是论者没有说这边的房屋是有是也主要是质量不好的房屋。如果不是,这是种浪费,且会引起房主不满。第三,犯罪率的下降和财税的上升与工业发展之间的因果关系不被保证。有可能该地区在这十年间制定了严格的法律,加派了更多的警察,并且还在发且比如商业、旅游业等项目。其实新建的工厂一般来说税交的都不多。
结论:就算发展工业好,条件是否具备也难说。闲置的空房不等于没人要,房主不一定会答应让别人来住。

231.The following appeared in a letter to the editor of a local newspaper.
     "The members of the town school board should not be reelected because they are not concerned about promoting high-quality education in the arts in our local schools. For example, student participation in the high school drama club has been declining steadily, and this year the school board refused to renew the contract of the drama director, even though he had received many awards for his original plays. Meanwhile, over $300,000 of the high school budget goes to athletic programs, and the head football coach is now the highest-paid member of the teaching staff."
      市学校委员会成员不应该再次当选,因为他们不关心在我们地区学校促进艺术教育的质量。举例而言,参加高中戏剧俱乐部的学生数量逐渐下降,今年校委拒绝与戏剧指导续约,尽管他以其原创剧作获得过很多奖项。同时,高中预算中超过三十万元用于体育项目,橄榄球总教练现在是全部教员中薪水最高的人。

school board are not promoting high-quality education 论断:镇上学校理事会不能再当选了因为他们不关心在当地学校推进高质量的艺术教育。比如说,参加学校戏剧俱乐部的学生一直在下降,今年他们拒绝了戏剧主任的重新任命,他可是获得了很多原创戏剧将。另外,越过三千万元的学校财政去了体育项目,足球教练的头现在是教职员中工资最高的。
        前提,不关心艺术教育。论剧是戏剧俱乐部,除了戏剧之外艺术教育还有很多内容。所以没有证据证明不关心艺术教育。俱乐部学生少了有可能是学生感兴趣的少了,或是主课实在太忙了。
        论据二,戏剧主任的再任命。首先获过奖不一定是好的教员,其次可能他提出了过高的要求,比如薪水高等等。
        论据三,足球队。教练工资和三千万元没有比较基础,也许其他艺术课程的投入也有这么多,或者其他地区的投入也是这样。不管怎样,体育教学也是很重要的。也许足球队给当地带来了很多荣耀。
结论:不关心艺术教育不等于不合格。毕竟它还仅仅是课外活动。应该看看学校在学生素质培养和知识教授上的情况。

232.The following appeared in a memo from the sales manager of Eco-Power, a company that manufactures tools and home appliances.
     "Many popular radio and television commercials use memorable tunes and song lyrics to call attention to the products being advertised. Indeed, a recent study of high school students showed that 85 percent could easily recognize the tunes used to advertise leading soft drinks and fast-food restaurants. Despite our company's extensive advertising in magazines during the past year, sales of our home appliances declined. Therefore, to boost company profits, we should now switch to advertisements featuring a distinctive song."
     很多受欢迎的广播和电视广告都使用易于记忆的音乐和歌曲来吸引人们对于广告商品的注意。确实,最近一次对于高中学生的研究发现85%的人能够轻易地识别畅销软饮料和快餐厅广告所用的音乐。尽管我们公司去年在杂志上做了大量广告,我们家居用品的销量还是下降了。因此,为增加公司利润,我们现在应该改为使用独特歌曲的广告。

memorable tunes and song lyrics being advertised 论断:我们应该转为用有特点的广告歌。因为最近对于高中学生的调查显示他们中85%的人能很容易地认出最流行的软饮料和快餐店的广告曲调。而我们公司过去几睥在杂志上投放了大量的广告,但销售却在下降。
        前提,销售下跌是因为广告。没有证据证明。可能是质量,款式一直没有跟上需求。
        前提,广告歌曲有效,论据的问题。首先调查是对高中生的,EP不可能希望高中生会是家庭用具的消费群体。而其消费群比如主妇等,是否也有对广告歌曲一样的反应,不被保证。其次,软饮料和家庭用品性质不一样,bought-once product/can make repeated buying.
结论:费用问题要考虑,广告歌曲请人写,还要通过电视广播广告,这些都比杂志的费用高。

233.The following appeared in a memo from the vice president of a company that builds shopping malls throughout the country.
"The surface of a section of Route 101, paved two years ago by McAdam Road Builders, is now badly cracked and marred by dangerous potholes. In another part of the state, a section of Route 66, paved by Appian Roadways more than four years ago, is still in good condition. Appian Roadways has recently purchased state-of-the-art paving machinery, and it has hired a new quality-control manager. Because of its superior work and commitment to quality, we should contract with Appian Roadways rather than McAdam Road Builders to construct the access roads for all our new shopping malls."
     两年前由McAdam筑路公司修筑的101公路的一段,表面现已严重开裂,并存在危险的坑洼。在同一州的另一个地区,四年多以前由Appian筑路公司修筑的66公路的一段现在依然状况良好。Appian公司最近购买了最先进的筑路机械,并雇佣了一名新的质量监控经理。由于Appian公司更优秀的工作和质量保证,我们应该和他们而不是McAdam公司签订合同修筑我们新的商业街前面的通道。

Route 101 badly cracked and marred by dangerous potholes. 论断:我们应该和A签合同建所有新商场的通道。因为它们的工作更好质量有承诺。M在两年前铺的101道现在已经严重损坏,而A四年前铺的66道现在还挺好。A现在买了新的机器,而且还雇了新的经理。
        前提,A修的路比M好。论据问题,可能101道在市中心,或是常有重型货车通过,再不然还有地面的地质问题,都是造成损毁严重的原因。66可能就容易多了。
        前提,A比M好。没有直接的证据证明A现在比M好,相反,新机器和新经理,都说明M和以前修66时不一样了,磨和问题,适应问题,都不是以前的经验能保证的。
结论:商场的活可能跟66完全不一样,另外就通过一次工程就得出永久结论,不公平。

234.The following appeared in a newspaper feature story.
      "There is now evidence that the relaxed manner of living in small towns promotes better health and greater longevity than does the hectic pace of life in big cities. Businesses in the small town of Leeville report fewer days of sick leave taken by individual workers than do businesses in the nearby large city of Mason City. Furthermore, Leeville has only one physician for its one thousand residents, but in Mason City the proportion of physicians to residents is five times as high. And the average age of Leeville residents is significantly higher than that of Mason City residents. These findings suggest that people seeking longer and healthier lives should consider moving to small communities."
      现有证据表明,在小城市的悠闲生活方式比在大城市快节奏的生活更有利于健康和长寿。小城镇Leeville的商业行业所上报的员工病假天数比邻近大城市Mason要少。而且,Leeville每千人只有一名医生,而Mason市医生的比例是Leeville的五倍。Leeville居民的平均年龄显著高于Mason居民。这些数据表明寻求更健康更长寿生活的人应该考虑移居到小型社区。

relaxed manner of living in small towns 论断:想长寿和生活健康的人应该考虑搬到小社区去。因为小镇L每年每位工人报告的病假比隔壁的大城市M少。另外L一千个居名只有一位医生,但M医生和居民的比例有五倍之高。而且L居民的寿命也明显比M高。
        前提,小镇居民身体好。病假没意义,别忘了有人goldbrick。小镇居民不需要医生。没证据,医生少可能医生不够,小镇的人还得跑大老远去找医生。而城市里医生多,很方便。
        前提,小镇居民长寿。值得怀疑,小镇可能经常会有退休的老人去居住,大城市里都是忙碌的年轻人。
结论:去哪里生活还有很多因素限制,又想长寿和健康,又想事业有成,就得权衡权衡了。

235.The following appeared in a memorandum from the general manager of KNOW radio station.
      "Several factors indicate that radio station KNOW should shift its programming from rock-and-roll music to a continuous news format. Consider, for example, the number of older people in our listening area has increased dramatically, while the total number of our listeners has recently declined. Also, music stores in our area report decreased sales of recorded music. Finally, continuous news stations in neighboring cities have been very successful, and a survey taken just before the recent election shows that local citizens are interested in becoming better informed about politics."
     若干因素表明KNOW电台应该把它的节目从摇滚乐转成连续的新闻节目。举例来说,我们覆盖范围内的老年人数量显著增加,而总的听众数量最近减少了。而且,我们地区音像店中录制音乐的销量下降。最后,邻近城市的连续新闻节目非常成功,最近大选前组织的一次调查显示,本地居民对于及时的政治报导很感兴趣。

shift program from rock-and-roll to a continuous news format 论断:K应该别再播摇滚乐而改成滚动新闻。因为听众地区的老年人急剧上升,而听众却在下降。另外,我们地区的音乐商店报告说音乐唱片的销售在下降。最后隔壁城的滚动新闻电台挺成功,一份在最近选举前做的调查显示当地居民对更了解政治有兴趣。
        前提,没什么人听音乐节目了。没有证据表明。老年观众上升和听众人数下降都不说明问题,因为我们不知道基数是多少。另外观众人数下降有可能跟近期节目做得不好有关系,电台应该努力把音乐节目质量提高,吸对音乐感兴趣的人。而对于老年人,他们也许不爱听摇滚乐,但也没有证据说他们会喜欢政治新闻。
        前提,没有听音乐了。音乐商店和电台不一样,一个花钱买片子,一个免费听歌曲。商店销售下降可能跟经济萧条购买力下降以及最近没有什么新唱片发行有关系,而这种情况可能正好会使听电台音乐的人增加。
        前提,滚动新闻节目好。没有证据说明新闻的观众比音乐的观众多,而且选举前情况特殊这种形势可能会很快改变。
结论:电台还是应该尽量按照现有的路子发展,因为毕竟积累了一定的经验和影响力,转变动静太大。

236.The following appeared as a letter to the editor of a local Solano newspaper.
     "The Solano school district should do away with its music education programs. After all, music programs are not especially popular in our schools: only 20 percent of our high school students participated in music programs last year. Furthermore, very few of the Solano district's college-bound students are interested in majoring in music at the university level. Also, when the school district of our neighboring town of Rutherford eliminated its music programs two years ago, the overall grade point average in the high school increased by 10 percent the next year. Surely the money spent on Solano music programs could be better used to improve the quality of traditional academic courses."
      Solano校区应该取消其音乐课程。无论如何,音乐课程在我们学校并不特别受欢迎:去年我们高中的学生只有20%参加了音乐课。而且,Solano区要考大学的学生中只有很少的人想在大学学习音乐。并且当我们的邻镇Rutherford两年前取消了音乐课的时候,其高中的综合成绩在随后一年中上升了10%。显然我们用于Solano音乐课程的开支可以更好地被用于提高传统课程的质量。

should do away with its music education programs 论断:应该取消音乐教育课,把花在音乐课上的钱花在提高传统学术课质量上应该更好。因为音乐在我们学校中不是特别流行:只有20%的高中学生去年上了音乐课。另外,要上大学的学生中只有很少有兴趣在大学读音乐专业。而且当隔壁镇两年前取消音乐课时,高中的整体成绩上第二年升了10%。
        前提,学生对音乐不感兴趣。首先没有学生说这样的话。其次,很少有学生想大学读音乐专业,不表示很少有学生感兴趣。专业意味着工作,学生可能会把它当兴趣。第三,20%的学生上音乐课,这不是个小数目,不应该忽视他们的需要。
        论据,隔壁镇的分数上升不一定是因为取消了音乐。可能是教师水平提高了,可能是打分不那么严了lower demand in scoring system.
结论:音乐虽然不是主课但仍然重要,传统课上花钱再多也不见得会出效果。

237.The following appeared as part of an article in a local Beauville newspaper.
      "According to a government report, last year the city of Dillton reduced its corporate tax rate by 15 percent; at the same time, it began offering relocation grants and favorable rates on city utilities to any company that would relocate to Dillton. Within 18 months, two manufacturing companies moved to Dillton, where they employ a total of 300 people. Therefore, the fastest way for Beauville to stimulate economic development and hence reduce unemployment is to provide tax incentives and other financial inducements that encourage private companies to *relocate here."
     根据一项政府报告,去年Dillton市把它的营业税率降低了15%;同时,该市给予迁移至Dillton的公司一定的搬迁费和相应城市设施使用费的优惠。在18个月内,两家生产商搬迁到了Dillton,他们在那里一共雇佣了300名工人。因此,Beauville市刺激经济发展从而降低失业率最快的方法就是提供税率以及其他经济上的优惠来鼓励私有企业搬迁到本地。

reduced its corporate tax rate 论断:B刺激经济发展降低失业率的最快方法是提供税收刺激和其它财政刺激鼓励私人企业再创业。因为D城去年把企业税降了15%,同时他们开始向所有到D来创业的企业提供资金和优惠的城市设备使用价格。18个月里,两家制造企业搬去了D,它们一共雇了300个人。 论断都从D城例子得来。
        两城情况不一样,地理位不同、发展水平不同、居民不同等等等。D城以前的税率不明,有可能降完了还比B高。
        两家公司不一定是因为低税来D的,有可能这里的潜在市场被它们看重,有可能其它自己的原因。
        D城的经济发展和失业问题解决得怎么样不知道,300个人的就业少了点吧。而且搞不好这三百人以前都是有工作的。
结论:首先D城例子里没说两家公司是私人企业,私人企业就算有刺激不一定能有能力再创业。其次,最快的方法,你就说了这一个呢。

238.The following appeared in a memorandum from the president of Mira Vista College to the college's board of trustees.
       "At nearby Green Mountain College, which has more business courses and more job counselors than does Mira Vista College, 90 percent of last year's graduating seniors had job offers from prospective employers. But at Mira Vista College last year, only 70 percent of the seniors who informed the placement office that they would be seeking employment had found full-time jobs within three months after graduation, and only half of these graduates were employed in their major field of study. To help Mira Vista's graduates find employment, we must offer more courses in business and computer technology and hire additional job counselors to help students with their resumès and interviewing skills."
      邻近的Green Mountain学院拥有比Mira Vista学院更多的商业课程和就业顾问,去年其毕业生有90%得到了公司的工作机会。但去年在Mira Vista,所有通知了就业办公室说他们要就业的毕业生中只有70%的人在毕业后三个月内找到全职工作,而且这些毕业生只有一半在本专业就业。为帮助Mira Vista的毕业生找到工作,我们必须提供更多的商业和计算机技术课程,并雇佣更多的就业顾问来帮助学生提高准备简历和面试的技巧。

more business courses and more job counselors 论断:我们必须提供更多的商业课和计算机技术,并雇用更多的职业顾问帮助学生做好简历和面见技巧。因为隔壁的G校比我们多有这些多西,去年他们90%的毕业生从潜在雇主那里拿到了工作offer,不过M只有70%报告办公室会去找工作的高年级学生在毕业后三个月里找到了全职工作,而且只有一半的学生工作与专业相关。
        前提,我们的学生不好找工作。首先70%和90%不说明问题,学生数目如果相差很大的话。其次,M校的学生拿到OFFER,没说去,可能工作很不好。而我们学校的学生70%找到的都是好工作。
        前提,提供商业课程会使学生好找工作。没有证据证明找到工作的学生是因为商业课程多。如果说毕业生大多工作与专业不对口,在一家企业工作是否就是从商呢?那要是卖书的公司,算是在干什么的。M校可能是商科学校,而我们主要是理科。事实上专业不同对课程需要也是不同。
结论:雇用职业顾问可能能够使学生更好地表现自己,但很多不是面试技巧和简历能解决的。还得看实力。

239.The following appeared as an editorial in the local newspaper of Dalton.
     "When the neighboring town of Williamsville adopted a curfew four months ago that made it illegal for persons under the age of 18 to loiter or idle in public places after 10 p.m., youth crime in Williamsville dropped by 27 percent during curfew hours. In Williamsville's town square, the area where its citizens were once most outraged at the high crime rate, not a single crime has been reported since the curfew was introduced. Therefore, to help reduce its own rising crime rate, the town of Dalton should adopt the same kind of curfew. A curfew that keeps young people at home late at night will surely control juvenile delinquency and protect minors from becoming victims of crime."
     4个月前当邻近城市Williamsville实行了宵禁,规定18岁以下人员在晚上10点以后在公共场所闲逛和逗留被视为非法的时候,Williamsville在宵禁时间内的青少年犯罪下降了27%。在Williamsville的市镇广场,一个曾经因发生在这里的高犯罪率而导致市民反感的地方,在宵禁实施以来没有一起犯罪被上报。因此,为降低日益增长的犯罪率,Dalton市也应该实行同样的宵禁。把青少年留在家中的宵禁肯定能控制青少年犯罪并保护未成年人不成为犯罪的牺牲品。

illegal under 18 to loiter or idle in public places after 10 p.m. 论断:D城应该采用同样的宵禁政策。因为让年轻人晚上必须呆在家里一定能控制青少年犯罪并使弱者不会成为犯罪的牺牲品。因为隔壁W四个月前采用了这样的政策,让18岁以下的人晚10点以后不得在公众场所闲逛,少年犯罪在宵禁期间降了27%。在W广场,以前臭名昭著的高儿犯罪场所,从那儿以后没有报告过一起犯罪。
        前提,D城的青少年犯罪是主要的。没有证据证明。
        前提,宵禁管用。论据的问题。首先宵禁期间犯罪少了,不证明整个少了。其次D城的广场上没有犯罪,不证明别的地方没有了。第三没有报告犯罪,也可能是还没有发现。non-open way.
结论:有人说D城的晚上不安全吗?

240.The following appeared in a memo written by a dean at Buckingham College.
     "To serve the housing needs of our students, Buckingham College should build a new dormitory. Buckingham's enrollment is growing and, based on current trends, should double over the next fifty years, thus making existing dormitories inadequate. Moreover, the average rent for an apartment in our town has increased in recent years. Consequently, students will find it increasingly difficult to afford off-campus housing. Finally, an attractive new dormitory would make prospective students more likely to enroll at Buckingham."
     为满足我们学生的住房需求,Buckingham学院应该建造一幢新的宿舍。Buckingham的报名人数正在增加,而且按照现有趋势,报名人数将会在未来50年中增加一倍,从而导致现有住宅不能满足要求。而且,我们镇上公寓的平均租金在近几年间上涨了。因此,学生将会越来越发现难于负担校外住宿费用。最后,一幢引人注目的新宿舍将会使未来的学生更愿意报考Buckingham。

enrollment is growing 论断:B校应该建一个新的宿舍。因为注册数在上升,依目前的趋势今后五十年里会翻番,那么现在的宿舍也就不够。另外,镇上租房子的钱每年都在上涨。结果学生会发现在校外找房子越来越难,最后漂亮的新宿舍会提高注册数。
        前提,现在需要一个新宿舍。没有证据证明现在宿舍不够。论者说五十年后,太多虑了。不足对目前的行为给予任何参考。每年也就增长2%。
        前提,房租上涨学生会愿意在学校住宿舍。房租上涨是很正常的,比如说存在通货膨胀,当地经济发展比较快。而且一般房租上涨说明该地各方面的工资和特价水平都在上涨,包括学费和住宿费。所以,学生可能还是愿意住在校外。
结论:宿舍怎么可能是吸引学生的主要原因呢。

241.The following appeared in a memo at the XYZ company.
      "When XYZ lays off employees, it pays Delany Personnel Firm to offer those employees assistance in creating resumès and developing interviewing skills, if they so desire. Laid-off employees have benefited greatly from Delany's services: last year those who used Delany found jobs much more quickly than did those who did not. Recently, it has been proposed that we use the less-expensive Walsh Personnel Firm in place of Delany. This would be a mistake because eight years ago, when XYZ was using Walsh, only half of the workers we laid off at that time found jobs within a year. Moreover, Delany is clearly superior, as evidenced by its bigger staff and larger number of branch offices. After all, last year Delany's clients took an average of six months to find jobs, whereas Walsh's clients took nine."
     当XYZ裁员的时候,它雇佣了Delany人事公司在这些下岗员工有需要的时候为他们在写简历和提高面试技巧方面提供帮助。下岗员工从Delany的服务中极大受益:去年获得了Delany帮助的员工找工作比没有找Delany的员工快的多。最近,有提案提出用收费较少的Walsh人事公司代替Delany。这将是一个错误,因为在8年前,XYZ使用的就是Walsh,我们当时裁减的员工只有一半在一年之内找到了工作。而且,Delany显然更好,理由是它的员工更多,分支机构也更多。不论如何,去年Delany的客户平均用六个月找到工作,而Walsh的客户平均用了9个月。

offer employees assistance in resumés and interviewing skills 论断:改用便宜的W而不用D是错误的。因为八年前我们就用了W,只有一半被裁的员工在一年里找到了工作。另外D显然更好,他们人多分支机构也多就是证据。去年D的顾客平均花六个月找工作,而W的顾客要九个月。
        前提,W不好。论据用的八年前,八年,够发生很多事情了。另外,被裁掉的员工的状况也应该考虑进去,也许以前我们裁员的标准非常严格,非常不合格的才裁掉。不像现在公司可能状况不好,挺好的员工也给裁了。
        前提,D好。首先人多机构多不代表效率高服务好,反而可能会增加成本。第二员工的经历,不可否认,接受这项服务的人会比不接受的人有更强烈的马上找到工作的愿望。所以找的时候,会比别人格外卖力。最后,顾客平均找工作的时间,工作性质、顾客能力都会影响。
结论:既然W便宜,可能公司最近是不是财政紧张。若是换换也行,总比没有强吧。

242.The following appeared as an editorial in the student newspaper of Groveton College.
      "To combat the recently reported dramatic rise in cheating among college and university students, these institutions should adopt honor codes similar to Groveton's, which calls for students to agree not to cheat in their academic endeavors and to notify a faculty member if they suspect that others have cheated. Groveton's honor code replaced an old-fashioned system in which students were closely monitored by teachers and an average of thirty cases of cheating per year were reported. The honor code has proven far more successful: in the first year it was in place, students reported twenty-one cases of cheating; five years later, this figure had dropped to fourteen. Moreover, in a recent survey conducted by the Groveton honor council, a majority of students said that they would be less likely to cheat with an honor code in place than without."
       为解决最近上报的大学生作弊现象显著增加的问题,大学和学院应该采取和Groveton学院类似的诚信制度,该制度要求学生同意在学业中不作弊,并且当他们怀疑别人作弊的时候通知老师。Groveton的诚信制度代替了原有的学生被老师严密监视的老体制,在老体制中平均每年有30起作弊被上报。该制度被证明成功的多:在它实施的第一年,学生上报了21起作弊;五年后,这一数值下降到了14起。而且,在最近一次由Groveton诚信委员会组织的调查中,大部分学生说有了诚信制度他们更加不太可能作弊。

dramatic rise in cheating 论断:为打击最近报告忽剧上升的做弊行为,各校应该采用和G校一样的H办法,这是让学生同意不在学术课程上做弊,并若怀疑有人做弊就报告老师。这个办法替换了一个过时的系统,就是让老师近距离地监督,而且平均每年都会有30起做弊报告。这个办法很成功,第一年21起,五年后,这个数字降到了14起。另外,最近的一份调查说大多数学生说有H他们比没有更不可能做弊。
        同意不做弊不等于就不做弊。没有学生会说自己就是要做弊。
        这个办法有问题,以怀疑为基础,但怀疑表示猜测,会不准确。有可能会毁坏学生关系。
        论据,报告的做弊数从五年前的21降到现在14,并不说明问题。首先,没有说趋势一直在下降,有可能去年是30多起。其次有可能学生已经对这个失去耐心,没有人报告了。而老师却还依赖学生之间的监督。
        调查,可能学生说的是本来也不想做弊。

使用道具 举报

Rank: 2

声望
9
寄托币
139
注册时间
2009-4-2
精华
0
帖子
28
25
发表于 2009-4-26 22:26:34 |只看该作者
终于粘完了。可能是因为级别太低,所以不能上传附件,只能这样了。给大家带来了一些不便,真是不好意思啊!:D

使用道具 举报

Rank: 1

声望
0
寄托币
81
注册时间
2008-5-28
精华
0
帖子
1
26
发表于 2009-4-27 02:20:27 |只看该作者
收了,谢谢!!!

使用道具 举报

Rank: 1

声望
5
寄托币
41
注册时间
2009-5-16
精华
0
帖子
1
27
发表于 2009-5-16 11:49:16 |只看该作者
谢谢搂住了.我下了几个,都不好使,就你这个直观

使用道具 举报

Rank: 4

声望
65
寄托币
2997
注册时间
2008-2-20
精华
0
帖子
67

GRE斩浪之魂 GRE梦想之帆

28
发表于 2009-5-17 22:38:10 |只看该作者
XIELE

使用道具 举报

Rank: 1

声望
0
寄托币
35
注册时间
2009-2-25
精华
0
帖子
3
29
发表于 2009-5-18 10:49:35 |只看该作者
niu ren

使用道具 举报

Rank: 1

声望
0
寄托币
28
注册时间
2009-3-14
精华
0
帖子
2
30
发表于 2009-7-5 19:11:21 |只看该作者
用了资源,感谢一下!谢谢!:loveliness:

使用道具 举报

RE: 可直接打印的242道argument题目及分析 [修改]
您需要登录后才可以回帖 登录 | 立即注册

问答
Offer
投票
面经
最新
精华
转发
转发该帖子
可直接打印的242道argument题目及分析
https://bbs.gter.net/thread-945516-1-1.html
复制链接
发送
报offer 祈福 爆照
回顶部